NCLEX Neuro 1 of 2

¡Supera tus tareas y exámenes ahora con Quizwiz!

Glasgow Coma Scale (GCS) scoring

-13-15 minor head trauma -9-12 moderate head trauma -<8 severe head trauma

Cerebral Angiogram

-assesses blood flow; involves catheter insertion, dye injection and sequential x-ray -know if pt has allergy to shellfish, iodine or has blood clotting disorder; these allergies require different contrast media -observe for catheter site bleeding 8-12hrs postprocedure

Electroencephalography (EEG)

-assesses electrical activity of brain noninvasively; detection of seizures, behavioural changes, sleep disorders; electrodes placed on head -review meds to be taken by pt with provider -shampoo hair--no gels, oils, sprays -pt to be sleep deprived if possible to increase intracranial stress

Intracranial Pressure Monitoring (ICP)

-continuous monitoring of intracranial pressure by an invasive transducer -rarely used; usually for comatose pts; GCS <8 -Nursing Priority: prevent infections from occuring.

ANTI-EPILEPTIC DRUGS

...

Carbamezapine

...

Compare and contrast the different types of seizures in children.

...

Compare the symptoms of PDD, Autism and Down syndrome.

...

Describe IQ related to MR.

...

Describe a nursing care for the patient with a brain tumor.

...

Discuss at risk population for PDD, Autism and Down syndrome.

...

Discuss community resources for home health care, meals, equipment, respite care, social services, professional or lay support, and shelters.

...

Discuss educational challenges in the care of the MR.

...

Discuss factors that may contribute to brain tumor formation.

...

Discuss health promotion techniques and teaching to be done for a patient and family with a seizure disorder.

...

Discuss local and national resources available to patients with Alzheimer's disease.

...

Discuss nursing care of the child with MR.

...

Discuss nursing implications regarding lab values for the patient prescribed anticonvulsants.

...

Discuss prenatal, perinatal, and postnatal causes of MR.

...

Discuss the importance of early intervention for the child with PDD, Autism and Down syndrome.

...

Discuss the local and national community resources available for patients with seizure disorders.

...

Discuss the nursing care of an individual with a seizure disorder.

...

Discuss the nursing care of the newborn and child with Down syndrome.

...

Do not abruptly discontinue use of the antiparkinsonism drugs. Can cause malignant-like syndrome.

...

IDIOPATHIC LOCALIZATION RELATED EPILEPSY

...

List two community resources for individuals and families of persons with CP.

...

List two community resources for individuals and families of persons with MR.

...

Coup-contrecoup

..., These injuries to the cerebrum occur when a blow to the head caused the brain to shift towards the area of impact and injure itself by hitting the inner surface of the skull, and then rebounding in the opposite direction and injuring itself again by hitting the skull on the opposite side of the skull where the original blow was delivered.

At what time of day should the nurse encourage a client with Parkinson's disease to schedule the most demanding physical activities to minimize the effects of hypokinesia? 1. Early in the morning, when the client's energy level is high. 2. To coincide with the peak action of drug therapy. 3. Immediately after a rest period. 4. When family members will be available.

.2. Demanding physical activity should be performed during the peak action of drug therapy. Clients should be encouraged to maintain independence in self-care activities to the greatest extent possible. Although some clients may have more energy in the morning or after rest, tremors are managed with drug therapy.

A client with Parkinson's disease asks the nurse to explain to his nephew "what the doctor said the pallidotomy would do." The nurse's best response includes stating that the main goal for the client after pallidotomy is improved: 1. Functional ability. 2. Emotional stress. 3. Alertness. 4. Appetite.

1

What is supratentorial cranial surgical approach? What are the interventions involved?

1) It is an incision behind the hairline 2) Main head of bed at 30-45 degrees with neck in neutral alignment. 3) Position patient on either side or back

What items come in the negative pressure sterile kit?

1) Wound vacuum unit 2) Sterile foam sponge kit 3) Measuring tool 4) NaCl 5) Skin prep 6) Gloves and PPE 7) A pad, towel and gauze pads

The nurse is admitting a person who has had a sudden loss of eyesight. On assessing this client, the nurse finds that the client is disoriented. The nurse will most suspect which of the following about the disorientation? 1. Disorientation is a normal reaction to sudden blindness. 2. Compensatory behavior to eyesight loss includes disorientation. 3. Client will compensate for the eyesight loss within 48 hours. 4. Disorientation is a symptom of the cause of sudden eyesight loss.

1. Disorientation is a normal reaction to sudden blindness; Sudden loss of eyesight can result in disorientation. With gradual loss of sensory function, individuals often develop behaviors to compensate for loss, whereas with sudden loss, the compensatory behavior often takes days or weeks to develop.

What are the symptoms of subarachnoid hemorrhage?

1. Exploding headache 2. Loss of consciousness 3. Seizure 4. Sluggish pupil reaction 5. Photophobia 6. Elevated blood pressure, heart rate, respiration 7. One sided weakness 8. Lethargic

Two types of seizures (define)

1. Focal - initial activation in 1 part of brain (EEG and clinical) 2. Generalized - initial involvement of both hemispheres

What is the proper way to converse with hearing impaired patients?

1. Get your patient's attention 2. Ask permission to turn off television and reduce noise 3. Face patient at eye level 4. Make sure light is on face 5. If patient has hearing aid, make sure it is in and turned on and functioning properly. 6. Use simple sentences, speak slowly and avoid shouting. 7. Supplement with gestures 8. Avoid jokes, slang and ask for oral feedback to assess understanding.

An 85-year-old client has impaired hearing. When creating the care plan, which intervention should have the highest priority? 1. Obtaining an amplified telephone. 2. Teaching the importance of changing his position. 3. Providing reading material with large print. 4. Checking expiration dates on food packages

1. Obtaining an amplified telephone; The amplified telephone helps with hearing and provides a means for communicating with others. Option 2 refers to a tactile impairment. Option 3 relates to a visual impairment, and option 4 an olfactory impairment.

What are the two types of GLAUCOMA?

1. Primary open-angle = chronic 2. Closed angle = acute and painful

What are the signs and symptoms of MYASTHENIA GRAVIS?

1. bobblehead 2. ptosis 3. diplopia 4. slow speech 5. frowning 6. decreased tongue movement 7. drooling 8. pupils slowly react 9. increased frowning 10. decreased chewing

Normal ICP

10-15mmHg

Number of thoracic nerves

12

A client has had multiple sclerosis (MS) for 15 years and has received various drug therapies. What is the primary reason why the nurse has found it difficult to evaluate the effectiveness of the drugs that the client has used? 1. The client exhibits intolerance to many drugs. 2. The client experiences spontaneous remissions from time to time. 3. The client requires multiple drugs simultaneously. 4. The client endures long periods of exacerbation before the illness responds to a particular drug.

2

Which of the following is not a typical clinical manifestation of multiple sclerosis (MS)? 1. Double vision. 2. Sudden bursts of energy. 3. Weakness in the extremities. 4. Muscle tremors.

2

Which of the following is essential when caring for a client who is experiencing delirium? 1. Controlling behavioral symptoms with low-dose psychotropics. 2. Identifying the underlying causative condition or illness. 3. Manipulating the environment to increase orientation. 4. Decreasing or discontinuing all previously prescribed medications.

2. The most critical aspect when caring for the client with delirium is to institute measures to correct the underlying causative condition or illness. Controlling behavioral symptoms with low-dose psychotropics, manipulating the environment, and decreasing or discontinuing all medications may be dangerous to the client's health.

While assessing a client diagnosed with dementia, the nurse notes that her husband is concerned about what he should do when she uses vulgar language with him. The nurse should: 1. Tell her that she is very rude. 2. Ignore the vulgarity and distract her. 3. Tell her to stop swearing immediately. 4. Say nothing and leave the room.

2. Vulgar language is common in clients with dementia when they are having trouble communicating about a topic. Ignoring the vulgarity and distracting her is appropriate. Telling the client she is rude or to stop swearing will have no lasting effect and may cause agitation. Just leaving the room is abandonment that the client will not understand.

For the client who is experiencing expressive aphasia, which nursing intervention is most helpful in promoting communication? 1. Speaking loudly. 2. Using a picture board. 3. Writing directions so client can read them. 4. Speaking in short sentences.

2. Expressive aphasia is a condition in which the client understands what is heard or written but cannot say what he or she wants to say. A communication or picture board helps the client communicate with others in that the client can point to objects or activities that he or she desires.

While performing a history, the nurse assesses sensory perceptions such as: 1. Kinesthetic perception 2. Mental status 3. Deep tendon reflexes 4. Cranial nerves

2. Mental status; Mental status is assessed while performing a history. Kinesthetic perception, deep tendon reflexes, and cranial nerves are assessed during the physical exam, not the history.

Which of the following interventions would most help reduce olfactory stimuli for a client who is hospitalized with a draining wound and is sensory overloaded? 1. Use strong disinfectants to clean the wound. 2. Place liquid deodorant on a gauze near the clean, covered wound. 3. Spray strong floral room deodorizer in room to mask wound odor. 4. Use strong disinfectant on everything possible in room.

2. Place liquid deodorant on a gauze near the clean, covered wound; The odor from a draining wound can be minimized by keeping the dressing dry and clean and applying a liquid deodorant on a gauze near the wound.

Your assigned first day post-operative client, who has a new colostomy, seems to worry a lot and has symptoms of sensory overload. Which of the following client goals, if met, would most contribute to reducing sensory overload for this client? 1. Will not sleep or nap during the day. 2. Will report pain at 4 or less on a 10-point scale. 3 .Will attend classes on colostomy care. 4. Will look at colostomy during colostomy care.

2. Will report pain at 4 or less on a 10-point scale; People who have sensory overload may appear fatigued. They cannot internalize new information and experience cognitive overload as a result of everything that is happening to them. Such factors as pain, lack of sleep, and worry can also contribute to sensory overload.

21. The nurse is assessing the adaptation of the female client to changes in functional status after a brain attack (stroke). The nurse assesses that the client is adapting most successfully if the client: a. Gets angry with family if they interrupt a task b. Experiences bouts of depression and irritability c. Has difficulty with using modified feeding utensils d. Consistently uses adaptive equipment in dressing self

21. Answer D. Clients are evaluated as coping successfully with lifestyle changes after a brain attack (stroke) if they make appropriate lifestyle alterations, use the assistance of others, and have appropriate social interactions. Options A, B, and C are not adaptive behaviors.

27. Female client is admitted to the hospital with a diagnosis of Guillain-Barre syndrome. The nurse inquires during the nursing admission interview if the client has history of: a. Seizures or trauma to the brain b. Meningitis during the last 5 years c. Back injury or trauma to the spinal cord d. Respiratory or gastrointestinal infection during the previous month.

27. Answer D. Guillain-Barré syndrome is a clinical syndrome of unknown origin that involves cranial and peripheral nerves. Many clients report a history of respiratory or gastrointestinal infection in the 1 to 4 weeks before the onset of neurological deficits. Occasionally, the syndrome can be triggered by vaccination or surgery.

A client is being switched from levodopa (l-dopa) to carbidopa-levodopa (Sinemet). The nurse should monitor for which of the following possible complications during medication changes and dosage adjustment? 1. Euphoria. 2. Jaundice. 3. Vital sign fluctuation. 4. Signs and symptoms of diabetes.

3

A client with multiple sclerosis (MS) is experiencing bowel incontinence and is starting a bowel retraining program. Which strategy is inappropriate? 1. Eating a diet high in fiber. 2. Setting a regular time for elimination. 3. Using an elevated toilet seat. 4. Limiting fluid intake to 1,000 mL/ day.

4

A new medication regimen is ordered for a client with Parkinson's disease. At which time should the nurse make certain that the medication is taken? 1. At bedtime. 2. All at one time. 3. Two hours before mealtime. 4. At the time scheduled.

4

The right hand of a client with multiple sclerosis trembles severely whenever she attempts a voluntary action. She spills her coffee twice at lunch and cannot get her dress fastened securely. Which is the best legal documentation in nurses' notes of the chart for this client assessment? 1. "Has an intention tremor of the right hand." 2. "Right-hand tremor worsens with purposeful acts." 3. "Needs assistance with dressing and eating due to severe trembling and clumsiness." 4. "Slight shaking of right hand increases to severe tremor when client tries to button her clothes or drink from a cup."

4

A client with a hemorrhagic stroke is slightly agitated, heart rate is 118, respirations are 22, bilateral rhonchi are auscultated, SpO2 is 94%, blood pressure is 144/ 88, and oral secretions are noted. What order of interventions should the nurse follow when suctioning the client to prevent increased intracranial pressure (ICP) and maintain adequate cerebral perfusion? 1. Suction the airway 2.hyperoxygenate 3.suction the mouth 4. Provide sedation

4,2,1,3

A hospitalized client is disoriented and believes she is in a train station. Which response from the nurse is the most appropriate? 1. "You wouldn't be getting a bath at the train station." 2. "Let's finish your bath before the train arrives." 3. "Don't you know where you are?" 4. "It may seem like a train station sometimes, but this is Hogwarts."

4. "It may seem like a train station sometimes, but this is Hogwarts."; Option 4 is the only response that helps orient the client and treats the client with respect.

Glioma

42% of all brain tumors are this type. 77% of malignant tumors are this type.

Number of cervical nerves

8

Ethosuximide (Zarontin)

Approved for: -Absence Side effects: -Nausea, sedation, BM suppression, rash

Lamotrigine (Lamictal)

Approved for: -Focal (when converting from another AED) -Generalized tonic-clonic -Lenox-Gastaut Side effects: -Stevens Johnson syndrome, dizziness, sedation, diplopia, headache, nausea

Discuss nursing care of the patient with neuromuscular disorder.

Assess ability to swallow, chew, and taste Assess weight daily Assess bowel sounds Assess/monitor changes in vital signs Assess respiratory rate, character, and use of accessory muscle Administer oxygen as ordered Administer medications as ordered Teach patient about disease process

Where do ulcers start?

At bone and work their way through the skin

How long should a patient wear a negative pressure system?

At least 22 hours a day to aid in wound healing.

In the Snellen chart, the NUMERATOR represents the A. distance a "normal" person could read a particular line B. distance that a person stands away from the chart C. % of the # of letters a person can read on a line D. # of feet that a person has to move forward to read a line

B. distance that a person stands away from the chart the denominator is the distance a "normal" person could read a particular line

The red light reflex is caused by the A. refraction of light off the conjunctiva B. reflection of light off the inner retina C. reflection of light off the choroids layer D. condenstaion of light as it passes through the aqueous humor.

B. reflection of light off the inner retina

4

Based on the life changes index which individual would have an increased possibility of illness in the near future? 1. A 25 year old man who recently married his high school sweetheart 2. 1 35 year old man fired from his job 3. a 40 year old woman beginning a nursing program 4. 50 year old woman whos husband died a month ago

Why should the patient with trigeminal neuralgia chew on the unaffected side? What is the outcome?

Because the unaffected side is painful. Patient would lose weight. The outcome should be weight gain as a result of chewing on unaffected side and eating a high calorie, high protein diet like custard, milk and eggs.

How do you describe a CATARACT vision?

Blurred, but vision can return to normal.

Formula for CPP (cerebral perfusion pressure)

CPP= MAP - ICP where MAP is mean arterial pressure and ICP is intracranial pressure

Semicomatose

Can be aroused by extreme or repeated stimuli

Topirimate

Can cause acidosis so increased risk of kidney stones; wt loss (popular) BUT can cause memory loss esp words

If the unit alarm on a negative pressure system indicates a lack of seal what should you do?

Check the patients skin folds and drape around the tubing assembly or curved body areas.

a. idiopathic generalized epilepsy (2 exs)

Childhood Absence Epilepsy Juvenile Absence Epilepsy

More about pharmacokinetics and complex kinetics of phenytoin

Complex kinetics - from right below therapeutic level then shoots up way beyond) - could make someone v. toxic quickly ⇒ serum level starts to rise exponentially after reaching a certain dose (different dose for different people)

Frontal Lobe

Concentration, abstract thought, information storage/memory, motor function, speech motor function, affect, judgement, personality, inhibitions.

If the wound using a negative pressure system appears dry what should you do?

Consider requesting a pressure reduction. If very wet, consider increase in pressure.

The healthcare provider orders 15 mg IV of ketorolac (Toradol) for a patient who has recently undergone a spinal fusion. The nurse has a 5 milliliter (mL) ampule containing 60 mg of ketorolac. How many milliliters of ketorolac does the nurse need to withdraw from the syringe?

Correct Answer: 1.25

An industrial nurse is conducting a class to teach methods to prevent back pain. What is the correct of steps for lifting heavy objects? Choice 1. Spread the feet apart to broaden the base of support. Choice 2. Use large leg muscles to push when lifting. Choice 3. Stand as closely as possible to the object to be moved. Choice 4. Rolling or pushing the obect insrtead of lifting.

Correct Answer: 2,3,1,4 Rationale: In teaching prevention of back injuries the nurse would incorporate principles of proper body mechanics, which are work as close to the object as possible, spread feet apart, use large leg muscles for leverage. Sometimes rolling or pushing will enable movement of a heavy object.

A patient with a spinal cord injury (SCI) is admitted to the unit and placed in traction. Which of the following actions is the nurse responsible for when caring for this patient? Select all that apply. 1. modifying the traction weights as needed 2. assessing the patient's skin integrity 3. applying the traction upon admission 4. administering pain medication 5. providing passive range of motion

Correct Answer: 2,4,5 Rationale: The healthcare provider is responsible for initial applying of the traction device. The weights on the traction device must not be changed without the order of a healthcare provider. When caring for a patient in traction, the nurse is responsible for assessment and care of the skin due to the increased risk of skin breakdown. The patient in traction is likely to experience pain and the nurse is responsible for assessing this pain and administering the appropriate analgesic as ordered. Passive range of motion helps prevent contractures; this is often performed by a physical therapist or a nurse.

Which of the following nursing actions is appropriate for preventing skin breakdown in a patient who has recently undergone a laminectomy? 1. Provide the patient with an air mattress. 2. Place pillows under patient to help patient turn. 3. Teach the patient to grasp the side rail to turn. 4. Use the log roll to turn the patient to the side.

Correct Answer: 4 Rationale: A patient who has undergone a laminectomy needs to be turned by log rolling to prevent pressure on the area of surgery. An air mattress will help prevent skin breakdown but the patient still needs to be turned frequently. Placing pillows under the patient can help take pressure off of one side but the patient still needs to change positions often. Teaching the patient to grasp the side rail will cause the spine to twist, which needs to be avoided.

A patient is admitted with signs of a stroke (CVA). On admission, vital signs were blood pressure 128/70, pulse 68, and respirations 20. Two hours later the patient is not awake, has a blood pressure of 170/70, pulse 52, and the left pupil is now slower than the right pupil in reacting to light. These findings suggest which of the following? 1. impending brain death 2. decreasing intracranial pressure 3. stabilization of the patient's condition 4. increased intracranial pressure

Correct Answer: 4 Rationale: Rising systolic blood pressure, falling pulse, and a pupil that has become sluggish suggest increasing intracranial pressure (IICP). This is an emergency situation that requires notification of the physician. This is an emergency situation that requires intervention as the patient's condition is becoming more unstable. Brain death is diagnosed by lack of brain waves and inability to maintain vital function.

If the negative pressure system is already in use, what should you do when changing dressings?

Count the sponges as you remove

Which of the following is NOT considered a border for the oral cavity? A. lips B. cheeks C. tongue D. tonsils

D. tonsils

What is the medical treatment for TRIGEMINAL NEURALGIA? What is the surgical treatment?

DILANTIN or TEGRETOL. For surgical, local nerve blocks or slow nerve transmission to decrease pain.

How do you describe a GLAUCOMA vision?

Described as a halo or tunnel vision. It never returns to normal

Akenesia

Difficulty initiating movement

Dyskenesia

Difficulty moving

Parkinson's Disease

Dopamine deficiency occurs in basil ganglia. the dopamine releasing pathway that connects the substantia nigra to the corpus stratum

If a patient with CVA is getting dressed which side should they address first?

Dress the weak side first

What makes someone with myasthenia gravis worse?

Exercise and Infection

What are the risk factors for GLAUCOMA?

Familial over age 40, diabetes, and history of ocular problems.

Autonomic Nervous System

Fight or flight response

If someone with GBS has voiding problems, what should you be concerned about?

Get a vent in place because diaphragm is going to be affected next.

Cranial Nerve IX

Glossopharyngeal -motor: depress tongue, say "Ahh"; uvula moves, gag reflex, voice sounds smooth

Discuss conditions that result in increased ICP.

Head Injury Hematoma CVA Tumors Infections

Spinal cord

Houses the reflex arc for actions such as the knee-jerk reflex

What should you teach your patient about TENS therapy?

How to apply the electrodes properly. Do not apply over eyes, carotid sinus nerves, or broken/irritated skin and how to operate, avoid using while sleeping, not without Doctor's approval.

What should you use to clean an artificial eye?

Hydrogen peroxide and normal saline.

Cranial Nerve XII

Hypoglossal -motor: assess tongue control

Discuss the nursing measures in a bowel/bladder training program.

I&O, q2hr offer bedpan or urinal, maintain skin integrity in the perineal area, promote daily intake of 2L, but limit intake at night, high-fiber diet, offer the bedpan/urinal at the same times each day, stool softeners as ordered, increase physical mobility as tolerated (increases peristalsis)

When should negative pressure wound care be used?

If a wound is difficult to manage or traditional methods are not working for healing.

Describe symptoms that may indicate a change in LOC.

Irritability Restlessness Personality changes Short-term memory changes Disorientation to place, time, and person

Sensoristasis

Is the term used to describe the state in which a person is in optimal arousal.

Describe strategies and approaches to prevent a CVA.

Ischemic Stroke Modifiers: • Hypertension (Because HBP damages arteries throughout the body, it is critical to keep your blood pressure within acceptable ranges to protect your brain from this often disabling or fatal event.) • Atrial fibrillation • Hyperlipidemia • Diabetes mellitus (associated with accelerated atherogenesis) • Smoking • Asymptomatic carotid stenosis • Obesity • Excessive alcohol consumption Hemorrhagic Stroke Modifiers: Primary prevention of hemorrhagic stroke is the best ap- proach and includes managing hypertension and ameliorat- ing other significant risk factors. Control of hypertension, especially in people older than 55 years of age, reduces the risk of hemorrhagic stroke. Additional risk factors are increased age, male gender, and excessive alcohol intake. Stroke risk screenings provide an ideal opportunity to lower hemorrhagic stroke risk by identifying high- risk individuals or groups and educating patients and the community about recognition and prevention.

What is PARKINSON'S DISEASE?

It is a progressive neurological disease with a slow onset that usually occurs after age 50, rarely occurring in the black population, and leading to a respiratory death. The motor symptoms of Parkinson's disease result from the death of dopamine-generating cells in the substantia nigra, a region of the midbrain; the cause of this cell death is unknown.

What is multiple sclerosis?

It is a progressive neurological disease with an onset among those who are at their 20's through 40's. It has a hereditary link and occurs most commonly among women. It involves the hardening of multiple nerves, and is aggravated by stress. It limits changes.

What is myasthenia gravis?

It is an autoimmune neuromuscular disease that affects the motor cranial nerves. It is the GRAVE MUSCLE WEAKNESS. Exacerbation and remissions are parts of the disease which tend to be progressive over time.

What is important to know about STAPEDECTOMY?

It is when bones from the inner ear are removed and replaced with artificial ones. Swelling and noise reduction is normal for 1 year. Sleep on non-surgical side and sneeze with mouth open

What should someone with a CVA do to prevent contractures?

Keep joints fully extended without bends.

What drug is commonly taken to help Parkinson's symptoms? What are the Considerations?

LEVODOPA. Watch for hypotension and place TED hose on your patient to prevent further complications.

Homonymous Hemianopia

Loss of vision in half of the visual field on the same side of both eyes

Lennox-Gastaut syndrome

Manifests between ages 2-8 yrs, triad of: 1. Mental retardation 2. Diffuse slow spike and wave pattern on EEG 3. Multiple types of generalized seizures -Pts. commonly have status epilepticus

Levodopa

Medication that crosses blood brain barrier. Goes into the brain and converts to dopamine.

Brain Stem includes

Midbrain, pons, and medulla

Medullablastoma

Most common malignant brain tumor in children. Occur in the cerebellum.

Is someone with Meniere's disease a priority disease?

NO, they can be discharged because they won't die from the condition.

What should you teach a patient about managing GLAUCOMA?

No straining for bowel movement, no vomiting or sneezing

What is characteristic of a Stage I pressure ulcer?

Nonblanchable erythema, redness, that remains red 30 minutes after pressure has been relieved. Epidermis remains intact.

Global Aphasia

Nonfluent aphasia w/ impaired comprehension. Both Broca's and Wernicke's areas affected.

Stage 1

Normal function; no memory problems

Discuss the nursing care for patients with Alzheimer's disease.

Nursing care focuses on assisting the client and caregiver to maintain the highest quality of life. Diags: Disturbed Thought Process, Self-Care Deficits, Caregiver Role Strain

Cranial Nerve I

Olfactory - sensory

What does hemiplegia mean? What are the considerations?

One half of the body is paralyzed. Cranial nerves are involved so at risk for aspiration.

Cranial Nerve II

Optic Nerve - sensory

What drug is used by someone experiencing autonomic dysreflexia symptoms at home?

PROCARDIA to decrease BP. Prick and squeeze under tongue. Works quickly and raise head of bed.

What does quadriplegia mean? What are the considerations?

Paralysis from shoulders down. Broken cervical area, C2-4. Need vent because diaphragm innervation is lost. Bowel care by RN is necessary.

What does paraplegia mean? What are the considerations?

Paralysis from waist down. Patient can insert own suppository and roll hands over bladder to facilitate.

Epilepsia partialis continua

Persistent focal motor seizure activity (i.e. focal motor status epilepticus) -Distal hand and foot muscles are most often affected -Active or passive movement of limb may exacerbate activity

How should you clean a hearing aid?

Pipe cleaner, toothpick to remove wax

Who may have some risk posed if using TENS?

Pregnant, demand type cardiac pacemaker, bladder stimulator, metal plates or pins, diabetes, or heart disease.

Describe the use of clinical pathway/care map to guide the care of the patient with a CVA.

Purpose: To reduce unnecessary utilization of hospital resources, to give the most efficient care possible (because time is of the essence). Clinical pathways are multidisciplinary plans (or blueprint for a plan of care) of best clinical practice for specified groups of patients with a particular diagnosis that aid in the coordination and delivery of high quality care.

What is the hallmark symptom of subarachnoid hemorrhage?

SEVERE HEADACHE THAT COMES SUDDENLY

Pregabalin (Lyrica)

Same risks and benefits as Gabapentoin though bit more S/E

Define seizure vs. epilepsy

Seizure = altered behavior or sensorium due to excessive or hypersynchronous discharge of neurons Epilepsy = predisposition to generate seizures + occurrence of at least 1 seizure

List three symptoms which may be present in CP.

Seizures, poor sucking, difficulty feeding.

Dorsal Spinal Nerves Function

Sensory

Parietal Lobe

Sensory lobe: person's awareness of body position in space, size and shape discrimination, right-left orientation.

What are the non-reversible risk factors for CVA?

Sex, age, race and heredity

How is the diagnosis of myasthenia gravis made versus cholinergic crisis?

Since both are present with the same symptoms, administer TENSILON to differentiate one from the other. If the symptoms improve, then it is myasthenia gravis. A lack of medication, PROSTIGMIN, would then be the problem. So, increase medication. If the symptoms get worse, then it is a cholinergic crisis. Too much medication is the problem with increased Prostigmin. So provide an antidote which is ATROPIN.

What is an appropriate intervention to provide safety in someone with decorticate posturing?

Since flexed, roll washcloth and place between hands to prevent fingernails from embedding into palms because tension is so tight.

Bradykenesia

Slowness of movement

What are CVA risk factors that are reversible?

Smoking, obesity, increased salt intake, sedentary lifestyle, increased stress, and birth control pills.

Right-sided CVA

Spatial-perceptual deficits, increased distractibility, impulsive behavior and poor judgement, lack of awareness of deficits

What kinds of food or vitamins should someone with Parkinson's avoid? Why?

Spinach, bananas, fish and pepper. Also, Vitamin B6, because they all deactivate LEVODOPA, the precursor to the neurotransmitters dopamine, norepinephrine (noradrenaline), and epinephrine (adrenaline) collectively known as catecholamines

Cranial Nerve XI

Spinal Accessory -motor: patient rotate head and shrug shoulder against resistance

What type of precautions should be used for applying a negative pressure system?

Standard precautions

Vagus Nerve Stimulation - what it is - effectiveness

Stimulator placed in the body below clavicle and lead connected to vagus nerve --> provides intermittent stimulation --> battery lasts 6 to 10 years Effectiveness / Outcome -30-50% of patients with reduction in seizure frequency -4-10% seizure free -About equal to AED efficacy, but no systemic side-effects

Atonic seizure

Sudden decrease in muscle tone -> loss of postural control -> patient may fall ("drop attack")

Absence seizures

Sudden interruption of activity associated with unresponsiveness and blank stare, sometimes with eyelid fluttering -Often precipitated by hyperventilation -Tx: ethosuximide (initial), valproic acid (long-term)

Infantile spasms

Sudden, rapid flexion of neck and truck, adduction of shoulders and outstretched arms, variable flexion of lower extremities -Tx: vigabatrin

What is the only treatment for cataracts?

Surgical removal

Patient Teaching consists of: (7)

Taking medications at prescribed times Good skin care to avoid skin ulcers Staying physically active (dance, yoga, pilates, biking) Proper ambulation (assistive devices) Proper positioning (avoid ulcers) High-Fiber diet & adequate fluid intake (Prevent constipation) Proper feeding techniques (avoid aspiration)

Reinforce teaching to patient/family healthy lifestyle choices.

Talk about changing modifiers to prevent future TIAs or CVAs: controlling hypertension (stress, meds), reinforcing the benefits of anticoagulant therapy and other info about medications, low-cholesterol and low-fat diet to reduce arteriosclerosis, etc. when to seek medical care; complications such as aspiration, pneumonia, UTI, skin breakdown; safety measures to prevent falls; psychologic support.

Identify support systems available to patient and family.

Teach about psychological support, respite care, community resources such as home health agency, meals on wheels, elder care, sources for special adaptive equipment, support groups and stroke clubs.

Discuss a teaching plan for the child with a head injury.

Teach patient/parent about: Dizziness, nausea vomiting, when to call HCP Visual disturbances; blurring, pupils Headaches LOC - Keep patient oriented, check pt at least every hour Avoiding contact sports

Examine health promotion techniques and available resources for the patient with a head injury.

Teaching prevention to avoid head injury is key.

Thoughts on pathogenesis in generalized epilepsy

Thalamocortical circuits going "haywire" eg. seizure due to cocaine use (hits throughout the brain)

Which two procedures are used to control tremors when medications are not working?

Thalmotomy (Thalamus) Pallidotomy (Globus Thalamus)

D

The clinical diagnosis of dementia is based on a. CT or MRS b. brain biopsy c. electroencephalogram d. patient history and cognitive assessment

B

The early stage of AD is characterized by a. no noticeable change in behavior b. memory problems and mild confusion c. increased time spent sleeping on or in bed. d. Incontinence, agitation, and wandering behavior

What causes tremors?

The globus thalamus and the hypothalamus become over active and diminish dopamine and motor movement that cause tremors

Neglect Syndrome

The result of certain right parietal lobe lesions that leave a patient completely inattentive to stimuli to her left, including the left side of her own body.

Discuss the etiology of seizures.

This is a disorder that involves a sudden episode of abnormal, uncontrolled dis- charge of the electrical activity of the neurons within the brain. The patient may experience a variety of symptoms depending on the type of seizure and the cause.

Cranial Nerve V

Trigeminal -mixed - motor: clenching teeth -sensory: assess ability to taste

Cranial Nerve IV

Trochlear - motor: eye movement

What are occupational therapist used for in PD patients

Used for alternative ways to handle ADLs (dress in slip on clothes and shoes, no buttons). Also, how to use utensils to eat.

When do symptoms occur?

When the affected brain cells can no longer perform their normal inhibitory function within the CNS

What is the purpose of a TENS system?

When used as an adjunctive therapy it can reduce the amount of pain meds a patient needs for relief. It is battery powered and allows the patient to administer therapy as needed.

B

Which of the following statements accurately describes mild cognitive impairment. (select all that apply) a. always progresses to AD b. Caused by variety of factors and may progress to AD c. Should be aggressively treated with acetylcholinesterase drugs d. Caused by vascular infarcts that if treated will delay progression to AD c. Patient is usually not aware that there is a problem with his or her memory.

3

Which one of the followin is an example of the emotional component of wellness? 1. the client chooses healthy foods 2. a new father decides to take parenting classes 3. A client expressess frustration with her partner's substance abuse 4. A widow with no family decides to join a bowling league

D

Which patient is most at risk for developing delirium? a. a 50 year old woman with cholecystitis b. a 19 year old man with a fractured femur c. A 42 year old woman having an elective hysterectomy d. A 78 year old man admitted to the medical unit with complications related to heart failure

2

While hospitalized a client is very worried aboiut business activities. The client spends a great deal opf time on the phone and with collegues instead of resting. Which principle of need therapy applies to this client? 1. his higher level need cannot be met unless the lower level physicological need is met 2. His lower level physiological needs are being deferred while higher need are addressed. 3. The higher need takes precedence and the lower need no longer must be met. 4. It is necessary for someone else to meet his higher level needs so he can focus on the lower level needs.

Explain the difference between a TIA and a CVA.

While transient ischemic attack (TIA) is often labeled "mini-stroke," it is more accurately characterized as a "warning stroke," a warning you should take very seriously. TIA is caused by a clot; the only difference between a stroke and TIA is that with TIA the blockage is transient (temporary). TIA symptoms occur rapidly and last a relatively short time. Most TIAs last less than five minutes; the average is about a minute. Unlike a stroke, when a TIA is over, there's no permanent injury to the brain.

What should you discuss with your patient regarding negative pressure wound care?

Why use it, how it works, and how frequently dressings need to be changed. Explain that dressing changes will be uncomfortable but that the discomfort should end as soon as negative pressure is applied.

How is multiple sclerosis treated?

With steroids when active. Patient then becomes immunocompromised.

Is someone with aphasia still considered competent?

YES

Should everyone young and old wear sunglasses when in the sun?

YES

Is it serious?

Yes, medical Emergency!! 20% mortality.

Describe techniques to promote independence for the patient with impaired cognition.

a daily routine that they can count on, everything familiar to them has one place inside of their room, encourage as much self-care as possible, demonstrate use of equipment, modify clothing with Velcro and lay out daily clothing, encourage "finger foods" during meals.

Define neuromuscular disorder.

a disorder involving the relationship between nerves and muscles, and especially the weakening or dysfunction of muscles.

MRI

a technique that uses magnetic fields and radio waves to produce computer-generated images that distinguish among different types of soft tissue; allows us to see structures within the brain

PET Scan

a visual display of brain activity that detects where a radioactive form of glucose goes while the brain performs a given task

Managing status epilepticus

a. ABC (airway/breathing/circulation) b. IV lorazepam, 0.1 mg/kg (a benzodiazepine) c. IV fosphenytoin (pre-cursor of phenytoin) (if available; otherwise, IV phenytoin) d. If still seizing: more fosphenytoin e. If still seizing: add another agent - may need to start thinking about inducing coma

Left-sided CVA

altered intellectual ability, slow cautious behavior, Aphasia - difficulty speaking, understanding speech, numbers, reading.

What are the risk factors for PD (7)

caucasians more common in men average age onset 60 Age 40-70 Genetics Exposure to toxins

Sensory deficit

defects in the function of one or more of the senses, resulting in visual, auditory, or olfactory impairments.

Emotional Lability

excessive emotional reposnisveness characterized by unstable and rapidly changing emotions

Classification of Epilepsies (name them)

idiopathic generalized epilepsy idiopathic localized epilepsy symptomatic generalized epilepsy (does it exist?) symptomatic localized epilepsy

Apraxia

impaired ability to carry out motor activities despite intact motor function

Sensory perception

involves the conscious organization and translation of the data or stimuli into meaningful information

Aricept (donepezil)

is a cholinesterase inhibitor block cholinesterase the enzyme responsible for breakdown of ach in the synaptic cleft.

Cerebral Arteriography

is a form of medical imaging that visualizes the arterial and venous supply of the brain. It was pioneered by Dr Egas Moniz in 1927, and is now the gold standard for detecting vascular problems of the brain.

Discuss the nursing implications for medications ordered for a patients with a head injury.

osmotic diuretics (Mannitol) expel large amount of h2o and electrolytes - may have to switch to loop diuretic. Corticosteroids reduce inflammation. Zantac, Protonix, or antacids are given to prevent GI irritation. Antemetics are used to prevent vomiting. Anticonvulsants (Dilantin, Valium, phenobarbital). Barbituates are given to induce coma, last resort, reduces metabolism and slows brain death. Nursing Implications:

decerebrate posturing

posturing in which the neck is extended with jaw clenched; arms are pronated, extended, and close to the sides; legs are extended straight out; more ominous sign of brain stem damage. Most Severe.

Disadvantages

• Complex kinetics - as increase dose, blood level rises gradually until takes off (see graph) • Many drug interactions

Advantages of Lamotragine

• broad spectrum • seems to be very potent • less sedating than other AEDs

Quirks / Adverse Effects

• gingival hyperplasia (overgrowth) • hirsutism • rash (occasionally serious) • osteoporosis (vit D metabolism issues) • cerebellar damage with long use

CPP normals

> 70-80mmHg

What is GLAUCOMA?

A condition characterized by increased intraocular pressure and progressive vision loss.

Which of the following nursing actions should be implemented in the care of a patient who is experiencing increased intracranial pressure (ICP)? A) Monitor fluid and electrolyte status astutely. B) Position the patient in a high Fowler's position. C) Administer vasoconstrictors to maintain cerebral perfusion. D) Maintain physical restraints to prevent episodes of agitation.

A) Monitor fluid and electrolyte status astutely Fluid and electrolyte disturbances can have an adverse effect on ICP and must be monitored vigilantly. The head of the patient's bed should be kept at 30 degrees in most circumstances, and physical restraints are not applied unless absolutely necessary. Vasoconstrictors are not typically administered in the treatment of ICP.

The roof of the mouth is divided into 2 parts known as the: A. hard and soft palates B. anterior and posterior palates C. frontal and ethmoid arches D. superior lingual and uvular arches

A. hard and soft palates

What drug should never be given to someone with Glaucoma?

ATROPINE

Discuss the teaching plan for the patient with an extra-pyramidal disorder.

Actions of medication Continued support and counseling

Advantages and Disadvantages

Adv: sustained release forms available Disadv: • drug interactions (note: erythromycin reduces carbamazepine clearance - potential toxicity)

What is ELASE?

An enzyme that dissolves eschar. Treatment is effective since good blood supplies an area of healing

Juvenile Absence Epilepsy

- 12 to 18 year olds - starts w/ myoclonic jerks - rx = lifelong valproic acid (lots of potential side effects)

Childhood Absence Epilepsy - age range - clinical manifestation (and EEG) - rx and prognosis

- 4 to 8 y/o - staring off for a few sec then back to "normal"; can elicit this via hyperventilation during P/E - AEDs - best is ethosusimide then valproic acid - 2/3 recover from this and stop rx

Benign Rolandic Epilepsy

- 5 to 10 y/o - hemifacial twitching and drooling esp at night - rx = AED (if needed) and resolves by teen years

When dealing with safety in a PD patient, what should one tell them?

Clear away all rugs from the home, make environment clutter free, and use electric shaver.

The oropharynx is separated from the mouth by which of the following? A. Frenulum B. posterior pharyngeal wall C. tonsillar pillars D. dorsum of the tongue

C. tonsillar pillars

On patients with spinal cord injuries, what areas must be placed on vents?

C1-C4 because the diaphragm is paralyzed.

Cranial Nerve VII

Facial -motor: symmetry and mobility of face -sensory: ability to taste

Levetiracitam

Few S/E and no DDIs so good drug esp not sure the type of seizure; drug of choice although can cause behavioral problems (affect, etc)

Focal (partial) seizure

First clinical and EEG changes indicate initial activation of a system of neurons limited to one part of one cerebral hemisphere -Tx: carbamazepine, phenytoin, oxcarbazepine, topiramate, valproic acid

Generalized tonic-clonic seizure

First clinical and EEG changes indicate initial involvement of both hemispheres -Sustained muscular contraction and rhythmic jerking of muscles -Upward eye deviation, pupillary dilatation, "epileptic cry", may have urinary incontinence -Patient is confused or sleepy afterwards with aching and stiffness -Tx: phenytoin, valproic acid, lamotrigine, levetiracetam, topiramate

Gabapentoin

First in the newer generation of drugs; Not as great as the others (efficacy) but no DDIs and less S/E; often use in pt w/ transplants; facing S/E

Complex partial seizure

Focal seizure in which consciousness IS impaired -Common feature: motionless stare during which patient does not respond to external stimuli and expresses automatisms -Patient does not remember seizure

Simple partial seizure

Focal seizure in which consciousness is NOT impaired

Discuss nursing care for the child with ICP.

Frequent assesment of vital signs Careful assement of neurological status Maintaining patent airway Maintaing fluid and electrolyte balance Assesing for s/s of bleeding Parental education and support Elevate HOB 30 degrees, keep head still Use logrolling Nutrition ROM - Mobility Avoid vaso Vagus stimulation Monitor lab values Assess for s/s of infection

Explain the importance of frequent neuro vital signs in the early phase of neurological injury.

Frequent vitals allows the LPN to report and changes in the vitals immediately to HCP. It also allows nurse to identify the types of interventions the patient may need.

Discuss nursing responsibilities of neurological vital signs.

Full Vital Signs Assess LOC, and orientation, if pt is unconcious use the Glascow Coma Scale Assess strength of hands grip and movement of extremities Assess pupils using PEERLA

What is characteristic of a Stage III pressure ulcer?

Full thickness skin loss down through the dermis which may include subcutaneous tissue, eschar = dead tissue

What is characteristic of a Stage IV pressure ulcer?

Full thickness skin loss extending into supportive structures, such as muscle, tendon, and bone.

Identify terms describing level of consciousness (LOC).

Full: alert, oriented to time place, and person, pt fully understands written and spoken words. Confusion: unable to think rapidly and clearly; easily bewildered (confused) with short attention span and poor memory. Disorientation: disorientened to time, place, and person. Obtundation: appears drowsy and lethargic; responds to verbal and tactile stimuli but quickly drifts back to sleep. Stupor: generally unresponsive; may withdraw purposefully with vigorous or painful stimuli. Coma: unarousable, does not stir or moan in response to stimuli

What is the correct way to help someone with a CVA get out of bed?

Functioning vs. affected side CVA. Place wheel chair on the patient's strong side, lock the wheels, elevate the head of bed, and stand on the patient's strong side to assist

What is characteristic of open angle glaucoma?

Gradual loss of peripheral vision - tunnel. Generally, PAINLESS, and blindness can result if not treated

What are the risk factors for a subarachnoid hemorrhage?

HTN, smoking, alcohol, or cocaine use and pregnancy

Explain the emergency care for a patient experiencing a CVA.

Helping to determine what kind of stroke it is, and acting appropriately. If ischemic, determine if pt is candidate for thrombolytic therapy. If hemorrhagic stroke, measures to reduce bleeding and IICP should be taken. Ischemic: (If non-thrombolytic therapy is needed) Interventions during this period include measures to reduce ICP, such as administering an osmotic diuretic (eg, mannitol), maintaining the partial pres- sure of carbon dioxide (PaCO2) within the range of 30 to 35 mm Hg, and positioning to avoid hypoxia. Other treatment measures include the following: • Elevation of the head of the bed to promote venous drainage and to lower increased ICP • Possible hemicraniectomy for increased ICP from brain edema in a very large stroke • Intubation with an endotracheal tube to establish a patent airway, if necessary • Continuous hemodynamic monitoring (the goals for blood pressure remain controversial for a patient who has not received thrombolytic therapy; antihyperten- sive treatment may be withheld unless the systolic blood pressure exceeds 220 mm Hg or the diastolic blood pressure exceeds 120 mm Hg) • Neurologic assessment to determine if the stroke is evolving and if other acute complications are devel- oping; such complications may include seizures, bleeding from anticoagulation, or medication- induced bradycardia, which can result in hypotension and subsequent decreases in cardiac output and cere- bral perfusion pressure During the acute phase, a neurologic flow sheet is main- tained to provide data about the following important mea- sures of the patient's clinical status: • Change in level of consciousness or responsiveness as evidenced by movement, resistance to changes of po- sition, and response to stimulation; orientation to time, place, and person • Presence or absence of voluntary or involuntary movements of the extremities; muscle tone; body pos- ture; and position of the head • Stiffness or flaccidity of the neck • Eye opening, comparative size of pupils and pupillary reactions to light, and ocular position • Color of the face and extremities; temperature and moisture of the skin • Quality and rates of pulse and respiration; arterial blood gas values as indicated, body temperature, and arterial pressure • Ability to speak • Volume of fluids ingested or administered; volume of urine excreted each 24 hours • Presence of bleeding • Maintenance of blood pressure within the desired pa- rameters

What is MENIERE's Disease?

It is called 'my ears disease'. An idiopathic chronic condition associated with water in the ear.

Cerebellum

Major motor and sensory pathway. Controls smooth, coordinated muscle mvmts and helps maintain equilibrium

Brain Stem

Major sensory and motor pathway for impulses running to and from the cerebrum. Regulates body functions (resp, auditory, visual, gag, swallowing, coughing)

Discuss prenatal, perinatal, postnatal, causes of CP.

May be caused prenatally by the mother contracting rubella or other infection, malnutrition, abnormal attachment of the placenta, toxemia, radiation, or medication. Perinatally, it may be caused by a difficult birth, prolapsed umbilical cord, or multiple births. Postnatally, an infant might develop it as a result of trauma and result in prolonged anorexia or decreased circulation to the brain.

Explain the differences between Medicare and Medicaid reimbursement.

Medicaid: U.S government sponsored program for low-income individuals and families to pay the cost of health care. Medicaid beneficiaries are low income families and individuals. Covers a wider range than Medicare: hospitalization, x-rays, laboratory services, midwife services, clinic treatment, pediatrics care, family planning, nursing services and in-home nursing facilities for 21+ years, medical and surgical dental care. In some states Medicaid beneficiaries are required to pay the provider a small fee (co-payment) of up to $30 per month for medical services. May require payment of deductibles and co-pay for certain services provided. Program is run by individual states so the type of coverage and policies may vary between states. But generally, patients usually pay no (or very little) part of costs for covered medical expenses. Medicare: U.S government sponsored health care program for people above 65 years of age, people under 65 with certain disabilities and all people with end stage renal disease. Medicare beneficiaries are senior citizens over the age of 65, end stage renal disease, and disabled eligible to receive social security benefits. Divided in to Part A which covers hospital care, Part B which covers medical insurance and Part D covers prescription drugs. May require payment of deductibles and co-pay for certain services provided., Medicare reserves the right to refuse to pay for treatments it deems unnecessary. Small monthly premiums are required for non-hospital coverage. Federally run so the program and coverage is uniform throughout the country. Run by the Health Care Financing Administration.

Myoclonic seizures

Nonrhythmic, rapid, jerking movements that can be local or widespread -Some patients may exhibit diffuse, severe nonepileptic myoclonus after anoxic brain injury -Tx: valproic acid (DOC)

Disoriented

Not oriented to time, place, or person.

Describe assessment findings: nuchal rigidity, photophobia, opisthotonus, Kerning's sign, and Brudzinksi's sign.

Nuchal rigidity: neck stiffness Photophobia: intolerance of bright light Opisthotnus: A type of spasm in which the head and heels arch backward in extreme hyperextension and the body forms a reverse bow Kerning's sign: is positive when the leg is bent at the hip and knee at 90 degree angles, and subsequent extension in the knee is painful Brudzinski's sign: is the appearance of involuntary lifting of the legs when lifting a patient's head.

Describe the role of the nurse in the community setting.

Nurses work in diverse community settings to provide primary nursing and health care across the lifespan. Traditionally community nurses meet a continuum-of-health needs that range from the management of specific disease/s to broader community development and public health promotion needs. Health promotion and intervention consciously centre on the client who is viewed holistically; thus, care also considers the social conditions and relationships that affect an individual or a population's health status. In recent years the community nurse's role has begun to shift, directing more attention to the provision of disease recovery nursing care for transitioning clients as they move out of the hospital environment and into the community context. Additionally, the community nurse's role has become more focused on the provision of early intervention measures to prevent exacerbations or complications for clients living with chronic illness/conditions to prevent unnecessary hospital (re)admission.

What are physical therapist used for in PD patients

Teach safety! Patient learns to walk again when shuffling (draw imaginary life and have them follow it. Life entire foot and do "u-turn"). Exercise very important, helps them gain control again (swimming, biking, yoga, dance).

Discuss Glasgow Coma scale.

The Glascow Coma Scale provides a quick guide for assesing LOC, It measures how well the pt responds to eye opening and verbal and motor responses.

Recognize the patient/family's ability to adapt to role changes.

The middle-aged adult family member may become the care-taker for an older parent. An older adult may be unable to care for a spouse who has had a stroke. They may have to accept placement of the spouse into an LTC. Emphasize that physical function may continue to improve for up to 3 months, and speech may continue to improve even longer.

Cranial Nerve III

Oculomotor - motor: Assess pupil size and light reflex

How often are hearing aid batteries changed?

On a fixed schedule every 6 months

When should you not use a negative pressure system?

On a wound with an infection, necrosis, malignancy, fistulas, osteomyelitis, or if can see arteries/veins in wound

Astrocytoma

The most common type of Glioma. Earliest S&S: headache. Also, seizures, memory loss, weakness, visual symptoms, personality changes.

Decerebrate posturing

-"extensor posturing"; -abduction of arms, elbow and wrist extension

Decorticate posturing

-"flexor posturing" or "mummy baby" (think Egyptian mummy preservation) -adduction of arms (arms fold to chest); flexion of elbows and wrists

Lumbar puncture

-CSF sample withdrawn from spinal canal

Lumbar puncture

-CSF sample withdrawn from spinal canal -

3 kinds of ICP monitoring systems

-Intraventricular catheter (ventriculostomy) -Subacrachnoid screw or bolt -Epidural or subdural sensor

What nursing interventions should be implemented for someone with aphasia?

1) Stand in front of client 2) Speak clearly, slowly. 3) Do not shout or speak loudly. They can hear. 4) Be patient and give client time to respond 5) Use nonverbal communication, e.g. touche, smile 6) Assist client with motor aphasia to practice simple words, 7) Listen carefully 8) Provide simple directions 9) Involve family in practice 10) Show picture cards to help convey a message

What are symptoms of stroke?

1) Sudden weakness or numbness of the face, arm and leg, usually on one side of the body. 2) Difficulty talking or understanding speech 3) Dimness or loss of vision on one eye 4) Unexplained dizziness and unsteadiness 5) Falls 6) Sudden severe headache

Process of Diagnosing Seizures and Recommending Mgmt

1) first seizure, unprovoked: - ID cause (labs eg. CBC, electrolytes, UA, toxicity screen, BUN, Glu). Cause found, then treat it - No cause ID --> no AED (b/c 50% will go on to have seizures, but 50% will not) - F/u tests (MRI, etc - missed anything/) - NO DRIVING 2) second seizure - AED (b/c most likely to continue having seizures) - which AED?

In planning for the discharge of a client with a cognitive disorder, it is important to assess the client's caregiver support system. Which aspects are the most crucial to assess? Select all that apply. 1. Availability of resources for caregiver support. 2. Ability to provide the level of care and supervision needed by the client. 3. Willingness to transport the client to medical and psychiatric services. 4. Interest in engaging the cognitively disordered family member in reminiscence and games. 5. Willingness to install door alarms and make other safety changes. 6. Understanding the client's abilities and limitations.

1, 2, 3, 5, 6. It is important for a caregiver to have support for herself as well as be able to provide adequate safety, supervision, and medical care to the client. The caregiver must also have realistic expectations of the client, given his abilities and limitations. Reminiscing and engaging the client in games is desirable but not crucial to care.

Discuss common drugs used in the treatment of Alzheimer's disease.

Cholinesterase inhibitor drugs, such as Cognex (40-80mh 4x/day, admin 1hr before or 2hr after meal), Aricept (5-10mg/day bedtime), and Exelon (1.5-6mg b.i.d), Reminyl (4-12mg b.i.d), and Namada (5-10mg b.i.d) block the breakdown of acetylcholine. Slows cognitive decline. Monitor ALT levels with cognex, elevated levels may indicate hepatoxicty Adverse Reactions: N/V/D, HA, confusion, upset stomach. SSRIs such as Prozac treat depression. Risperdal or Seroquel is used to control behavioral symptoms.

The nurse is assessing for sensory function. Match the assessment tool to the specific sense it will be testing. 1. Identifying taste a. Hearing 2. Stereognosis b. Gustatory 3. Snellen chart c. Visual 4. Identifying aromas d. Olfactory 5. Tuning fork e. Tactile

1. Identifying taste c. Visual 2. Stereognosis e. Tactile 3. Snellen chart b. Gustatory 4. Identifying aromas d. Olfactory 5. Tuning fork a. Hearing

Classifications of Epilepsies

1. Idiopathic vs. symptomatic (of known or suspected CNS pathology) 2. Generalized vs. localization-related (focal) - re. most of the seizures the person experiences • Result: 4 categories

Managing Epilepsy w/ AEDs (algorithm)

1. Increase dose until no seizures and DON'T stop just because serum drug level is high - "normal" ranges are population averages, not nec individuals 2. If seizures not controlled on maximum tolerated dose, add a 2nd AED 2a. If seizure control achieved: start to taper the first AED 2b. If patient still having seizures: taper one of the two AEDs and start a third 3. If seizures not controlled after trying several AEDs at adequate doses, then a. re-assess diagnosis b. consider surgical management

Classifications of focal seizures

1. Local - stay in that same focus 2. Regional - spreads to other areas of that region (eg from legs to arms in that side of the body) 3. Bilateral (more diffuse spreading) - spreads to both sides of body

What are the signs and symptoms of Parkinson's Disease?

1. Mask like blank expression 2. pill rolling 3. Shuffling gait 4. propulsive gait 5. tremors 6. bradykinesia 7. loss of ability to swallow 8. decreased blinking 9. muscle rigidity

What lid is the correct way to administer Eyedrops? What is the sequential order?

1. Offer tissue paper. Make sure patient has no contact lenses. 2. Do hand hygiene, wear gloves. 3. Clean eyes with gauze or moistened cotton balls 4. Ask patient to tilt head back 5. Turn head slightly to treated side 6. Pull lower lid down with thumb. 7. Place bottle 1/2 to 3/4 inches from above the conjunctival sac 8. Administer dose, ask to gently close eyes and move them while closed to help distribute medication. 9. Do not squeeze, shut or rub eyes. 10. Press on inner canthus for about 30 seconds to slow drainage of medication. 11. Use tissue paper to remove excess medication. 12. Give water drops before oil and leave for 5 minutes between eyedrop administration.

What should be done on a post retinal detachment surgery?

1. Position face down because gas was placed in eye 2. Apply ice compresses and provide analgesics as needed 3. No lifting of anything heavier than 20 lbs. 4. No rubbing of eyes 5. No coughing or suctioning 6. Needs 2 eye patches at night. 2 words = 2 patches

A client with Parkinson's disease needs a long time to complete her morning hygiene, but she becomes annoyed when the nurse offers assistance and refuses all help. Which action is the nurse's best initial response in this situation? 1. Tell the client firmly that she needs assistance and help her with her care. 2. Praise the client for her desire to be independent and give her extra time and encouragement. 3. Tell the client that she is being unrealistic about her abilities and must accept the fact that she needs help. 4. Suggest to the client that if she insists on self-care, she should at least modify her routine.

2

A client with multiple sclerosis (MS) lives with her daughter and 3-year-old granddaughter. The daughter asks the nurse what she can do at home to help her mother. Which of the following measures would be most beneficial? 1. Psychotherapy. 2. Regular exercise. 3. Day care for the granddaughter. 4. Weekly visits by another person with MS.

2

The nurse is preparing a client with multiple sclerosis (MS) for discharge from the hospital to home. The nurse should tell the client: 1. "You will need to accept the necessity for a quiet and inactive lifestyle." 2. "Keep active, use stress reduction strategies, and avoid fatigue." 3. "Follow good health habits to change the course of the disease." 4. "Practice using the mechanical aids that you will need when future disabilities arise."

2

During a home visit to an elderly client with mild dementia, the client's daughter reports that she has one major problem with her mother. She says, "She sleeps most of the day and is up most of the night. I can't get a decent night's sleep anymore." Which suggestions should the nurse make to the daughter? Select all that apply. 1. Ask the client's physician for a strong sleep medicine. 2. Establish a set routine for rising, hygiene, meals, short rest periods, and bedtime. 3. Engage the client in simple, brief exercises or a short walk when she gets drowsy during the day. 4. Promote relaxation before bedtime with a warm bath or relaxing music. 5. Have the daughter encourage the use of caffeinated beverages during the day to keep her mother awake.

2, 3, 4. A set routine and brief exercises help decrease daytime sleeping. Decreasing caffeine and fluids and promoting relaxation at bedtime promote nighttime sleeping. A strong sleep medicine for an elderly client is contraindicated due to changes in metabolism, increased adverse effects, and the risk of falls. Using caffeinated beverages may stimulate metabolism but can also have long-lasting adverse effects and may prevent sleep at bedtime.

When communicating with a client who has aphasia, which of the following nursing interventions is not appropriate? 1. Present one thought at a time. 2. Encourage the client not to write messages. 3. Speak with normal volume. 4. Make use of gestures.

2. The nurse should encourage the client to write messages or use alternative forms of communication to avoid frustration. Presenting one thought at a time decreases stimuli that may distract the client, as does speaking in a normal volume and tone. The nurse should ask the client to "show me" and should encourage the use of gestures to assist in getting the message across with minimal frustration and exhaustion for the client.

The nursing diagnosis Risk for Impaired Skin Integrity related to sensory-perceptual disturbance would best fit a client who: 1. Cut a foot by stepping on broken glass. 2. Uses a wheelchair due to paraplegia. 3. Wears glasses because of poor vision. 4. Is legally blind and smokes in bed.

2. Uses a wheelchair due to paraplegia. Because of the paraplegia (paralysis of lower body), the client is unable to feel discomfort. The client will be taught to lift self using chair arms every 10 minutes if possible.

The nurse observes that a client's upper arm tremors disappear as he unbuttons his shirt. Which statement best guides the nurse's analysis of this observation about the client's tremors? 1. The tremors are probably psychological and can be controlled at will. 2. The tremors sometimes disappear with purposeful and voluntary movements. 3. The tremors disappear when the client's attention is diverted by some activity. 4. There is no explanation for the observation; it is probably a chance occurrence.

2. Voluntary and purposeful movements often temporarily decrease or stop the tremors associated with Parkinson's disease. In some clients, however, tremors may increase with voluntary effort. Tremors associated with Parkinson's disease are not psychogenic but are related to an imbalance between dopamine and acetylcholine. Tremors cannot be reduced by distracting the client.

20. The client with a brain attack (stroke) has residual dysphagia. When a diet order is initiated, the nurse avoids doing which of the following? a. Giving the client thin liquids b. Thickening liquids to the consistency of oatmeal c. Placing food on the unaffected side of the mouth d. Allowing plenty of time for chewing and swallowing

20. Answer A. Before the client with dysphagia is started on a diet, the gag and swallow reflexes must have returned. The client is assisted with meals as needed and is given ample time to chew and swallow. Food is placed on the unaffected side of the mouth. Liquids are thickened to avoid aspiration.

What is a normal eye pressure? What device is used?

22 mm Hg, measured by TONOMETRY

Which intervention should the nurse suggest to help a client with multiple sclerosis avoid episodes of urinary incontinence? 1. Limit fluid intake to 1,000 mL/ day. 2. Insert an indwelling urinary catheter. 3. Establish a regular voiding schedule. 4. Administer prophylactic antibiotics, as ordered.

3

Which of the following is not a realistic outcome to establish with a client who has multiple sclerosis (MS)? The client will: 1. Develop joint mobility. 2. Develop muscle strength. 3. Develop cognition. 4. Develop mood elevation.

3

A health care provider has ordered carbidopa-levodopa (Sinemet) four times per day for a client with Parkinson's disease. The client states that he wants "to end it all now that the Parkinson's disease has progressed." What should the nurse do? Select all that apply. 1. Explain that the new prescription for Sinemet will treat his depression. 2. Encourage the client to discuss his feelings as the Sinemet is being administered. 3. Contact the health care provider before administering the Sinemet. 4. Determine if the client is on antidepressants or monoamine oxidase (MAO) inhibitors. 5. Determine if the client is at risk for suicide.

3, 4, 5. The nurse should contact the health care provider before administering Sinemet because this medication can cause further symptoms of depression. Suicide threats in clients with chronic illness should be taken seriously. The nurse should also determine if the client is on an MAO inhibitor because concurrent use with Sinemet can cause a hypertensive crisis. Sinemet is not a treatment for depression. Having the client discuss his feelings is appropriate when the prescription is finalized.

A client is experiencing agnosia as a result of vascular dementia. She is staring at dinner and utensils without trying to eat. Which intervention should the nurse attempt first? 1. Pick up the fork and feed the client slowly. 2. Say, "It's time for you to start eating your dinner." 3. Hand the fork to the client and say, "Use this fork to eat your green beans." 4. Save the client's dinner until her family comes in to feed her.

3. Agnosia is the lack of recognition of objects and their purpose. The nurse should inform the client about the fork and what to do with it. Feeding the client does not address the agnosia or give the client specific directions. It should only be attempted if identifying the fork and explaining what to do with it is ineffective. Waiting for the family to care for the client is not appropriate unless identifying the fork and explaining or feeding the client are not successful.

Sinemet

This drug is given to Parkinson's Disease patients early in the disease course. It is very effective for the management of akinetic symptoms. Tremor and rigidity may also respond to this drug. After a few years of therapy the effectiveness of sinemet wears off and other drugs are prescribed.

Pregnancy, pre-pregnancy and AEDs

Use an AED -try to use only one, -at lowest effective dose; -follow free serum drug levels (monitor levels during pregnancy) -try to keep them constant; -try to avoid valproate (most teratogenic) Risk of AED use during pregnancy << risk of seizures on fetus during pregnancy Pre-pregnancy -- consider extra folate if taking AEDs (which often interrupt folic acid pathway)

3

Using maslow's framework which statement charecterizes the highest level of need? 1. Nurse my pain is severe. . . is it time for my shot? 2. I felt welcomed when i first joined the group and i look forward to the monthly meetings 3. Im very proud of recieving the employee of the month award 4. There have been home breakins with burglary in our neighborhood. we are thinking of moving.

How should a wound be correctly measured?

Using millimeters or centimeters, measure length from 12 to 6 o'clock and width from 3 to 9 o'clock. Measure wound depth, distance from surface to deepest point. Carefully insert sterile 6" cotton tipped applicator into the deepest portion of wound and look for tunneling. Don't use subjective terms like, small, large, et. al.

Cranial Nerve X

Vagus

C

Vascular dementia is associated with a. transient ischemic attacks b. bacterial or viral infection of neuronal tissue c. cognitive changes secondary to cerebral eschemia d. abrupt changes in cognitive function that are irreversibe

Stage 2

Very mild cognitive decline; may be normal age-related changes or very early signs of AD; forgetfulness, especially of everyday objects (eyeglasses or wallet; no memory problems evident to provider, friends, or coworkers

Stage 7

Very severe cognitive decline; severe or late-stage AD; ability to respond to environment, speak, & control movement is lost; unrecognizable speech; general urinary incontinence; inability to eat w/out assistance & impaired swallowing; gradual loss of all ability to move extremities (ataxia)

Occipital Lobe

Visual interpretation and memory.

Somatic Nervous System

Voluntary control of body movements

What should you do for excessive skin moisture?

Wash and dry skin and apply protective sealant

What is speech therapy used for in patients with PD

When a patient is at risk for aspiration, swallow education, dysphagia diet

Discuss the primary characteristics of extra-pyramidal disorders.

akinesia (inability to initiate movement) and akathisia (inability to remain motionless), dystonia. relating to the part of the nervous system that affects body posture and promotes smooth and uninterrupted movement of various muscle groups.

Identify community care and resources to assist patient and family with chronic and long-term disabilities.

physical therapy will demonstrate assistive devices, social services can arrange referral to home health agency, transfer to rehab center, or job retraining program.

Agnosia

any of many types of loss of neurological function associated with interpretation of sensory information

Receptive Aphasia

aphasia characterized by fluent but meaningless speech and severe impairment of the ability to understand spoken or written words

decorticate posturing

characterized by upper extremities flexed at the elbows and held closely to the body and lower extremities that are externally rotated and extended. occurs when the brainstem is not inhibited by the motor function of the cerebral cortex.

Explain interventions to prevent patient aspiration and assist with feeding a patient with a swallowing disorder.

place client in upright positon for meals and 30 minutes afterward. tild head slightly forward. do not feed client who does not have functioning gag reflex or has altered LOC. provide oral care before meals. serve thickened liquids and pureed or soft food and place foods on unaffected side of mouth. limit distractions at meal time. have suction equipment available during mealtimes.

Recognize patient's response to SC injury (anger, grief, hopeless or suicidal) .

preventions: allow the client time to grieve or to express denial, depression, and anger over the changes in social, financial, and personal roles - the patient needs time to adjust to lifestyle changes. provide accurate information based on the physician's prognosis. include family and significant others to treat the client as normally as possible. refer the client and family to support groups.

Discuss the local and national community resources available for patients for home care.

psychological support, respite care, meals on wheels, sources for special adaptive equipment, support groups, social services

Lumbar Puncture

removal by centesis of fluid from the subarachnoid space of the lumbar region of the spinal cord for diagnostic or therapeutic purposes

Discuss the nursing care for a patient experiencing increased intracranial presure.

drugs - osmotic/loop diuretics, elevate HOB 30 degrees, midline position, o2 as ordered, avoid hip flexion and abdominal distention (stool softeners as ordered), monitor temp q2hrs for hyperthermia (no rectal temps), reduce stimulation of environment, turn client gently, limit fluid over 24hr period. barbiturates is used to induce coma, reduces (glucose) metabolism to decrease continued damage to the brain surgical interventions include burr holes (to evacuate hematoma or remove blood clot), craniotomy (relieves pressure of brain tumor), and a brain flap may be removed (to allow room for the brain to expand). post-op care is important, especially relating to IICP and respiratory function. For Head Injuries: tetanus immunization status should be checked and updated, especially when lacerations or contaminated wounds are present. Anticonvulsants may be needed to control or provide prophylaxis for seizure activity. Nonsteroidal anti-inflammatory drugs (NSAIDs) may be used for minor pain control. Beta-blockers can be prescribed for patients with trauma-induced migraines. hypotension is a indicative of morbidity

Dysarthria

the inability to use speech that is distinct and connected because of a loss of muscle control after damage to the peripheral or central nervous system

Discuss nursing care to promote independence with ADL's.

encourage the client to use the unaffected arm, teach family/client to put clothing on the affected extremity first and then dress the unaffected extremity, consult with occupational therapist to teach the client how to use assistive devices for eating, hygiene, and dressing.

Ketogenic diet - which pts - what it is - disadvantages

most children w/ difficult seizures - increases ketones and acids in the bloodstream - compliance is tough

Types of generalized seizures

• Absence (formerly called "petit mal"): staring for a few seconds and unresponsive • Myoclonic: quick jerks • Clonic: rhythmic jerking (eg. dorsiflex the ankle and it jerks) • Tonic: stiffening (sustained posture) • Tonic-clonic: stiffening → jerking (start tonic eg. pt w/ seizures in the hospitalist shadowing) • Atonic: brief loss of muscle tone

What is status epilepticus?

• Defn: Recurrent seizures that aren't stopping; If seizure persists for more than 5 minutes, likely to continue - this is status epilepticus • Most seizures resolve spontaneously in 45-90 seconds; no acute treatment necessary (except to turn patient on side if possible to reduce aspiration risk) - DON'T put tongue blade or anything else in mouth

Childhood absence epilepsy

Absence seizures that begin in children ages 4-8 y.o. and usually resolve before adulthood

Cranial Nerve VIII

Acoustic or Vestibulocochlear

(SELECT ALL THAT APPLY) A client with a history of epilepsy is admitted to the medical-surgical unit. While assisting the client from the bathroom, the nurse observes the start of a tonic-clonic seizure. Which nursing interventions are appropriate for this client?

(1) Assist the client to the floor., (2) Turn the client to his side., (3) Place a pillow under the client's head.

(SELECT ALL THAT APPLY) The nurse is teaching a client with trigeminal neuralgia how to minimize pain episodes. Which comments by the client indicate that he understands the instructions?

(2) "I'll try to chew my food on the unaffected side.", (4) "Drinking fluids at room temperature should reduce pain.", (5) "If brushing my teeth is too painful, I'll try to rinse my mouth instead."

Mesial Temporal Sclerosis

- Simple partial or complex partial szs w/ focus in TEMPORAL lobe then bilateral, often among those w/ history of febrile seizures Rx = may respond to meds, may need surgery

Ex: Pathogenesis in mesial temporal sclerosis

- mossy fibers of hippocampus (key to episodic memory) sprout collateral fibers to facilitate episodic memory so hippo is prone to re-organization of processes being disrupted - seizures themselves may induce collateral sprouting so the more you seize, the more you seize

Partial/Focal Seizure Manifestations

- motor - sensory eg visual hallucinations; burnt rubber smell - autonomic - emotional/psychic - fear, deja vue - simple partial or complex partial (simple partial = affects consciousness)

Mode of Action

- operate on synapses that control electrical activity of the pre or post-synaptic neuron (see visual - no detail at all!!) - decrease excitation via targeting of various ion channels (depends on the drug) eg. Na channel blockers; GABA enhancers; Glutamate blockers

Explain the nursing care for the patient receiving anticoagulant therapy.

-Monitor labs, monitor VS, monitor for signs of bleeding, reduce risk factors such as shaving (electric), etc. -Patients should be given the NPSA booklet (see guidance and resources) -On discharge, nurses should ensure patients know their drug dosage and arrange follow-up care -There is no evidence to suggest grapefruit juice should be avoided but cranberry juice can affect INR results. Foods rich in vitamin K can affect INR results if eaten in large quantities -Almost any drug can interact with oral anticoagulants, including herbal remedies. Most increase the effect but some reduce it. The INR should be closely monitored when a new drug is started or dose altered -Patients must know to seek medical attention for injuries, particularly head injuries, due to haemorrhage risk

Computed Tomography (CT) scan

-cross sectional images; contrast optional -know if pt has allergy to shellfish or iodine: these allergies require different contrast media -Assess renal fxn (BUN); contrast excreted renally

Discuss health promotion techniques for a patient taking anticonvulsants.

...

Discuss safety measures for a patient in status epilepticus.

...

Discuss techniques used to evaluate the credibility and usefulness of health related information.

...

Discuss the adaptions the nurse makes to provide care in the home environment.

...

Discuss the differences among home care agencies.

...

Examine safety measures for the child with seizures.

...

Explain ketogenic diet.

...

Explain measures used to keep populations healthy.

...

Explain nursing responsibilities included in the referral process.

...

Explain the teaching plan for a patient taking anticonvulsants.

...

OTHER OPTIONS

...

Phenytoin - key characteristics

...

SYMPTOMATIC LOCALIZATION-RELATED EPILEPSY

...

What are the nursing goals related to CVA care?

1) Airway and oxygenation 2) Decreased ICP 3) Nutrition 4) Preserve function 5) Safety 6) Rehabilitation 7) Education

What are the causes of CVA?

1) Atherosclerosis 2) Thrombosis 3) Embolism 4) Cerebral hemorrhage (due to trauma or tissue damage)

What are the steps for applying a negative pressure system?

1) Clean wound with Saline solution 2) Clean and ry area around wound 3) Trim sponge to fit entire wound bed but not overflow 4) Fill wound with sponge 5) Apply adherent ,transparent drape over wound to secure sponge, cut hole in drape over wound to secure sponge, cut hole in drape about 2 cm. 6) Insert end of tubing and secure

What are the three types of abnormal posturing?

1) Flexor (Decorticate - toward core) 2) Extenser (Decerebrate) and 3) Flaccid

What is infratentorial cranial surgical approach? What are the interventions involved?

1) It is an incision made at nape of neck. 2) Maintain neck in straight alignment and position patient on either side, not on back

What is transphenoidal cranial surgical approach? What are the interventions involved?

1) It is an incision made beneath upper lip to gain access into nasal cavity 2) Instruct patient to avoid blowing his nose and keep head of bed elevated to promote venous drainage from surgical site

What 3 kinds of therapy will someone with a post CVA receive?

1) Occupational therapy, activities of daily living 2) Physical therapy, ambulation 3) Speech therapy, speech and swallowing

An 83-year-old woman is admitted to the unit after being examined in the emergency department (ED) and diagnosed with delirium. After the admission interviews with the client and her grandson, the nurse explains that there will be more laboratory tests and X-rays done that day. The grandson says, "She has already been stuck several times and had a brain scan or something. Just give her some medicine and let her rest." The nurse should tell the grandson which of the following? Select all that apply. 1. "I agree she needs to rest, but there is no one specific medicine for your grandmother's condition." 2. "The doctor will look at the results of those tests in the ED and decide what other tests are needed." 3. "Delirium commonly results from underlying medical causes that we need to identify and correct." 4. "Tell me about your grandmother's behaviors and maybe I could figure out what medicine she needs." 5. "I'll ask the doctor to postpone more tests until tomorrow."

1, 2, 3. The client does need rest and it is true that there is no specific medicine for delirium, but it is crucial to identify and treat the underlying causes of delirium. Other tests will be based on the results of already completed tests. Although some medications may be prescribed to help the client with her behaviors, this is not the primary basis for medication orders. Because the underlying medical causes of delirium could be fatal, treatment must be initiated as soon as possible. It is not the nurse's role to determine medications for this client. Postponing tests until the next day is inappropriate.

A client has been in the critical care unit for 3 days following a severe myocardial infarction. Although he is medically stable, he has begun to have fluctuating episodes of consciousness, illogical thinking, and anxiety. He is picking at the air to "catch these baby angels flying around my head." While waiting for medical and psychiatric consults, the nurse must intervene with the client's needs. Which of the following needs have the highest priority? Select all that apply. 1. Decreasing as much "foreign" stimuli as possible. 2. Avoiding challenging the client's perceptions about "baby angels." 3. Orienting the client about his medical condition. 4. Gently presenting reality as needed. 5. Calling the client's family to report his onset of dementia.

1, 2, 4. The abnormal stimuli of the critical care unit can aggravate the symptoms of delirium. Arguing with hallucinations is inappropriate. When a client has illogical thinking, gently presenting reality is appropriate. Dementia is not the likely cause of the client's symptoms. The client is experiencing delirium, not dementia.

(SATA) The nurse is planning the care of a hemiplegic client to prevent joint deformities of the arm and hand. Which of the following positions are appropriate? 1. Placing a pillow in the axilla so the arm is away from the body. 2. Inserting a pillow under the slightly flexed arm so the hand is higher than the elbow. 3. Immobilizing the extremity in a sling. 4. Positioning a hand cone in the hand so the fingers are barely flexed. 5. Keeping the arm at the side using a pillow.

1, 2, 4. Placing a pillow in the axilla so the arm is away from the body keeps the arm abducted and prevents skin from touching skin to avoid skin breakdown. Placing a pillow under the slightly flexed arm so the hand is higher than the elbow prevents dependent edema. Positioning a hand cone (not a rolled washcloth) in the hand prevents hand contractures. Immobilization of the extremity may cause a painful shoulder-hand syndrome. Flexion contractures of the hand, wrist, and elbow can result from immobility of the weak or paralyzed extremity. It is better to extend the arms to prevent contractures

Transfer data for a client brought by ambulance to the hospital's psychiatric unit from a nursing home indicate that the client has become increasingly confused and disoriented. The client's behavior is found to be the result of cerebral arteriosclerosis. Which of the following behaviors of the nursing staff should positively influence the client's behavior? Select all that apply. 1. Limiting the client's choices. 2. Accepting the client as he is. 3. Allowing the client to do as he wishes. 4. Acting nonchalantly. 5. Explaining to the client what he needs to do step-by-step.

1, 2, 5. Confused clients need fewer choices, acceptance as a person, and step-by-step directions. Allowing the client to do as he wishes can lead to substandard care and the risk of harm. Acting nonchalantly conveys a lack of caring.

A client with impaired vision is admitted to the hospital. Which interventions are most appropriate to meet the client's needs? Select all that apply. 1. Identify yourself by name. 2. Decrease background noise before speaking. 3. Stay in the client's field of vision. 4. Explain the sounds in the environment. 5. Keep your voice at the same level throughout the conversation.

1, 3, & 4. Identify yourself by name, stay in the client's field of vision, and explain the sounds in the environment. Options 2 and 5 relate to interventions for a client with a hearing impairment.

(SATA) Which of the following should the nurse include in the discharge plan for a client with multiple sclerosis who has an impaired peripheral sensation? Select all that apply. 1. Carefully test the temperature of bath water. 2. Avoid kitchen activities because of the risk of injury. 3. Avoid hot water bottles and heating pads. 4. Inspect the skin daily for injury or pressure points. 5. Wear warm clothing when outside in cold temperatures.

1,3,4,5

What are the five stages of involvement in PD patients?

1- unilateral shaking or tremor of one limb 2- Bilateral limb involvement occurs making walking and balance difficult 3-Physical movements slow down significantly, affecting walking more 4-Tremors may decrease but akinesia and rigidity make daily to day tasks difficult 5-client unable to stand or walk, is dependent for all care and

The nurse is making a home visit with a client diagnosed with Alzheimer's disease. The client recently started on lorazepam (Ativan) due to increased anxiety. The nurse is cautioning the family about the use of lorazepam (Ativan). The nurse should instruct the family to report which of the following significant side effects to the health care provider? 1. Paradoxical excitement. 2. Headache. 3. Slowing of reflexes. 4. Fatigue.

1. Although all of the side effects listed are possible with Ativan, paradoxical excitement is cause for immediate discontinuation of the medication. (Paradoxical excitement is the opposite reaction to Ativan than is expected.) The other side effects tend to be minor and usually are transient.

The client with dementia states to the nurse, "I know you. You're Margaret, the girl who lives down the street from me." Which of the following responses by the nurse is most therapeutic? 1. "Mrs. Jones, I'm Rachel, a nurse here at the hospital." 2. "Now Mrs. Jones, you know who I am." 3. "Mrs. Jones, I told you already, I'm Rachel and I don't live down the street." 4. "I think you forgot that I'm Rachel, Mrs. Jones."

1. Because of the client's short-term memory impairment, the nurse gently corrects the client by stating her name and who she is. This approach decreases anxiety, embarrassment, and shame and maintains the client's self-esteem. Telling the client that she knows who the nurse is or that she forgot can elicit feelings of embarrassment and shame. Saying, "I told you already" sounds condescending, as if blaming the client for not remembering.

When developing the plan of care for a client with Alzheimer's disease who is experiencing moderate impairment, which of the following types of care should the nurse expect to include? 1. Prompting and guiding activities of daily living. 2. Managing a medication schedule. 3. Constant supervision and total care. 4. Supervision of risky activities such as shaving.

1. Considerable assistance is associated with moderate impairment when the client cannot make decisions but can follow directions. Managing medications is needed even in mild impairment. Constant care is needed in the terminal phase, when the client cannot follow directions. Supervision of shaving is appropriate with mild impairment— that is, when the client still has motor function but lacks judgment about safety issues.

The nurse is attempting to draw blood from a woman with a diagnosis of delirium who was admitted last evening. The client yells out, "Stop; leave me alone. What are you trying to do to me? What's happening to me?" Which response by the nurse is most appropriate? 1. "The tests of your blood will help us figure out what is happening to you." 2. "Please hold still so I don't have to stick you a second time." 3. "After I get your blood, I'll get some medicine to help you calm down." 4. "I'll tell you everything after I get your blood tests to the laboratory."

1. Explaining why blood is being taken responds to the client's concerns or fears about what is happening to her. Threatening more pain or promising to explain later ignores or postpones meeting the client's need for information. The client's statements do not reflect loss of self control requiring medication intervention.

In addition to developing over a period of hours or days, the nurse should assess delirium as distinguishable by which of the following characteristics? 1. Disturbances in cognition and consciousness that fluctuate during the day. 2. The failure to identify objects despite intact sensory functions. 3. Significant impairment in social or occupational functioning over time. 4. Memory impairment to the degree of being called amnesia.

1. Fluctuating symptoms are characteristic of delirium. The failure to identify objects despite intact sensory functions, significant impairment in social or occupational functioning over time, and memory impairment to the degree

Which of the following should the nurse expect to include as a priority in the plan of care for a client with delirium based on the nurse's understanding about the disturbances in orientation associated with this disorder? 1. Identifying self and making sure that the nurse has the client's attention. 2. Eliminating the client's napping in the daytime as much as possible. 3. Engaging the client in reminiscing with relatives or visitors. 4. Avoiding arguing with a suspicious client about his perceptions of reality.

1. Identifying oneself and making sure that the nurse has the client's attention addresses the difficulties with focusing, orientation, and maintaining attention. Eliminating daytime napping is unrealistic until the cause of the delirium is determined and the client's ability to focus and maintain attention improves. Engaging the client in reminiscing and avoiding arguing are also unrealistic at this time.

Discuss common causes of a head injury.

MVAs, falls, violent assaults, sports injuries, IEDs at war. the cause is what influences the kind of head injury they have.

The client in the early stage of Alzheimer's disease and his adult son attend an appointment at the community mental health center. While conversing with the nurse, the son states, "I'm tired of hearing about how things were 30 years ago. Why does Dad always talk about the past?" The nurse should tell the son: 1. "Your dad lost his short-term memory, but he still has his long-term memory." 2. "You need to be more accepting of your dad's behavior." 3. "I want you to understand your dad's level of anxiety." 4. "Telling your dad that you are tired of hearing about the past will help him stop."

1. The son's statements regarding his father's recalling past events is typical for family members of clients in the early stage of Alzheimer's disease, when recent memory is impaired. Telling the son to be more accepting is critical and not an attempt to educate. Understanding the client's level of anxiety is unrelated to the memory loss of Alzheimer's disease. The client cannot stop reminiscing at will.

When helping the families of clients with Alzheimer's disease cope with vulgar or sexual behaviors, which of the following suggestions is most helpful? 1. Ignore the behaviors, but try to identify the underlying need for the behaviors. 2. Give feedback on the inappropriateness of the behaviors. 3. Employ anger management strategies. 4. Administer the prescribed risperidone (Risperdal).

1. The vulgar or sexual behaviors are commonly expressions of anger or more sensual needs that can be addressed directly. Therefore, the families should be encouraged to ignore the behaviors but attempt to identify their purpose. Then the purpose can be addressed, possibly leading to a decrease in the behaviors. Because of impaired cognitive function, the client is not likely to be able to process the inappropriateness of the behaviors if given feedback. Likewise, anger management strategies would be ineffective because the client would probably be unable to process the inappropriateness of the behaviors. Risperidone (Risperdal) may decrease agitation, but it does not improve social behaviors.

Nursing staff are trying to provide for the safety of an elderly female client with moderate dementia. She is wandering at night and has trouble keeping her balance. She has fallen twice but has had no resulting injuries. The nurse should: 1. Move the client to a room near the nurse's station and install a bed alarm. 2. Have the client sleep in a reclining chair across from the nurse's station. 3. Help the client to bed and raise all four bedrails. 4. Ask a family member to stay with the client at night.

1. Using a bed alarm enables the staff to respond immediately if the client tries to get out of bed. Sleeping in a chair at the nurse's station interferes with the client's restful sleep and privacy. Using all four bedrails is considered a restraint and unsafe practice. It is not appropriate to expect a family member to stay all night with the client.

The husband of a client with Alzheimer's disease that was diagnosed 6 years ago approaches the nurse and says, "I'm so excited that my wife is starting to use donepezil (Aricept) for her illness." The nurse should tell the husband: 1. The medication is effective mostly in the early stages of the illness. 2. The adverse effects of the drug are numerous. 3. The client will attain a functional level of that of 6 years ago. 4. Effectiveness in the terminal phase of the illness is scientifically proven.

1. When compared with other similar medications, donepezil (Aricept) has fewer adverse effects. Donepezil is effective primarily in the early stages of the disease. The drug helps to slow the progression of the disease if started in the early stages. After the client has been diagnosed for 6 years, improvement to the level seen 6 years ago is highly unlikely. Data are not available to support the drug's effectiveness for clients in the terminal phase of the disease.

Which of the following questions would be easiest for a client with a hearing deficit to understand? 1. "Water?" 2. "Would you like a drink of water?" 3. "Want a drink?" 4. "Are you thirsty?"

1. "Water?"; A simple, clearly spoken, one-word question is less confusing and easier to understand than more complex phrases. Simple is more easily heard than complex. The more words there are in a sentence, the more likely it is that some will not be understood, causing a distortion in meaning.

What are the 4 areas of the spinal cord?

1. Cervical 2. Thoracic 3. Lumbar 4. Sacral

What is the correct way to instill eardrops in a sequential order?

1. Check order 2. Warm medication to body temperature 3. Wash your hands and wear clean gloves 4. Check patient ID and then explain the process 5. Have patient lie on side with affected ear up 6. Straighten ear canal by pulling auricle up and back 7. Gently clean drainage with tissue paper or cotton tipped applicator. 8. Support hand hold dropper against head 9. Hold dropper 1/2 inch above ear, aim at canal and administer. Administer water based drops before oil based 10. Gently massage ear to move meds. 11. Lie on side for 5-10 minutes and tuck cotton ball into ear 12. After 15 minutes, remove and dry ear.

What drugs help decrease the fluid in the eyeball and therefore the intraocular pressure?

1. DIAMOX - decreases production of fluid 2. MANNITOL - draws fluid out of the eyeball

What is the treatment for Meniere's disease?

1. Decreased sodium in diet 2. Stop smoking 3. Benadryl, atropine, Dramamine and 4. Lasix to decrease water in ear.

Peripheral neuropathy and paresthesias become the etiology for other nursing diagnoses. An example of such a diagnosis is: 1. Risk for injury 2. Impaired swallowing 3. Fluid volume overload 4. Social isolation

1. Risk for injury; The nurse determines what effects peripheral neuropathy will have on the client. Swallowing takes place in the posterior pharynx and esophagus. This is centrally located; unrelated to peripheral neuropathy. Fluid overload is related to excess intake relative to output, or organ failure such as heart failure, not sensory perception. It is unlikely paresthesias would cause social isolation.

Requirements for Epilepsy Surgery

1. Spells must be epileptic 2. Seizures must be focal (partial) a. All seizures must have the same focus 3. The focus must be in a region of brain that can be safely removed

Epilepsy syndromes

1. Symptomatic, localized -Post-stroke epilepsy, mesial temporal sclerosis 2. Symptomatic, generalized -Lennox-Gastaut syndrome 3. Idiopathic, localized -Benign childhood epilepsy 4. Idiopathic, generalized -Childhood absence epilepsy -Juvenile myoclonic epilepsy

What is the correct way to help a patient who is visually impaired?

1. Tell the patient each time you enter the room and say your name. 2. Always explain what you are going to do 3. When walking, ask which side they prefer you on, then offer your arm for them to grasp 4. Position self on half step beside and ahead of them

What is the primary goal collaboratively established by the client with Parkinson's disease, nurse, and physical therapist? 1. To maintain joint flexibility. 2. To build muscle strength. 3. To improve muscle endurance. 4. To reduce ataxia.

1. The primary goal of physical therapy and nursing interventions is to maintain joint flexibility and muscle strength. Parkinson's disease involves a degeneration of dopamine-producing neurons; therefore, it would be an unrealistic goal to attempt to build muscles or increase endurance. The decrease in dopamine neurotransmitters results in ataxia secondary to extrapyramidal motor system effects. Attempts to reduce ataxia through physical therapy would not be effective.

In planning care for the client who has had a stroke, the nurse should obtain a history of the client's functional status before the stroke because? 1. The rehabilitation plan will be guided by it. 2. Functional status before the stroke will help predict outcomes. 3. It will help the client recognize his physical limitations. 4. The client can be expected to regain much of his functioning.

1. The primary reason for the nursing assessment of a client's functional status before and after a stroke is to guide the plan. The assessment does not help to predict how far the rehabilitation team can help the client to recover from the residual effects of the stroke, only what plans can help a client who has moved from one functional level to another. The nursing assessment of the client's functional status is not a motivating factor.

What is the correct way to reinsert an artificial eye?

1. Thumb of nondominant hand raise upper eyelid. 2. Dominant hand grasp the eyeball so that indented part is facing the patient's nose and slide it in as far as possible. 3. Depress the lower lid and pull forward to cover prosthesis.

What is the correct way to remove an artificial eye?

1. Using your dominant hand, raise the upper eyelid with thumb. 2. Cup nondominant hand under clients lower lid 3. Apply slight pressure with index finger between brow and artificial eye and remove

What are the proper steps for obtaining a wound drainage specimen?

1. Wash hands 2. Use disposable gloves 3. Remove old dressing 4. Apply sterile gloves 5. Assess wound appearance (color, odor, et. al) 6. Irrigate wound with NaCl solution 7. Absorb excess with sterile gauze 8. Remove culture tube 9. Gently roll swab over granulation tissue 10. Replace in tube 11. Crush ampule of medium in tube. 12. Remove gloves 13. Apply sterile gloves 14. Dress wound 15. Label specimen 16. Arrange transport to laboratory

Pathogenesis

1. excessive neuronal excitability 2. synapse connections between hypersynchronous neurons - consider contributions of voltage gated ion channels in this process (AEDs act on these ion channels) - consider balance of GABAergic neurons (inhibitory pathways) and excitatory (esp Glutaminergic) neurons (excitatory pathways)

Differential Diagnosis for Seizures

1. syncope - warning of lightheadedness (eg sudden emotion) but no other manifestations 2. movement disorders - usually ongoing and variable duration; NOT STEREOTYPED 3. Sleep disorders - occur in spec phases of sleep 4. TIAs - usually negative sx 5. Migraines - longer duration, often w/ headache 6. Non-epileptic spells (less stereotyped, more variable than seizures, bilateral movements, asynchronous limb movements; side to side head turning)

What are the signs and symptoms of multiple sclerosis?

1. tinnitus 2. decreased hearing 3. urinary retention 4. spastic bladder 5. constipation 6. nystagmus 7. diplopia 8. blurred vision 9. dysarthria 10. dysphagia 11. numbness 12. tingling 13. weakness 14. paralysis 15. muscle spasticity 16. ataxia 17. vertigo

What are the signs and symptoms of MENIERE's disease?

1. vertigo 2. tinnitus 3. unilateral nerve deafness

Describe nursing care to assist the patient with mobility, gait, strength, and motor skills.

1.Reposition the client on a regular schedule as dictated by individual situation. ® Allow proper blood circulation, prevents venous stasis and formation of decubitus ulcers 2.Place patient on moderate high back rest position with head at the midline ® allows greater lung expansion and prevent compression on the diaphragm from prolong bed rest. 3.Support body part especially the affected side using pillows or rolls ®Prevention from developing pressure ulcers particularly on bony prominences 4.Keep body aligned and place extremities in proper position ® Proper positioning and turning maintains joint function and prevents contractures. 5.Perform active range of motion on unaffected extremities and passive range of motion exercises on affected extremities every 4 hours. ®Active range of motion exercise improves muscle strength while passive range of motion exercise improves joint mobility 6.Encourage patient to perform certain movements according to ones capability such as moving left upper and lower extremities, moving tongue, and moving head. ®To maintain strength and integrity of the functioning body parts. 7. Raise the siderails and provide a responsible watcher. ®weakness and loss of body coordination are at risk for fall or accidents. 8.Provide enteral feeding via NGT ®Provision of nutrition for metabolic and energy demand. 9. Perform regular skin care. (e.i sponge bath,apply lotion) ®Maintains skin integrity and decreases risk for skin breakdown. 10.Schedule activities with adequate rest periods ®To reduce fatigue and decrease energy demand 11. Provide a positive atmosphere while acknowledging ones difficulty. ®Helps minimize frustration and rechannel energy.

10. For a male client with suspected increased intracranial pressure (ICP), a most appropriate respiratory goal is to: a. prevent respiratory alkalosis. b. lower arterial pH. c. promote carbon dioxide elimination. d. maintain partial pressure of arterial oxygen (PaO2) above 80 mm Hg

10. Answer C. The goal of treatment is to prevent acidemia by eliminating carbon dioxide. That is because an acid environment in the brain causes cerebral vessels to dilate and therefore increases ICP. Preventing respiratory alkalosis and lowering arterial pH may bring about acidosis, an undesirable condition in this case. It isn't necessary to maintain a PaO2 as high as 80 mm Hg; 60 mm Hg will adequately oxygenate most clients.

11. Nurse Maureen witnesses a neighbor's husband sustain a fall from the roof of his house. The nurse rushes to the victim and determines the need to opens the airway in this victim by using which method? a. Flexed position b. Head tilt-chin lift c. Jaw thrust maneuver d. Modified head tilt-chin lift

11. Answer C. If a neck injury is suspected, the jaw thrust maneuver is used to open the airway. The head tilt-chin lift maneuver produces hyperextension of the neck and could cause complications if a neck injury is present. A flexed position is an inappropriate position for opening the airway.

12. The nurse is assessing the motor function of an unconscious male client. The nurse would plan to use which plan to use which of the following to test the client's peripheral response to pain? a. Sternal rub b. Nail bed pressure c. Pressure on the orbital rim d. Squeezing of the sternocleidomastoid muscle

12. Answer B. Motor testing in the unconscious client can be done only by testing response to painful stimuli. Nail bed pressure tests a basic peripheral response. Cerebral responses to pain are tested using sternal rub, placing upward pressure on the orbital rim, or squeezing the clavicle or sternocleidomastoid muscle.

13. A female client admitted to the hospital with a neurological problem asks the nurse whether magnetic resonance imaging may be done. The nurse interprets that the client may be ineligible for this diagnostic procedure based on the client's history of: a. Hypertension b. Heart failure c. Prosthetic valve replacement d. Chronic obstructive pulmonary disorder

13. Answer C. The client having a magnetic resonance imaging scan has all metallic objects removed because of the magnetic field generated by the device. A careful history is obtained to determine whether any metal objects are inside the client, such as orthopedic hardware, pacemakers, artificial heart valves, aneurysm clips, or intrauterine devices. These may heat up, become dislodged, or malfunction during this procedure. The client may be ineligible if significant risk exists.

14. A male client is having a lumbar puncture performed. The nurse would plan to place the client in which position? a. Side-lying, with a pillow under the hip b. Prone, with a pillow under the abdomen c. Prone, in slight-Trendelenburg's position d. Side-lying, with the legs pulled up and head bent down onto chest.

14. Answer D. The client undergoing lumbar puncture is positioned lying on the side, with the legs pulled up to the abdomen and the head bent down onto the chest. This position helps open the spaces between the vertebrae.

15. The nurse is positioning the female client with increased intracranial pressure. Which of the following positions would the nurse avoid? a. Head mildline b. Head turned to the side c. Neck in neutral position d. Head of bed elevated 30 to 45 degrees

15. Answer B. The head of the client with increased intracranial pressure should be positioned so the head is in a neutral midline position. The nurse should avoid flexing or extending the client's neck or turning the head side to side. The head of the bed should be raised to 30 to 45 degrees. Use of proper positions promotes venous drainage from the cranium to keep intracranial pressure down.

16. A female client has clear fluid leaking from the nose following a basilar skull fracture. The nurse assesses that this is cerebrospinal fluid if the fluid: a. Is clear and tests negative for glucose b. Is grossly bloody in appearance and has a pH of 6 c. Clumps together on the dressing and has a pH of 7 d. Separates into concentric rings and test positive of glucose

16. Answer D. Leakage of cerebrospinal fluid (CSF) from the ears or nose may accompany basilar skull fracture. CSF can be distinguished from other body fluids because the drainage will separate into bloody and yellow concentric rings on dressing material, called a halo sign. The fluid also tests positive for glucose.

17. A male client with a spinal cord injury is prone to experiencing automatic dysreflexia. The nurse would avoid which of the following measures to minimize the risk of recurrence? a. Strict adherence to a bowel retraining program b. Keeping the linen wrinkle-free under the client c. Preventing unnecessary pressure on the lower limbs d. Limiting bladder catheterization to once every 12 hours

17. Answer D. The most frequent cause of autonomic dysreflexia is a distended bladder. Straight catheterization should be done every 4 to 6 hours, and foley catheters should be checked frequently to prevent kinks in the tubing. Constipation and fecal impaction are other causes, so maintaining bowel regularity is important. Other causes include stimulation of the skin from tactile, thermal, or painful stimuli. The nurse administers care to minimize risk in these areas.

18. The nurse is caring for the male client who begins to experience seizure activity while in bed. Which of the following actions by the nurse would be contraindicated? a. Loosening restrictive clothing b. Restraining the client's limbs c. Removing the pillow and raising padded side rails d. Positioning the client to side, if possible, with the head flexed forward

18. Answer B. Nursing actions during a seizure include providing for privacy, loosening restrictive clothing, removing the pillow and raising side rails in the bed, and placing the client on one side with the head flexed forward, if possible, to allow the tongue to fall forward and facilitate drainage. The limbs are never restrained because the strong muscle contractions could cause the client harm. If the client is not in bed when seizure activity begins, the nurse lowers the client to the floor, if possible, protects the head from injury, and moves furniture that may injure the client. Other aspects of care are as described for the client who is in bed.

19. The nurse is assigned to care for a female client with complete right-sided hemiparesis. The nurse plans care knowing that this condition: a. The client has complete bilateral paralysis of the arms and legs. b. The client has weakness on the right side of the body, including the face and tongue. c. The client has lost the ability to move the right arm but is able to walk independently. d. The client has lost the ability to move the right arm but is able to walk independently.

19. Answer B. Hemiparesis is a weakness of one side of the body that may occur after a stroke. Complete hemiparesis is weakness of the face and tongue, arm, and leg on one side. Complete bilateral paralysis does not occur in this condition. The client with right-sided hemiparesis has weakness of the right arm and leg and needs assistance with feeding, bathing, and ambulating.

A client is at risk for sensory deprivation. Which of the following clinical signs would the nurse observe? Select all that apply. 1. Sleeplessness 2. Decreased attention span 3. Irritability 4. Excessive sleeping 5. Crying, depression

2, 4, & 5. Decreased attention span, irritability, crying, and depression. Options 1 & 3 are clinical signs of sensory overload.

The family of a client, diagnosed with Alzheimer's disease, wants to keep the client at home. They say that they have the most difficulty in managing his wandering. The nurse should instruct the family to do which of the following? (Select all that apply). 1. Ask the physician for a sleeping medication. 2. Install motion and sound detectors. 3. Have a relative sit with the client all night. 4. Have the client wear a Medical Alert bracelet. 5. Install door alarms and high door locks.

2, 4, 5. Motion and sound detectors, a Medical Alert bracelet, and door alarms are all appropriate interventions for wandering. Sleep medications do not prevent wandering before and after the client is asleep and may have negative effects. Having a relative sit with the client is usually an unrealistic burden.

The nurse is teaching a client with bladder dysfunction from multiple sclerosis (MS) about bladder training at home. Which instructions should the nurse include in the teaching plan? Select all that apply. 1. Restrict fluids to 1,000 mL/ 24 hours. 2. Drink 400 to 500 mL with each meal. 3. Drink fluids midmorning, midafternoon, and late afternoon. 4. Attempt to void at least every 2 hours. 5. Use intermittent catheterization as needed.

2,3,4,5

Which of the following is a priority to include in the plan of care for a client with Alzheimer's disease who is experiencing difficulty processing and completing complex tasks? 1. Repeating the directions until the client follows them. 2. Asking the client to do one step of the task at a time. 3. Demonstrating for the client how to do the task. 4. Maintaining routine and structure for the client.

2. Because the client is experiencing difficulty processing and completing complex tasks, the priority is to provide the client with only one step at a time, thereby breaking the task up into simple steps, ones that the client can process. Repeating the directions until the client follows them or demonstrating how to do the task is still too overwhelming to the client because of the multiple steps involved. Although maintaining structure and routine is important, it is unrelated to task completion.

A client with early dementia exhibits disturbances in her mental awareness and orientation to reality. The nurse should expect to assess a loss of ability in which of the following other areas? 1. Speech. 2. Judgment. 3. Endurance. 4. Balance.

2. Clients with chronic cognitive disorders experience defects in memory orientation and intellectual functions, such as judgment and discrimination. Loss of other abilities, such as speech, endurance, and balance, is less typical.

Which of the following is an initial sign of Parkinson's disease? 1. Rigidity. 2. Tremor. 3. Bradykinesia. 4. Akinesia.

2. The first sign of Parkinson's disease is usually tremors. The client commonly is the first to notice this sign because the tremors may be minimal at first. Rigidity is the second sign, and brady-kinesia is the third sign. Akinesia is a later stage of bradykinesia.

3

A nurse is planning a workshop on health promotion for older adults. Which topic will be included? 1. prevention of falls 2. cardiovascular risk factors 3. adequate sleep 4. how to stop smoking

A client diagnosed with dementia wanders the halls of the locked nursing unit during the day. To ensure the client's safety while walking in the halls, the nurse should do which of the following? 1. Administer PRN haloperidol (Haldol) to decrease the need to walk. 2. Assess the client's gait for steadiness. 3. Restrain the client in a geriatric chair. 4. Administer PRN lorazepam (Ativan) to provide sedation.

2. Elderly clients have increased risk for falls due to balance problems, medication use, and decreased eyesight. Haldol may cause extrapyramidal side effects (EPSE) which increase the risk for falls. The client is not agitated, so restraints are not indicated. Ativan may increase fall risk and cause paradoxical excitement.

The term motor apraxia relates to a decline in motor patterns essential for complex motor tasks. However, the client with severe dementia may be able to perform which of the following actions? 1. Balance a checkbook accurately. 2. Brush the teeth when handed a toothbrush. 3. Use confabulation when telling a story. 4. Find misplaced car keys.

2. Highly conditioned motor skills, such as brushing the teeth, may be retained by the client who has dementia and motor apraxia. Balancing a checkbook involves calculations, a complex skill that is lost with severe dementia Confabulation is fabrication of details to fill a memory gap. This is more common when the client is aware of a memory problem, not when dementia is severe. Finding keys is a memory factor, not a motor function.

A 69-year-old client is admitted and diagnosed with delirium. Later in the day, he tries to get out of the locked unit. He yells, "Unlock this door. I've got to go see my doctor. I just can't miss my monthly Friday appointment." Which of the following responses by the nurse is most appropriate? 1. "Please come away from the door. I'll show you your room." 2. "It's Tuesday and you are in the hospital. I'm Anne, a nurse." 3. "The door is locked to keep you from getting lost." 4. "I want you to come eat your lunch before you go the doctor."

2. Loss of orientation, especially for time and place, is common in delirium. The nurse should orient the client by telling him the time, date, place, and who the client is with. Taking the client to his room and telling him why the door is locked does not address his disorientation. Telling the client to eat before going to the doctor reinforces his disorientation.

A nurse on the Geropsychiatric unit receives a call from the son of a recently discharged client. He reports that his father just got a prescription for memantine (Namenda) to take "on top of his donepezil (Aricept)." The son then asks, "Why does he have to take extra medicines?" The nurse should tell the son: 1. "Maybe the Aricept alone isn't improving his dementia fast enough or well enough." 2. "Namenda and Aricept are commonly used together to slow the progression of dementia." 3. "Namenda is more effective than Aricept. Your father will be tapered off the Aricept." 4. "Aricept has a short half-life and Namenda has a long half-life. They work well together."

2. The two medicines are commonly given together. Neither medicine will improve dementia, but may slow the progression. Neither medicine is more effective than the other; they act differently in the brain. Both medicines have a half-life of 60 or more hours.

Which statement indicates the client needs a sensory aid in the home? 1. "I tripped over that throw rug again." 2. "I can't hear the doorbell." 3. "My eyesight is good if I wear my glasses." 4. "I can hear the TV if I turn it up high."

2. "I can't hear the doorbell."; This client could use an assistive device that flashes a light when the doorbell rings.

. The nurse is teaching the family of a client with dysphagia about decreasing the risk of aspiration while eating. Which of the following strategies is not appropriate? 1. Maintaining an upright position. 2. Restricting the diet to liquids until swallowing improves. 3. Introducing foods on the unaffected side of the mouth. 4. Keeping distractions to a minimum.

2. A client with dysphagia (difficulty swallowing) commonly has the most difficulty ingesting thin liquids, which are easily aspirated. Liquids should be thickened to avoid aspiration. Maintaining an upright position while eating is appropriate because it minimizes the risk of aspiration. Introducing foods on the unaffected side allows the client to have better control over the food bolus. The client should concentrate on chewing and swallowing; therefore, distractions should be avoided.

Which client is most likely to experience sensory deprivation? 1. A blind 93-year-old bedridden resident of a nursing home 2. A deaf 88-year-old single client with +4 edema who lives in an upstairs apartment 3. A child with genetic anomalies, abandoned in infancy, cared for in a special needs foster home, who attends preschool three times a week 4. A premature infant transferred to a Neonatal Intensive Care Unit

2. A deaf 88-year-old single client with +4 edema who lives in an upstairs apartment; Sensory stimulation comes from our senses, environment, and presence of meaningful data. Although the client has no sight and is unable to get out of bed, she is still capable and likely to receive sensory stimulation. She may converse with staff and other residents, feel the touch of bathing, and taste a variety of foods. There is a potential for sensory deprivation related to abandonment and the presence of anomalies. Since the child is being cared for in a special needs foster home, and attends preschool, one can reasonably assume that the child receives some stimulation. Premature infants in Neonatal Intensive Care Units often suffer from sensory overload.

The client has some equipment that is noisy, and the roommate also has equipment that makes noise, and the room is close to a noisy nursing station, where they can be watched a little closer. Which of the following interventions by the nurse would be best for the client as well as reduce the risk of sensory overload? 1. Move the client away from the nurses' station area. 2. Explain the sounds in the environment. 3. Tell the client to ignore the sounds. 4. Play the client's favorite music louder than the sounds.

2. Explain the sounds in the environment; Moving the client is not wise, as he and his roommate obviously need to be watched closely. Telling them to ignore the noise or playing music to cover it is not as helpful as explaining the sounds in the environment. When clients understand the meaning of the sounds, the stimuli are frequently less confusing and more easily ignored.

Which food-related behaviors are expected in a client who has had a stroke that has left him with homonymous hemianopia? 1. Increased preference for foods high in salt. 2. Eating food on only half of the plate. 3. Forgetting the names of foods. 4. Inability to swallow liquids.

2. Homonymous hemianopia is blindness in half of the visual field; therefore, the client would see only half of his plate. Eating only the food on half of the plate results from an inability to coordinate visual images and spatial relationships. There may be an increased preference for foods high in salt after a stroke, but this would not be related to homonymous hemianopia. Forgetting the names of foods would be aphasia, which involves a cerebral cortex lesion. Being unable to swallow liquids is dysphagia, which involves motor pathways of cranial nerves IX and X, including the lower brain stem.

What is a priority nursing assessment in the first 24 hours after admission of the client with a thrombotic stroke? 1. Cholesterol level. 2. Pupil size and pupillary response. 3. Bowel sounds. 4. Echocardiogram.

2. It is crucial to monitor the pupil size and pupillary response to indicate changes around the cranial nerves. The cholesterol level is not a priority assessment, although it may be an assessment to be addressed for long-term healthy lifestyle rehabilitation. Bowel sounds need to be assessed because an ileus or constipation can develop, but this is not a priority in the first 24 hours, when the primary concerns are cerebral hemorrhage and increased intracranial pressure. An echocardiogram is not needed for the client with a thrombotic stroke without heart problems.

A client with Parkinson's disease is prescribed levodopa (l-dopa) therapy. Improvement in which of the following indicates effective therapy? 1. Mood. 2. Muscle rigidity. 3. Appetite. 4. Alertness.

2. Levodopa is prescribed to decrease severe muscle rigidity. Levodopa does not improve mood, appetite, or alertness in a client with Parkinson's disease.

The culturally sensitive nurse will realize which of the following about a client from a large active Latino family who is put into isolation for a communicable disease? 1. The number of visitors greatly needs to be 2. May be accustomed to, and need, high stimulation level 3. Is a likely candidate for sensory overload 4. Will need more personal space than other clients

2. May be accustomed to, and need, high stimulation level; An individual's culture often determines the amount of stimulation that the client considers normal or usual. A person raised in a large active Latino family may be accustomed to more stimulation than an only child raised in a European American family. A decrease in sensory stimulation could result in sensory deprivation or culture shock.

Which of the following techniques does the nurse avoid when changing a client's position in bed if the client has hemiparalysis? 1. Rolling the client onto the side. 2. Sliding the client to move up in bed. 3. Lifting the client when moving the client up in bed. 4. Having the client help lift off the bed using a trapeze.

2. Sliding a client on a sheet causes friction and is to be avoided. Friction injures skin and predisposes to pressure ulcer formation. Rolling the client is an acceptable method to use when changing positions as long as the client is maintained in anatomically neutral positions and her limbs are properly supported. The client may be lifted as long as the nurse has assistance and uses proper body mechanics to avoid injury to himself or herself or the client. Having the client help lift herself off the bed with a trapeze is an acceptable means to move a client without causing friction burns or skin breakdown.

22. Nurse Kristine is trying to communicate with a client with brain attack (stroke) and aphasia. Which of the following actions by the nurse would be least helpful to the client? a. Speaking to the client at a slower rate b. Allowing plenty of time for the client to respond c. Completing the sentences that the client cannot finish d. Looking directly at the client during attempts at speech

22. Answer C. Clients with aphasia after brain attack (stroke) often fatigue easily and have a short attention span. General guidelines when trying to communicate with the aphasic client include speaking more slowly and allowing adequate response time, listening to and watching attempts to communicate, and trying to put the client at ease with a caring and understanding manner. The nurse would avoid shouting (because the client is not deaf), appearing rushed for a response, and letting family members provide all the responses for the client.

23. A female client has experienced an episode of myasthenic crisis. The nurse would assess whether the client has precipitating factors such as: a. Getting too little exercise b. Taking excess medication c. Omitting doses of medication d. Increasing intake of fatty foods

23. Answer C. Myasthenic crisis often is caused by undermedication and responds to the administration of cholinergic medications, such as neostigmine (Prostigmin) and pyridostigmine (Mestinon). Cholinergic crisis (the opposite problem) is caused by excess medication and responds to withholding of medications. Too little exercise and fatty food intake are incorrect. Overexertion and overeating possibly could trigger myasthenic crisis.

24. The nurse is teaching the female client with myasthenia gravis about the prevention of myasthenic and cholinergic crises. The nurse tells the client that this is most effectively done by: a. Eating large, well-balanced meals b. Doing muscle-strengthening exercises c. Doing all chores early in the day while less fatigued d. Taking medications on time to maintain therapeutic blood levels

24. Answer D. Clients with myasthenia gravis are taught to space out activities over the day to conserve energy and restore muscle strength. Taking medications correctly to maintain blood levels that are not too low or too high is important. Muscle-strengthening exercises are not helpful and can fatigue the client. Overeating is a cause of exacerbation of symptoms, as is exposure to heat, crowds, erratic sleep habits, and emotional stress.

25. A male client with Bell's palsy asks the nurse what has caused this problem. The nurse's response is based on an understanding that the cause is: a. Unknown, but possibly includes ischemia, viral infection, or an autoimmune problem b. Unknown, but possibly includes long-term tissue malnutrition and cellular hypoxia c. Primary genetic in origin, triggered by exposure to meningitis d. Primarily genetic in origin, triggered by exposure to neurotoxins

25. Answer A. Bell's palsy is a one-sided facial paralysis from compression of the facial nerve. The exact cause is unknown, but may include vascular ischemia, infection, exposure to viruses such as herpes zoster or herpes simplex, autoimmune disease, or a combination of these factors.

The nurse is reviewing the care plan of a client with Multiple Sclerosis. Which of the following nursing diagnoses should receive further validation? 1. Impaired mobility related to spasticity and fatigue. 2. Risk for falls related to muscle weakness and sensory loss. 3. Risk for seizures related to muscle tremors and loss of myelin. 4. Impaired skin integrity related bowel and bladder incontinence.

3

26. The nurse has given the male client with Bell's palsy instructions on preserving muscle tone in the face and preventing denervation. The nurse determines that the client needs additional information if the client states that he or she will: a. Exposure to cold and drafts b. Massage the face with a gentle upward motion c. Perform facial exercises d. Wrinkle the forehead, blow out the cheeks, and whistle

26. Answer A. Prevention of muscle atrophy with Bell's palsy is accomplished with facial massage, facial exercises, and electrical stimulation of the nerves. Exposure to cold or drafts is avoided. Local application of heat to the face may improve blood flow and provide comfort.

28. A female client with Guillian-Barre syndrome has ascending paralysis and is intubated and receiving mechanical ventilation. Which of the following strategies would the nurse incorporate in the plan of care to help the client cope with this illness? a. Giving client full control over care decisions and restricting visitors b. Providing positive feedback and encouraging active range of motion c. Providing information, giving positive feedback, and encouraging relaxation d. Providing intravaneously administered sedatives, reducing distractions and limiting visitors

28. Answer C. The client with Guillain-Barré syndrome experiences fear and anxiety from the ascending paralysis and sudden onset of the disorder. The nurse can alleviate these fears by providing accurate information about the client's condition, giving expert care and positive feedback to the client, and encouraging relaxation and distraction. The family can become involved with selected care activities and provide diversion for the client as well.

29. A male client has an impairment of cranial nerve II. Specific to this impairment, the nurse would plan to do which of the following to ensure client to ensure client safety? a. Speak loudly to the client b. Test the temperature of the shower water c. Check the temperature of the food on the delivery tray. d. Provide a clear path for ambulation without obstacles

29. Answer D. Cranial nerve II is the optic nerve, which governs vision. The nurse can provide safety for the visually impaired client by clearing the path of obstacles when ambulating. Testing the shower water temperature would be useful if there were an impairment of peripheral nerves. Speaking loudly may help overcome a deficit of cranial nerve VIII (vestibulocochlear). Cranial nerve VII (facial) and IX (glossopharyngeal) control taste from the anterior two thirds and posterior third of the tongue, respectively.

A client with multiple sclerosis (MS) is receiving baclofen (Lioresal). The nurse determines that the drug is effective when it achieves which of the following? 1. Induces sleep. 2. Stimulates the client's appetite. 3. Relieves muscular spasticity. 4. Reduces the urine bacterial count.

3

The physician orders risperidone (Risperdal) for a client with Alzheimer's disease. The nurse anticipates administering this medication to help decrease which of the following behaviors? 1. Sleep disturbances. 2. Concomitant depression. 3. Agitation and assaultiveness. 4. Confusion and withdrawal.

3. Antipsychotics are most effective with agitation and assaultiveness. Antipsychotics have little effect on sleep disturbances, concomitant depression, or confusion and withdrawal.

When communicating with the client who is experiencing dementia and exhibiting decreased attention and increased confusion, which of the following interventions should the nurse employ as the first step? 1. Using gentle touch to convey empathy. 2. Rephrasing questions the client doesn't understand. 3. Eliminating distracting stimuli such as turning off the television. 4. Asking the client to go for a walk while talking.

3. Competing and excessive stimuli lead to sensory overload and confusion. Therefore, the nurse should first eliminate any distracting stimuli. After this is accomplished, then using touch and rephrasing questions are appropriate. Going for a walk while talking has little benefit on attention and confusion.

Which of the following is a realistic short-term goal to be accomplished in 2 to 3 days for a client with delirium? 1. Explain the experience of having delirium. 2. Resume a normal sleep-wake cycle. 3. Regain orientation to time and place. 4. Establish normal bowel and bladder function.

3. In approximately 2 to 3 days, the client should be able to regain orientation and thus become oriented to time and place. Being able to explain the experience of having delirium is something that the client is expected to achieve later in the course of the illness, but ultimately before discharge. Resuming a normal sleep-wake cycle and establishing normal bowel and bladder function probably will take longer, depending on how long it takes to resolve the underlying condition.

When caring for the client diagnosed with delirium, which condition is the most important for the nurse to investigate? 1. Cancer of any kind. 2. Impaired hearing. 3. Prescription drug intoxication. 4. Heart failure.

3. Polypharmacy is much more common in the elderly. Drug interactions increase the incidence of intoxication from prescribed medications, especially with combinations of analgesics, digoxin, diuretics, and anticholinergics. With drug intoxication, the onset of the delirium typically is quick. Although cancer, impaired hearing, and heart failure could lead to delirium in the elderly, the onset would be more gradual.

The nurse observes a client in a group who is reminiscing about his past. Which effect should the nurse expect reminiscing to have on the client's functioning in the hospital? 1. Increase the client's confusion and disorientation. 2. Cause the client to become sad. 3. Decrease the client's feelings of isolation and loneliness. 4. Keep the client from participating in therapeutic activities.

3. Reminiscing can help reduce depression in an elderly client and lessens feelings of isolation and loneliness. Reminiscing encourages a focus on positive memories and accomplishments as well as shared memories with other clients. An increase in confusion and disorientation is most likely the result of other cognitive and situational factors, such as loss of short-term memory, not reminiscing. The client will not likely become sad because reminiscing helps the client connect with positive memories. Keeping the client from participating in therapeutic activities is less likely with reminiscing.

The nurse discusses the possibility of a client's attending day treatment for clients with early Alzheimer's disease. Which of the following is the best rationale for encouraging day treatment? 1. The client would have more structure to his day. 2. Staff are excellent in the treatment they offer clients. 3. The client would benefit from increased social interaction. 4. The family would have more time to engage in their daily activities.

3. The best rationale for day treatment for the client with Alzheimer's disease is the enhancement of social interactions. More daily structure, excellent staff, and allowing caregivers more time for themselves are all positive aspects, but they are less focused on the client's needs.

The client with Alzheimer's disease may have delusions about being harmed by staff and others. When the client expresses fear of being killed by staff, which of the following responses is most appropriate? 1. "What makes you think we want to kill you?" 2. "We like you too much to want to kill you." 3. "You are in the hospital. We are nurses trying to help you." 4. "Oh, don't be so silly. No one wants to kill you here."

3. The nurse needs to present reality without arguing with the delusions. Therefore, stating that the client is in the hospital and the nurses are trying to help is most appropriate. The client doesn't recognize the delusion or why it exists. Telling the client that the staff likes him too much to want to kill him is inappropriate because the client believes the delusions and doesn't know that they are false beliefs. It also restates the word, kill, which may reinforce the client's delusions. Telling the client not to be silly is condescending and disparaging and therefore inappropriate.

A client with dementia who prefers to stay in his room has been brought to the dayroom. After 10 minutes, the client becomes agitated and retreats to his room again. The nurse decides to assess the conditions in the dayroom. Which is the most likely occurrence that is disturbing to this client? 1. There is only one other client in the dayroom; the rest are in a group session in another room. 2. There are three staff members and one physician in the nurse's station working on charting. 3. A relaxation tape is playing in one corner of the room, and a television airing a special on crime is playing in the opposite corner. 4. A housekeeping staff member is washing off the countertops in the kitchen, which is on the far side of the dayroom.

3. The tape and television are competing, even conflicting, stimuli. Crime events portrayed on television could be misperceived as a real threat to the client. A low number of clients and the presence of a few staff members quietly working are less intense stimuli for the client and not likely to be disturbing.

A client is experiencing mood swings after a stroke and often has episodes of tearfulness that are distressing to the family. Which is the best technique for the nurse to instruct family members to try when the client experiences a crying episode? 1. Sit quietly with the client until the episode is over. 2. Ignore the behavior. 3. Attempt to divert the client's attention. 4. Tell the client that this behavior is unacceptable.

3. A client who has brain damage may be emotionally labile and may cry or laugh for no explainable reason. Crying is best dealt with by attempting to divert the client's attention. Ignoring the behavior will not affect the mood swing or the crying and may increase the client's sense of isolation. Telling the client to stop is inappropriate.

The nurse advises a woman considering pregnancy of the importance of being tested for syphilis and rubella. What is most likely the reason the nurse is offering this advice? 1. Suspicion that a client has high risk of sexual and drug behaviors 2. Assess factors that mainly cause visual impairments in baby 3. Assess factors that mainly cause hearing impairments in baby 4. The health history assessment findings

3. Assess factors that mainly cause hearing impairments in baby; Women who are considering pregnancy should be advised of the importance of testing for syphilis and rubella, which can cause hearing impairments in newborns.

In responding to visceral stimuli, the client would be most likely to experience which of the following? 1. Being aware train is coming because of hearing whistle 2. Being aware of which foot is forward when walking 3. Awareness of a full stomach 4. Being aware of an unpleasant smell

3. Awareness of a full stomach; Visceral refers to organs that may produce stimuli that make a person aware of them, e.g., a full stomach.

During the first 24 hours after thrombolytic treatment for an ischemic stroke, the primary goal is to control the client's: 1. Pulse. 2. Respirations. 3. Blood pressure. 4. Temperature.

3. Control of blood pressure is critical during the first 24 hours after treatment because an intracerebral hemorrhage is the major adverse effect of thrombolytic therapy. Vital signs are monitored, and blood pressure is maintained as identified by the physician and specific to the client's ischemic tissue needs and risk of bleeding from treatment. The other vital signs are important, but the priority is to monitor blood pressure.

An alert 80-year-old client is transferred to a long-term care facility. On the second night he becomes confused and agitated. What is the most appropriate nursing diagnosis? 1. Chronic Confusion 2. Impaired Memory 3. Disturbed Sensory Perception 4. Disturbed Thought Processes

3. Disturbed sensory perception; The transfer to a different setting can change the amount or pattern of incoming stimuli, and the client may have a diminished, exaggerated, distorted, or impaired response to such stimuli. Disturbed Thought Processes is applied when cognitive abilities (e.g., dementia) interfere with the ability to interpret stimuli accurately. Options 1 & 2 offer no evidence to support chronic confusion or impaired memory.

Nurses can increase environmental stimuli for clients with sensory deficit by: 1. Keeping the radio on throughout the day to provide auditory stimulation 2. Keeping the bathroom light on at night to avoid complete darkness 3. Establishing a routine identified with each meal 4. Ensuring the client's safety

3. Establishing a routine identified with each meal; Regular meaningful stimuli will benefit the client. The radio can provide meaningful or meaningless stimuli. The nurse must carefully choose programming based on the client's preferences and expose the client to that programming only at appropriate times. Listening to the radio constantly can introduce meaningless stimuli that confuse the client. A 24-hour light may actually keep clients awake, leading to sleep deprivation. Safety is a priority diagnosis but is not an intervention to provide environmental stimuli.

You are about to bathe an unconscious client. Which of the following interventions are most important on your part? 1. Vary the schedule of bathing and care from day to day. 2. Tune the radio to client's favorite music during bath time. 3. Explain procedures to client, and talk as if client can hear. 4. Speak louder to the client than to other clients.

3. Explain procedures to client, and talk as if client can hear; The person who is unconscious and unable to respond to the spoken word nevertheless can often hear what is spoken. It is important for nurses to speak in a normal tone of voice and before touching the client. The environmental noise should be kept at a minimum so client can focus on words. Maintain the same schedule every day.

A client is exhibiting signs and symptoms of acute confusion/delirium. Which strategy should the nurse implement to promote a therapeutic environment? 1. Keep lights in the room dimmed during the day to decrease stimulation. 2. Keep the environmental noise level high to increase stimulation. 3. Keep the room organized and clean. 4. Use restraints for client safety.

3. Keep the room organized and clean; A disorganized, cluttered environment increases confusion. Keeping the room well-lit during waking hours (option 1) promotes adequate sleep at night. It is important to eliminate unnecessary noise (option 2). Client does not meet the standard criteria for restraint application (option 4).

A client arrives in the emergency department with an ischemic stroke and receives tissue plasminogen activator (t-PA) administration. The nurse should first: 1. Ask what medications the client is taking. 2. Complete a history and health assessment. 3. Identify the time of onset of the stroke. 4. Determine if the client is scheduled for any surgical procedures.

3. Studies show that clients who receive recombinant t-PA treatment within 3 hours after the onset of a stroke have better outcomes. The time from the onset of a stroke to t-PA treatment is critical. A complete health assessment and history is not possible when a client is receiving emergency care. Upcoming surgical procedures may need to be delayed because of the administration of t-PA, which is a priority in the immediate treatment of the current stroke. While the nurse should identify which medications the client is taking, it is more important to know the time of the onset of the stroke to determine the course of action for administering t-PA.

The nurse develops a teaching plan for a client newly diagnosed with Parkinson's disease. Which of the following topics that the nurse plans to discuss is the most important? 1. Maintaining a balanced nutritional diet. 2. Enhancing the immune system. 3. Maintaining a safe environment. 4. Engaging in diversional activity.

3. The primary focus is on maintaining a safe environment because the client with Parkinson's disease usually has a propulsive gait, characterized by a tendency to take increasingly quicker steps while walking. This type of gait commonly causes the client to fall or to have trouble stopping. The client should maintain a balanced diet, enhance the immune system, and enjoy diversional activities;

Which nursing intervention has been found to be the most effective means of preventing plantar flexion in a client who has had a stroke with residual paralysis? 1. Place the client's feet against a firm footboard. 2. Reposition the client every 2 hours. 3. Have the client wear ankle-high tennis shoes at intervals throughout the day. 4. Massage the client's feet and ankles regularly.

3. The use of ankle-high tennis shoes has been found to be most effective in preventing plantar flexion (footdrop) because they add support to the foot and keep it in the correct anatomic position. Footboards stimulate spasms and are not routinely recommended. Regular repositioning and range-of-motion exercises are important interventions, but the client's foot needs to be in the correct anatomic position to prevent overextension of the muscle and tendon. Massaging does not prevent plantar flexion and, if rigorous, could release emboli.

What is the expected outcome of thrombolytic drug therapy for stroke? 1. Increased vascular permeability. 2. Vasoconstriction. 3. Dissolved emboli. 4. Prevention of hemorrhage.

3. Thrombolytic enzyme agents are used for clients with a thrombotic stroke to dissolve emboli, thus reestablishing cerebral perfusion. They do not increase vascular permeability, cause vasoconstriction, or prevent further hemorrhage.

30. A female client has a neurological deficit involving the limbic system. Specific to this type of deficit, the nurse would document which of the following information related to the client's behavior. a. Is disoriented to person, place, and time b. Affect is flat, with periods of emotional lability c. Cannot recall what was eaten for breakfast today d. Demonstrate inability to add and subtract; does not know who is president

30. Answer B. The limbic system is responsible for feelings (affect) and emotions. Calculation ability and knowledge of current events relates to function of the frontal lobe. The cerebral hemispheres, with specific regional functions, control orientation. Recall of recent events is controlled by the hippocampus.

When the nurse talks with a client with multiple sclerosis who has slurred speech, which nursing intervention is contraindicated? 1. Encouraging the client to speak slowly. 2. Encouraging the client to speak distinctly. 3. Asking the client to repeat indistinguishable words. 4. Asking the client to speak louder when tired.

4

(SATA) The client who has had a stroke with residual physical handicaps becomes discouraged by his physical appearance. What approach to the client is best for the nurse to use to help the client overcome his negative self-concept? Select all that apply. 1. Helpfulness. 2. Charity. 3. Firmness. 4. Encouragement. 5. Patience.

4, 5. When offering emotional support to a client who is discouraged and has a negative self-concept because of physical handicaps, the nurse should approach the client with encouragement and patience. The client should be praised when he or she shows progress in efforts to overcome handicaps. An attitude of helpfulness and sympathy allows the client to assume a role of someone not ordinary, someone who is not like others. Regardless of the handicap, the client still feels the same on the inside and has the same innate needs for his or her growth and developmental age-group. An attitude of charity tends to make the client feel like a "charity case" or like someone who is given something free because of his "condition." The client feels unequal to his peers or unable to fulfill the role relationships that were obtained before the stroke. An approach using firmness is inappropriate because it implies that the client can do better if he just tries harder and leaves no room for softness in the approach to overcoming a negative self-concept.

When providing family education for those who have a relative with Alzheimer's disease about minimizing stress, which of the following suggestions is most relevant? 1. Allow the client to go to bed four to five times during the day. 2. Test the cognitive functioning of the client several times a day. 3. Provide reality orientation even if the memory loss is severe. 4. Maintain consistency in environment, routine, and caregivers.

4. Change increases stress. Therefore, the most important and relevant suggestion is to maintain consistency in the client's environment, routine, and caregivers. Although rest periods are important, going to bed interferes with the sleep-wake cycle. Rest in a recliner chair is more useful. Testing cognitive functioning and reality orientation are not likely to be successful and may increase stress if memory loss is severe.

An elderly woman's husband died. When her brother arrives for the funeral, he notices her short-term memory problems and occasional disorientation. A few weeks later, she calls him to say that her husband just died. She says, "I didn't know he was so sick. Why did he die now?" She also complains of not sleeping, urinary frequency and burning, and seeing rats in the kitchen. A home care nurse is sent to evaluate her situation and finds the woman reclusive and passive, but pleasant. The nurse calls the woman's primary care physician to discuss the client's situation and background, and give his assessment and recommendations. The nurse concludes that the woman: 1. Is experiencing the onset of Alzheimer's disease. 2. Is having trouble adjusting to living alone without her husband. 3. Is having delayed grieving related to her Alzheimer's disease. 4. Is experiencing delirium and a urinary tract infection.

4. Delirium is commonly due to a medical condition such as a UTI in the elderly. Delirium often involves memory problems, disorientation, and hallucinations. It develops rather quickly. There is not enough data to suggest Alzheimer's disease especially given the quick onset of symptoms. Delayed grieving and adjusting to being alone are unlikely to cause hallucinations.

The son of an elderly client who has cognitive impairments approaches the nurse and says, "I'm so upset. The physician says I have 4 days to decide on where my dad is going to live." The nurse responds to the son's concerns, gives him a list of types of living arrangements, and discusses the needs, abilities, and limitations of the client. The nurse should intervene further if the son makes which comment? 1. "Boy, I have a lot to think about before I see the social worker tomorrow." 2. "I think I can handle most of Dad's needs with the help of some home health care." 3. "I'm so afraid of making the wrong decision, but I can move him later if I need to." 4. "I want the social worker to make this decision so Dad won't blame me."

4. Expecting the social worker to make the decision indicates that the son is avoiding participating in decisions about his father. The other responses convey that the son understands the importance of a careful decision, the availability of resources, and the ability to make new plans if needed.

While educating the daughter of a client with dementia about the illness, the daughter complains to the nurse that her mother distorts things. The nurse understands that the daughter needs further teaching about dementia when she makes which statement? 1. "I tell her reality, such as, 'That noise is the wind in the trees.'" 2. "I understand the misperceptions are part of the disease." 3. "I turn off the radio when we're in another room." 4. "I tell her she is wrong and then I tell her what's right."

4. Telling the client that she is wrong and then telling her what is right is argumentative and challenging. Arguing with or challenging distortions is least effective because it increases defensiveness. Telling the client about reality indicates awareness of the issues and is appropriate. Acknowledging that misperceptions are part of the disease indicates an understanding of the disease and an awareness of the issues. Turning off the radio helps to limit environmental stimuli and indicates an awareness of the issues.

4. A female client with Guillain-Barré syndrome has paralysis affecting the respiratory muscles and requires mechanical ventilation. When the client asks the nurse about the paralysis, how should the nurse respond? a. "You may have difficulty believing this, but the paralysis caused by this disease is temporary." b. "You'll have to accept the fact that you're permanently paralyzed. However, you won't have any sensory loss." c. "It must be hard to accept the permanency of your paralysis." d. "You'll first regain use of your legs and then your arms."

4. Answer A. The nurse should inform the client that the paralysis that accompanies Guillain-Barré syndrome is only temporary. Return of motor function begins proximally and extends distally in the legs.

A 90-year-old client diagnosed with major depression is suddenly experiencing sleep disturbances, inability to focus, poor recent memory, altered perceptions, and disorientation to time and place. Lab results indicate the client has a urinary tract infection and dehydration. After explaining the situation and giving the background and assessment data, the nurse should make which of the following recommendations to the client's physician? 1. An order to place the client in restraints. 2. A reevaluation of the client's mental status. 3. The transfer of the client to a medical unit. 4. A transfer of the client to a nursing home.

4. The client is showing symptoms of delirium, a common outcome of UTI in older adults. The nurse can request a transfer to a medical unit for acute medical intervention. The client's symptoms are not just due to a worsening of the depression. There are not indications that the client needs restraints or a transfer to a nursing home at this point.

The nurse determines that the son of a client with Alzheimer's disease needs further education about the disease when he makes which of the following statements? 1. "I didn't realize the deterioration would be so incapacitating." 2. "The Alzheimer's support group has so much good information." 3. "I get tired of the same old stories, but I know it's important for Dad." 4. "I woke up this morning expecting that my old Dad would be back."

4. The statement about expecting that the old Dad would be back conveys a lack of acceptance of the irreversible nature of the disease. The statement about not realizing that the deterioration would be so incapacitating is based in reality. The statement about the Alzheimer's group is based in reality and demonstrates the son's involvement with managing the disease. Stating that reminiscing is important reflects a realistic interpretation on the son's part.

Which client is at greatest risk for experiencing sensory overload? 1. A 40-year-old client in isolation with no family. 2. A 28-year-old quadriplegic client in a private room. 3. A 16-year-old listening to loud music. 4. An 80-year-old client admitted for emergency surgery.

4. An 80-year-old client admitted for emergency surgery; A sudden, unexpected admission for surgery may involve many experiences (e.g., lab work, x-rays, signing of forms) while the client is in pain or discomfort. The time for orientation will thus be lessened. After surgery, the client may be in pain and possibly in a critical care setting. Options 1 and 2 would more likely be at risk for sensory deprivation. Option 3 is considered a normal activity for most teenagers.

Number of lumbar nerves

5

65. Which goal is the most realistic and appropriate for a client diagnosed with Parkinson's disease? 1. To cure the disease. 2. To stop progression of the disease. 3. To begin preparations for terminal care. 4. To maintain optimal body function.

4. Helping the client function at his or her best is most appropriate and realistic. There is no known cure for Parkinson's disease. Parkinson's disease progresses in severity, and there is no known way to stop its progression. Many clients live for years with the disease, however, and it would not be appropriate to start planning terminal care at this time.

Clients admitted into the emergency department may experience behavior changes due to: 1. Sensoristasis 2. Sensory reception 3. Stereognosis 4. Sensory overload

4. Sensory overload; Sensoristasis is time of optimum arousal, not too much or too little. Sensory reception is the process of receiving internal and external data. This is partially correct, in that the client does receive data that may result in behavior changes. However, Answer 4 is a better answer in that it more directly addresses the situation presented. Stereognosis is the awareness of an object's size, shape, and texture.

A nurse is teaching a client who had a stroke about ways to adapt to a visual disability. Which does the nurse identify as the primary safety precaution to use? 1. Wear a patch over one eye. 2. Place personal items on the sighted side. 3. Lie in bed with the unaffected side toward the door. 4. Turn the head from side to side when walking.

4. To expand the visual field, the partially sighted client should be taught to turn the head from side to side when walking. Neglecting to do so may result in accidents. This technique helps maximize the use of remaining sight. Covering an eye with a patch will limit the field of vision. Personal items can be placed within sight and reach, but most accidents occur from tripping over items that cannot be seen. It may help the client to see the door, but walking presents the primary safety hazard.

What is the mortality rate associated with subarachnoid hemorrhage?

40% mortality rate

5. The nurse is working on a surgical floor. The nurse must logroll a male client following a: a. laminectomy. b. thoracotomy. c. hemorrhoidectomy. d. cystectomy.

5. Answer A. The client who has had spinal surgery, such as laminectomy, must be logrolled to keep the spinal column straight when turning. The client who has had a thoracotomy or cystectomy may turn himself or may be assisted into a comfortable position. Under normal circumstances, hemorrhoidectomy is an outpatient procedure, and the client may resume normal activities immediately after surgery.

7. During a routine physical examination to assess a male client's deep tendon reflexes, the nurse should make sure to: a. use the pointed end of the reflex hammer when striking the Achilles tendon. b. support the joint where the tendon is being tested. c. tap the tendon slowly and softly d. hold the reflex hammer tightly.

7. Answer B. To prevent the attached muscle from contracting, the nurse should support the joint where the tendon is being tested. The nurse should use the flat, not pointed, end of the reflex hammer when striking the Achilles tendon. (The pointed end is used to strike over small areas, such as the thumb placed over the biceps tendon.) Tapping the tendon slowly and softly wouldn't provoke a deep tendon reflex response. The nurse should hold the reflex hammer loosely, not tightly, between the thumb and fingers so it can swing in an arc.

8. A female client is admitted in a disoriented and restless state after sustaining a concussion during a car accident. Which nursing diagnosis takes highest priority in this client's plan of care? a. Disturbed sensory perception (visual) b. Self-care deficient: Dressing/grooming c. Impaired verbal communication d. Risk for injury

8. Answer D. Because the client is disoriented and restless, the most important nursing diagnosis is risk for injury. Although the other options may be appropriate, they're secondary because they don't immediately affect the client's health or safety.

9. A female client with amyotrophic lateral sclerosis (ALS) tells the nurse, "Sometimes I feel so frustrated. I can't do anything without help!" This comment best supports which nursing diagnosis? a. Anxiety b. Powerlessness c. Ineffective denial d. Risk for disuse syndrome

9. Answer B. This comment best supports a nursing diagnosis of Powerlessness because ALS may lead to locked-in syndrome, characterized by an active and functioning mind locked in a body that can't perform even simple daily tasks. Although Anxiety and Risk for disuse syndrome may be diagnoses associated with ALS, the client's comment specifically refers to an inability to act autonomously. A diagnosis of Ineffective denial would be indicated if the client didn't seem to perceive the personal relevance of symptoms or danger.

Identify specific medications and usage for various sign/symptoms of MS.

ABC+R - Avonex, Betaseron, Copaxone. All for relapsing and/or reducing lesions.

How do you describe a MACULAR DEGENERATION vision?

A blind spot / black spot in the center of the vision field. Peripheral vision is retained.

Who is eligible for epilepsy surgery? (what %)

About half of patients with partial seizures who don't respond to AEDs are eligible for surgery.

c

A nurse and a primary care provider inform a client that chemotherapy is recommended for a diagnosis of cancer. Which nursing action is most representative of the concept of holism? a. Offer to come to the client's home to provide needed physical care b. Contact the client's spiritual advisor c. Inquire how this will affect other aspects of the client's life d. Provide the client with information about how to join a support group 1. ch 16

Discuss the age related (pediatric) nursing care and interdisciplinary care for a head injury.

A child advocacy team or child protective services should be contacted if child abuse is suspected, the mechanism of injury is unknown or unexplained, or the history is inconsistent. NB shock is rarely due to isolated head injury except in young children and in patients with medullary injuries or large scalp lacerations. Pediatric head injury has unique issues that make patient management and outcome different from that of adult head injury. Age related aspects will determine a greater or lesser degree of craniocervical junction injuries (disproportionate cranial size to trunk in infancy and early childhood). Other factors are potential underlying congenital anomalies, physiological factors (cerebrovascular reactivity and blood flow), differing support systems needed from that of adults for neuro imaging and specialized medical, nursing and allied health care support. Pediatric rehabilitation and educational needs and goals are different to that of adult head injury. intubating a child is harder than intubating and adult The physical exam is frequently normal CT scan = significant radiation exposure children sometimes cannot talk but frequently vomit due to stress (instead of head injury) Brain is less myelinated, results in greater sensitivity to shearing forces Cranial bones thinner, resulting in greater transmission of a single force to brain Non-fused sutures makes skull easily deformable Children (particularly < 24 months old) are at increased risk of cerebral hypo-perfusion after TBI

What is a cataract?

A condition in which the lens of the eye becomes opaque or cloudy and making vision difficult. It is usually a slow gradual loss of vision in the affected eye. They are common in elderly but may also be caused by some drugs like corticosteroids.

What is TRIGEMINAL NEURALGIA? What are the considerations?

A cranial nerve disorder affecting sensory branches of the trigeminal nerve (CN V). Lukewarm food, chew on unaffected side, eye care, tearing, blinking, oral hygiene, increased protein, calories, room temperature and avoid touching client.

How do you describe a RETINAL DETACHMENT vision?

A curtain in front of or partially over your eye.

a

A major goal of treament for the patient with AD is to: a. maintain patient safety b. maintain or increase body weight c. return to a higher level of self care d. enhance functional ability over time

What is HOMONYMOUS HEMIANOPSIA?

A pupillary abnormality associated with CVA where vision is lost on the same side of each eye so when driving must turn head to see (CN II is affected)

What is retinal detachment?

A small horseshoe shaped tear that allows sub retinal fluid to leak between the retinal pigment epithelium and the layer of rods and cones. This leakage splits the retinas outer layer, pulling it away from its blood supply.

Coma

A state of being unaware of one's surroundings and being unable to react or respond to people, places, or things.

Discuss the discharge-teaching plan for anticoagulants (and anti-platelets).

A thorough review of the dosage regimen, possible adverse drug reactions, and early signs of bleeding tendencies help the patient cooperate with the prescribed therapy. Teach: -Follow the dosage schedule prescribed by the PHCP, and report any signs of active bleeding immediately. (gums bleeding, bruising, bloody stools, black and tarry stools, vomit that is bright red or looks like coffee grounds). If these are found, d/c the next dose and contact your PHCP immediately. -The INR will be monitored periodically. Keep all appointments, because dosage changes may be necessary. -Do not take or stop taking other drugs except on the advice of the PHCP. -Inform your dentist and other PHCP of therapy. -Take the drug at the same time each day. -Do not change brands of anticoagulants without consulting a physician or pharmacist. -Avoid alcohol unless use has been approved by the primary health care provider. -Be aware of foods high in vit-K, such as leafy green vegetables, beans, broccoli, cabbage, cauliflower, cheese, fish, and yogurt. Maintaining a healthy diet including these foods may help maintain a consistent INR value. -Keep in mind that anti-platelet drugs can lower all blood counts, including the WBC count. Patients may be at greater risk for infection during the first 3 months of treatment. -Use a soft toothbrush. -Use an electric razor when possible. -Wear or carry medical identification.

A patient with a suspected closed head injury has bloody nasal drainage. The nurse suspects that this patient has a cerebrospinal fluid (CSF) leak when observing which of the following? A) A halo sign on the nasal drip pad B) Decreased blood pressure and urinary output C) A positive reading for glucose on a Test-tape strip D) Clear nasal drainage along with the bloody discharge

A) Halo sign on the nasal drip pad When drainage containing both CSF and blood is allowed to drip onto a white pad, within a few minutes the blood will coalesce into the center, and a yellowish ring of CSF will encircle the blood, giving a halo effect. The presence of glucose would be unreliable for determining the presence of CSF because blood also contains glucose.

Magnetic resonance imaging (MRI) has revealed the presence of a brain tumor in a patient. The nurse would recognize the patient's likely need for which of the following treatment modalities? A) Surgery B) Chemotherapy C) Radiation therapy D) Pharmacologic treatment

A) Surgery Surgical removal is the preferred treatment for brain tumors.

The nurse is caring for a patient admitted for evaluation and surgical removal of a brain tumor. The nurse will plan interventions for this patient based on knowledge that brain tumors can lead to which of the following complications (select all that apply)? A) Vision loss B) Cerebral edema C) Pituitary dysfunction D) Parathyroid dysfunction E) Focal neurologic deficits

A,B,C,E Brain tumors can manifest themselves in a wide variety of symptoms depending on location, including vision loss and focal neurologic deficits. Tumors that put pressure on the pituitary can lead to dysfunction of the gland. As the tumor grows, clinical manifestations of increased intracranial pressure (ICP) and cerebral edema can appear. The parathyroid gland is not regulated by the cerebral cortex or the pituitary gland.

An internal corneal reflex would be demonstrated by which of the following? A. a bilateral blink B. a bilateral pupillary constriction C. a bilateral horizontal deviation of the eye D. bilateral tearing

A. a bilateral blink reflect to protect the eyes from damage by foreign objects

Tuning fork hearing tests measure hearing by which of the following mechanisms? A. air/bone conduction B. bone/vestibular conduction C. air/water conduction D. bone/water conduction

A. air/bone conduction

When scheduling electroconvulsive therapy (ECT), which client should the nurse prioritize? A. A client in bed in a fetal position who is experiencing active suicidal ideations B. A client with an irritable mood and exhibiting angry outbursts C. A client experiencing command hallucinations and delusions of reference D. A client experiencing manic episodes of bipolar disorder

ANS: A A client who is experiencing suicidal ideations is in need of an immediate intervention to prevent self-harm and must be prioritized when the nurse schedules ECT.

A client scheduled for electroconvulsive therapy (ECT) at 9:00 a.m. is discovered eating breakfast at 8:00 a.m. Based on this observation, which is the most appropriate nursing action? A. The nurse notifies the client's physician of the situation and cancels the ECT. B. The nurse removes the breakfast tray and assists the client to the ECT treatment room. C. The nurse allows the client to finish breakfast and reschedules ECT for 10:00 a.m. D. The nurse increases the client's fluid intake to facilitate the digestive process.

ANS: A A client who is scheduled for ECT treatments is given nothing by mouth (NPO) for a minimum of 6 to 8 hours before treatment.

A chronically depressed and suicidal client is admitted to a psychiatric unit. The client is scheduled for electroconvulsive therapy (ECT). During the course of ECT treatments, a nurse should recognize the continued need for which critical intervention? A. Suicide assessment must continue throughout the ECT course of treatment. B. Antidepressant medications are contraindicated throughout the ECT course of treatment. C. Discourage expressions of hopelessness throughout the ECT course of treatment. D. Encourage high-caloric diet throughout the ECT course of treatment.

ANS: A ECT is an intervention for major depression that often includes suicidal ideations as a symptom. Continued suicide assessment is needed because mood improvement due to ECT may cause the client to act on suicidal ideations.

Immediately after an initial electroconvulsive therapy (ECT) treatment a client states, "I'm not hungry and just want to stay in bed and sleep." Based on this information, which is the most appropriate nursing intervention? A. Allow the client to remain in bed. B. Encourage the client to join the milieu to promote socialization. C. Obtain a physician's order for parenteral nutrition. D. Involve the client in physical activities to stimulate circulation.

ANS: A Immediately after electroconvulsive therapy a nurse should monitor pulse, respirations, and blood pressure every 15 minutes for the first hour, during which time the client should remain in bed.

Immediately after electroconvulsive therapy, in which position should a nurse place the client? A. On his or her side to prevent aspiration B. In semi-Fowler's position to promote oxygenation C. In Trendelenburg's position to promote blood flow to vital organs D. In prone position to prevent airway blockage

ANS: A The nurse should place a client who has received electroconvulsive therapy on his or her side to prevent aspiration. After the treatment, most clients will awaken within 10 to 15 minutes and will be confused and disoriented. Some clients will sleep for 1 to 2 hours. All clients require close observation following treatment.

A nurse should recognize that electroconvulsive therapy (ECT) would potentially improve the symptoms of clients with which of the following Axis I diagnoses? (Select all that apply.) A. Major depressive disorder B. Bipolar disorder: manic phase C. Schizoaffective disorder D. Obsessive-compulsive anxiety disorder E. Body dysmorphic disorder

ANS: A, B, C ECT has been shown to be effective in the treatment of severe depression; acute mania; and acute schizophrenia, particularly if it is accompanied by catatonic or affective (depression or mania) symptomatology. ECT has also been tried with other disorders, such as obsessive-compulsive disorder (OCD) and anxiety disorders, but little evidence exists to support its efficacy in the treatment of these conditions

During a course of 12 electroconvulsive therapy (ECT) treatments, an anxious client diagnosed with major depression refuses to bathe or attend group therapy. At this time, which of the following nursing diagnoses should be assigned to this client? (Select all that apply.) A. Anxiety R/T post-ECT confusion and memory loss B. Risk for injury R/T post-ECT confusion and memory loss C. Disturbed thought processes R/T post-ECT confusion and memory loss D. Altered sensory perception R/T post-ECT confusion and memory loss E. Social isolation R/T post-ECT confusion and memory loss

ANS: A, B, C, E Because of the post-ECT thought alterations of confusion and memory loss, the client is anxious, accident prone, and has socially isolated self. Altered sensory perception is related to psychotic thoughts of a sensory nature such as hallucinations, and because this client is diagnosed with major depression, not schizophrenia, altered sensory perception would not be anticipated.

Which of the following conditions would place a client at risk for injury during electroconvulsive therapy (ECT) treatments? (Select all that apply.) A. Severe osteoporosis B. Acute and chronic pulmonary disorders C. Hypothyroidism D. Recent cardiovascular accident E. Prostatic hypertrophy

ANS: A, B, D Severe osteoporosis, acute and chronic pulmonary disorders, and a recent history of cardiovascular accident (CVA) can render clients at high risk for injury during electroconvulsive therapy.

Which assessment test results should a nurse evaluate and report in the process of clearing a client for electroconvulsive therapy (ECT)? (Select all that apply.) A. Electrocardiogram graphic records B. Pulmonary function study results C. Electroencephalogram analysis D. Complete blood count values E. Urinalysis results

ANS: A, B, D, E A nurse should evaluate electrocardiogram graphic records, pulmonary function study results, complete blood count, and urinalysis results and report any abnormalities to the client's physician. The client must be medically cleared prior to ECT treatment.

A nursing student is observing an electroconvulsive therapy (ECT) treatment. The student notices a blood pressure cuff on the client's lower leg. The student questions the instructor about the cuff placement. Which is the most accurate instructor reply? A. "The cuff has to be placed on the leg because both arms are used for intravenous fluids." B. "The cuff functions to prevent succinylcholine from reaching the foot." C. "The cuff position gives a more accurate blood pressure reading during the treatment." D. "The cuff is placed on the leg so that arms can easily be restrained during seizure."

ANS: B A blood pressure cuff is placed on the lower leg and inflated above systolic pressure before injection of succinylcholine. This is to ensure that seizure activity can be observed and timed in this one limb that is unaffected by the paralytic agent.

A nursing instructor is teaching about electroconvulsive therapy (ECT). Which student statement indicates that learning has occurred? A. "During ECT a state of euphoria is induced." B. "ECT induces a grand mal seizure." C. "During ECT a state of catatonia is induced." D. "ECT induces a petit mal seizure."

ANS: B Electroconvulsive therapy is the induction of a grand mal seizure through the application of electrical current to the brain for the purpose of decreasing depression.

A client states, "My doctor has told me I am a candidate for electroconvulsive therapy (ECT). Where will the treatment take place and how much time would this entail?" Which is the most accurate nursing reply? A. "Clients typically receive ECT in their hospital room, daily for 1 month." B. "Clients typically receive 6 to 12 ECT treatments, three times a week in an outpatient setting." C. "Clients typically receive an unlimited number of treatments in the hospital treatment room." D. "Clients typically receive two to three treatments in either an outpatient or inpatient setting."

ANS: B Most clients require an average of 6 to 12 ECT treatments, but some may require up to 20 treatments. Treatments are usually administered every other day, three times per week. Treatments are performed on either an inpatient or outpatient basis depending on the need for client monitoring.

A client is scheduled for an initial treatment of electroconvulsive therapy (ECT). Which information should a nurse include when teaching about the potential side effects of this procedure? A. "You may experience transient tangential thinking." B. "You may experience some memory deficit surrounding the ECT." C. "You may experience avolution for the remainder of the day." D. "You may experience a higher risk for subsequent seizures."

ANS: B The most common side effect of ECT is temporary amnesia following the ECT procedure.

A client with cognitive deficits is extremely suicidal. The client has not responded to antidepressants and the treatment team is considering electroconvulsive therapy (ECT). What client information would determine the feasibility of this treatment option? A. Because the client is extremely suicidal, ECT is an appropriate option. B. Because antidepressant medications have been ineffective, ECT is a good alternative. C. Because informed consent is required for ECT, cognitive deficits preclude this option. D. Because of the client's cognitive deficits, a signed consent is waived.

ANS: C A client who is experiencing cognitive deficits cannot give informed consent that is required prior to ECT treatment. A court proceeding could determine the client's level of competency and, if necessary, appoint a guardian.

A client who is learning about electroconvulsive therapy (ECT) treatment asks a nurse "Isn't this treatment dangerous?" Which is the most appropriate nursing reply? A. "No, this treatment is side-effect free." B. "There can be temporary paralysis but full functioning returns within 3 hours of treatment." C. "There are some risks, but a thorough examination will determine your candidacy for ECT." D. "Transient ischemic attacks (TIA) can occur but are rare."

ANS: C Clients are given medical clearance for ECT. This decreases the risk of injury from the treatment.

A nursing instructor is teaching about the medications given prior to and during electroconvulsive therapy (ECT) treatments. Which student statement indicates that learning has occurred? A. "Atropine (Atro-Pen) is administered to paralyze skeletal muscles during ECT." B. "Succinylcholine chloride (Anectine) decreases secretions to prevent aspiration." C. "Thiopental sodium (Pentothal) is a short-acting anesthesia to render the client unconscious." D. "Glycopyrrolate (Robinul) is given to prevent severe muscle contractions during seizure."

ANS: C In order to render a client unconscious during the ECT procedure, an anesthesiologist administers intravenously, a short-acting anesthetic like thiopental sodium (Pentothal).

After receiving two of nine electroconvulsive therapy (ECT) treatments, a client states, "I can't even remember eating breakfast, so I want to stop the ECT treatments." Which is the most appropriate nursing reply? A. "After you begin the course of treatments, you must complete all of them." B. "You'll need to talk with your doctor about what you're thinking." C. "It is within your right to discontinue the treatments, but let's talk about your concerns." D. "Memory loss is a rare side effect of the treatment. I don't think it should be a concern."

ANS: C The client has the right to terminate treatment. This nursing reply acknowledges this right but focuses on the client's concerns so that the nurse can provide needed information.

A nurse administers pure oxygen to a client during and after electroconvulsive therapy treatment. What is the nurse's rationale for this procedure? A. To prevent increased intracranial pressure resulting from anoxia B. To prevent hypotension, bradycardia, and bradypnea due to electrical stimulation C. To prevent anoxia due to medication-induced paralysis of respiratory muscles D. To prevent blocked airway resulting from seizure activity

ANS: C The nurse administers 100% oxygen during and after electroconvulsive therapy to prevent anoxia due to medication-induced paralysis of respiratory muscles. Electroconvulsive therapy is the induction of a grand mal seizure through the application of electrical current to the brain.

A client experienced bradycardia during electroconvulsive therapy (ECT) treatment. A nurse assigns a nursing diagnosis of decreased cardiac output R/T vagal stimulation occurring during ECT. Which outcome would the nurse expect the client to achieve? A. The client will verbalize an understanding of the need for moving slowly after treatment. B. The client will maintain an oxygen saturation level of 88% 1 hour after treatment. C. The client will continue adequate tissue perfusion 1 hour after treatment. D. The client will verbalize an understanding of common side effects of ECT.

ANS: C Vagal stimulation induced by ECT may cause a client to experience bradycardia. Adequate tissue perfusion would be a realistic expectation when normal cardiac output is restored.

A nurse administers ordered preoperative glycopyrrolate (Robinul) 30 minutes prior to a client's electroconvulsive therapy (ECT) treatment. What is the rationale for administering this medication? A. Robinul decreases anxiety during the ECT procedure. B. Robinul induces an unconscious state to prevent pain during the ECT procedure. C. Robinul prevents severe muscle contractions during the ECT procedure. D. Robinul decreases secretions to prevent aspiration during the ECT procedure.

ANS: D Glycopyrrolate (Robinul) is the standard preoperative medication given prior to ECT treatments to decrease secretions and prevent aspiration.

Cranial Nerve VI

Abducens - motor: muscles that move the eye

Describe nursing and interdisciplinary care for a patient with a spinal cord injury.

Acute phase: Aggressive respiratory therapy. Above C5 injury are intubated/ventilator. Intermittent positive-pressure breathing (IPPB) are used to prevent atelectasis. Foley catheter, surgery/immobilization, tracheotomy if long-term ventilation is needed. Parentreal nutrition and fluids until the GI tract starts functioning. A diet high in protein and fiber. Bowel program during spinal shock: manual disipaction and small-volume enemas. PT and OT therapy: passive ROM and then aggressive rehab long term. Chronic phase: orthostatic hypotension prevention, dietary management (weight gain likely), skin care/turning, respiratory management.

What should you do immediately for someone suspected of subarachnoid hemorrhage?

Administer oxygen, place IV line, do an EKG, once preliminary diagnosis is made: prepare patient for cerebral angiogram to locate site.

What should you make sure to discuss with someone who has multiple sclerosis?

Advanced directives because patient will eventually end up on vent and the use of a peak flow meter.

Advantages

Advantages: • easy to load (PO or IV) • once a day dosing • low cost • in use since 1938 (no surprises)

Full consciousness

Alert; oriented to person, time, place; understands verbal and written words

What is motor aphasia?

Also known as EXPRESSIVE APHASIA, Broca's area, it is the inability to speak or write. However, patient can comprehend the spoken or written form of communication.

What is AMYOTROPHIC LATERAL SCLEROSIS? What is important to know about it?

Also known as Lou Gehrig's Disease, it is a progressive neurological disease characterized by neuron death resulting in muscle weakness and eventually paralysis. The patient will die when respiratory paralysis reaches diaphragm so discuss advance directives and make sure to check gag reflex before feeding this patient.

What is sensory aphasia?

Also known as RECEPTIVE APHASIA, a patient cannot understand oral or written forms of communication.

What is autonomic dysreflexia? What is the cause? What are the signs and symptoms?

Also known as hyperreflexia, it is a potentially life threatening condition involving exaggeration of the sympathetic response to stimulation. The condition occurs in people with spinal cord injuries at T-6 or higher. It is triggered by a sustained stimuli such as restrictive clothing, pressure areas, FULL BLADDER, UTI or FECAL IMPACTION. Signs and symptoms above the level of injury: 1) flushed face 2) increased blood pressure 200/100 3) headache 4) distended neck veins 5) decreased heart rate 6) increased sweating, vasodilation Signs and symptoms below level of injury: 1. Pale 2. Cool 3. No sweating, vasoconstriction

Describe predisposing factors linked to Alzheimer's disease.

Alzheimer's effects cranial nerves, especially #19. patho 1. loss of nerve cells 2. reduce brain size 3. presence of neurofibrillary tangles 4. neuritic plaques by amyloid protein. Aging. One out of eight people over age 65 has Alzheimer's. Nearly half of people over age 85 have the disease. Family history and genetics Another risk factor is family history. Research has shown that those who have a parent, brother or sister with Alzheimer's are two to three times more likely to develop the disease. There appears to be a strong link between serious head injury and future risk of Alzheimer's. It's important to protect your head by buckling your seat belt, wearing your helmet when participating in sports and "fall-proofing" your home. Some evidence suggests that strategies for general healthy aging may also help reduce the risk of developing Alzheimer's. These measures include controlling blood pressure, weight and cholesterol levels; exercising both body and mind; eating a balanced diet; and staying socially active. Scientists don't know yet exactly how Alzheimer's and diabetes are connected, but they do know that excess blood sugar or insulin can harm the brain in several ways: Diabetes raises the risk of heart disease and stroke, which hurt the heart and blood vessels. Damaged blood vessels in the brain may contribute to Alzheimer's disease. The brain depends on many different chemicals, which may be unbalanced by too much insulin. Some of these changes may help trigger Alzheimer's disease. High blood sugar causes inflammation. This may damage brain cells and help Alzheimer's to develop.

What is Guillain Barre Syndrome?

An autoimmune disease of the nervous system due to damage of myelin sheath around the nerves., progresses rapidly or over 2-3 weeks, characterized by muscle weakness or symmetrical paralysis. Pig Head. Also, Landry's paralysis, an acute polyneuropathy affecting the PNS. The most typical symptoms cause change in sensation or pain, as well as dysfunction of the ANS. It can cause complications, in particularly in the respiratory muscles if the ANS is involved. It is usually triggered by an infection.

Explain the surgical treatment for a TIA.

Angioplasty In selected cases, a procedure called carotid angioplasty, or stenting, is an option. This procedure involves using a balloon-like device to open a clogged artery and placing a small wire tube (stent) into the artery to keep it open. If you have a moderately or severely narrowed neck (carotid) artery, your doctor may suggest carotid endarterectomy (end-ahr-tur-EK-tuh-me). This preventive surgery clears carotid arteries of fatty deposits (atherosclerotic plaques) before another TIA or stroke can occur. An incision is made to open the artery, the plaques are removed, and the artery is closed. Carotid endarterectomy is often not done until several months after a TIA, but a large study showed that people benefit most from the surgery if it is done within 2 weeks of a TIA. Delaying surgery longer than 2 weeks increases the risk for stroke, because a person is more likely to have a stroke in the first few days and weeks after a TIA. Each person must carefully weigh the benefits and risks of surgery and compare them with the benefits and risks of using medicine to reduce the risk of TIA or stroke. The success of either treatment will depend on the amount of blockage you have and which medicine you use. Risks of surgery depend on your age, your health status, the skill and experience of the surgeon, and the experience of the medical center where the surgery is done.

6. A female client with a suspected brain tumor is scheduled for computed tomography (CT). What should the nurse do when preparing the client for this test? a. Immobilize the neck before the client is moved onto a stretcher. b. Determine whether the client is allergic to iodine, contrast dyes, or shellfish. c. Place a cap over the client's head. d. Administer a sedative as ordered.

Answer B. Because CT commonly involves use of a contrast agent, the nurse should determine whether the client is allergic to iodine, contrast dyes, or shellfish. Neck immobilization is necessary only if the client has a suspected spinal cord injury. Placing a cap over the client's head may lead to misinterpretation of test results; instead, the hair should be combed smoothly. The physician orders a sedative only if the client can't be expected to remain still during the CT scan.

2. The nurse is teaching a female client with multiple sclerosis. When teaching the client how to reduce fatigue, the nurse should tell the client to: a. take a hot bath. b. rest in an air-conditioned room c. increase the dose of muscle relaxants. d. avoid naps during the day

Answer B. Fatigue is a common symptom in clients with multiple sclerosis. Lowering the body temperature by resting in an air-conditioned room may relieve fatigue; however, extreme cold should be avoided. A hot bath or shower can increase body temperature, producing fatigue. Muscle relaxants, prescribed to reduce spasticity, can cause drowsiness and fatigue. Planning for frequent rest periods and naps can relieve fatigue. Other measures to reduce fatigue in the client with multiple sclerosis include treating depression, using occupational therapy to learn energy conservation techniques, and reducing spasticity.

What population has the highest risk for stroke?

Black males (most risk), white males, black females, then white females (least risk).

1. A white female client is admitted to an acute care facility with a diagnosis of cerebrovascular accident (CVA). Her history reveals bronchial asthma, exogenous obesity, and iron deficiency anemia. Which history finding is a risk factor for CVA? a. Caucasian race b. Female sex c. Obesity d. Bronchial asthma

Answer C. Obesity is a risk factor for CVA. Other risk factors include a history of ischemic episodes, cardiovascular disease, diabetes mellitus, atherosclerosis of the cranial vessels, hypertension, polycythemia, smoking, hypercholesterolemia, oral contraceptive use, emotional stress, family history of CVA, and advancing age. The client's race, sex, and bronchial asthma aren't risk factors for CVA.

3. A male client is having a tonic-clonic seizures. What should the nurse do first? a. Elevate the head of the bed. b. Restrain the client's arms and legs. c. Place a tongue blade in the client's mouth. d. Take measures to prevent injury.

Answer D. Protecting the client from injury is the immediate priority during a seizure. Elevating the head of the bed would have no effect on the client's condition or safety. Restraining the client's arms and legs could cause injury. Placing a tongue blade or other object in the client's mouth could damage the teeth

Discuss the use of anti platelet drugs.

Anti platelet drugs prevent thrombus formation in the arterial system (as opposed to anticoagulants, that prevent thrombosis in the venous system). they work by decreasing the platelet's ability to stick together in the blood, thus forming a clot. Often prescribed prophylactically to pts with a-fib for risk of embolic strokes, but have no other warning signs or indicators of future stroke. Compared with antiplatelet therapy, oral anticoagulation significantly reduces stroke at an average follow-up of one to three years, but does not reduce mortality. Applied to all-comers with atrial fibrillation, aspirin reduces stroke by 20 percent, whereas warfarin (Coumadin) reduces it by 65 percent. But SEVERE Intracranial or extracranial hemorrhage is more common with anticoagulation and must be weighed against its therapeutic benefit.

Describe two classifications of drugs used to treat a TIA.

Anti-platelet drugs. These medications make your platelets, one of the circulating blood cell types, less likely to stick together. When blood vessels are injured, sticky platelets begin to form clots, a process completed by clotting proteins in blood plasma. Anticoagulants. They affect clotting-system proteins instead of platelet function.

Hemianopsia

Blidness or defective vision in half of the visual field in one or both eyes, usually due to stroke, brain tumor, or trauma.

Clonazepam (Klonopin)

Approved for: -Absence (failed ethosuximide) -Myoclonic -Akinetic Side effects: -Sedation, cognitive dysfunction, ataxia

Valproic acid (Depakote, Depacon)

Approved for: -Focal -Absence (long-term) Side effects: -Tremor, weight gain, nausea, sedation, hepatotoxicity, thrombocytopenia, hair loss, pancreatitis, neural tube defects

Levetiracetam (Keppra)

Approved for: -Focal -Generalized tonic-clonic -Juvenile myoclonic epilepsy Side effects: -Somnolence, dizziness, anxiety, psychiatric manifestations

Topiramate (Topamax)

Approved for: -Focal -Generalized tonic-clonic -Lennox-Gastaut Side effects: -Sedation, kidney stones, *mental dulling, weight loss*

Phenytoin (Dilantin)

Approved for: -Focal -Generalized tonic-clonic Side effects: -CYP inducer -Teratogenic (fetal hydantoin syndrome) -Nystagmus, ataxia, diplopia, sedation -SLE-like syndrome ("purple glove syndrome") -Gum hyperplasia -Hirsutism -Megaloblastic anemia, generalized lymphadenopathy

Phenobarbital

Approved for: -Focal -Generalized tonic-clonic Side effects: -Sedation, dizziness, ataxia, osteopenia, behavior disturbance in children

Gabapentin (Neurontin)

Approved for: -Focal Side effects: -Sedation, ataxia, *weight gain, peripheral edema

Pregabalin (Lyrica)

Approved for: -Focal Side effects: -Weight gain, edema, somnolence, dizziness, ataxia, tremor

Carbamazepine (Tegretol)

Approved for: -Focal, generalized tonic-clonic seizures Side effects: -Ataxia, nystagmus, diplopia, hepatotoxicity, rash -*Agranulocytosis/aplastic anemia (BM suppression), teratogenic, CYP inducer, SIADH -> hyponatremia*

Vigabatrin (Sabril)

Approved for: -Infantile spasms Side effects: -Irreversible visual field constriction, headache, somnolence, dizziness, ataxia, weight gain

Discuss assessment methods used to identify changes in patient neurological status?

Ask family what the patient's baseline is Assess LOC, and orientation, if pt is unconcious use the Glascow Coma Scale Assess for numbness and tingling in extremeties Determine if pt has difficulty with sensory functions Assess strength of hands grip and movement of extremities Assess pupils using PEERLA Obtain past medical hx

Who identifies hearing?

Audiologist

Temporal Lobe

Auditory receptive areas, plays a role in memory of sound and understanding language and music.

Define aura; define automatism

Aura = subjective; at onset of seizure but w/o clinical signs of seizure Automatism = coordinated, stereotyped, involuntary motor activity (lip smacking, hand wringing, verbalization of short, stereotyped phrases)

Describe the phases of a tonic-clonic seizure.

Aura: bright light Tonic phase: muscles are rigid with the arms extended and jaws clenched Clonic phase: movements are jerky as the muscles alternately contract and relax Postictal phase: the pt is unconscious for 30 minutes and then regains conciousness slowly

How can someone with a C1-C4 injury maintain some independence?

Blow stick/mouth stick on wheelchair that allows them to control the chair.

The nurse is caring for a patient admitted with a subdural hematoma following a motor vehicle accident. Which of the following changes in vital signs would the nurse interpret as a manifestation of increased intracranial pressure? A) Tachypnea B) Bradycardia C) Hypotension D) Narrowing pulse pressure

B) Bradycardia Changes in vital signs indicative of increased intracranial pressure are known as Cushing's triad, which consists of increasing systolic pressure with a widening pulse pressure, bradycardia with a full and bounding pulse, and irregular respirations.

The nurse is providing care for a patient who has been admitted to the hospital with a head injury and who requires regular neurologic vital signs. Which of the following assessments will be components of the patient's score on the Glasgow Coma Scale (GCS) (select all that apply)? A) Judgment B) Eye opening C) Abstract reasoning D) Best verbal response E) Best motor response F) Cranial nerve function

B, D, E The three dimensions of the GCS are eye opening, best verbal response, and best motor response.

Patency of the nostrils is assessed by having the individual: A. breathe through both nares rapidly B. sniff inward through one nairs while the other is occluded C. blow of through each naris while the other is occluded D. observe each naris for flaring during each inhalation and exhalation

B. sniff inward through one nairs while the other is occluded

How does GBS progress?

Begins in lower extremities and ascends bilaterally starting with weakness, then ataxia, then bilateral paresthesia progressing in paralysis.

What is BELLS PALSY? What are the considerations?

Bell's palsy is a form of facial paralysis resulting from a dysfunction of the cranial nerve VII (the facial nerve) causing an inability to control facial muscles on the affected side. The biggest complications are pain and an eyelid that won't shut. To protect the eyes from corneal abrasions, use drops and eye patches at night or tape shut to protect.

Benign childhood epilepsy w/centrotemporal spikes

Benign "rolandic" epilepsy -Simple partial seizures with abnormal movements and sensation of the mouth, sometimes the hand, with excessive salivation and drooling -May have secondary generalization -Typically present at age 5-10 yrs, almost always resolve by age 18

Glasgow Coma Scale (GCS) categories

Best response for each: -Eye (4 points max) -Verbal (5 points max) -Motor (6 points max)

Explain the use of emergency equipment for patients with a seizure disorder.

Bite stick Suction O2

Define "pill rolling" and "bradykinesia".

Bradykinesia: since the extrapyramidal system regulates posture and skeletal muscle tone, a result is the characteristic of bradykinesia of Parkinson's. It is a slowness of movement. Slowness in the execution of movement, not initiation (like akinesia). "Stone face". Pill Rolling: The Parkinson's tremor tends to more often affect the hands and causes a movement sometimes referred to as "pill rolling". This "pill rolling" 'tremor' involves the uncontrolled movement of the thumb and finger(s) in a back and forth motion. This may also appear as the thumb and fingers are rubbing together, hence the term "pill rolling" movement. These tremors are usually rhythmic and may occur between 4 to 5 cycles per second. It may only affect one side of the body, or one hand, but as the disease progresses, the tremor may become more generalized affecting many parts of the body.

Epilepsy

Brain disorder characterized by enduring predisposition to generate seizures + actual occurrence of at least one seizure

Deep brain stimulation

Brain pacemaker. Delivers shocks to the brain and helps stop signals that cause PD

If sound lateralizes to one ear when performing the Weber test, which of the following occurring? A. sound is heard LONGER in one ear than the other. B. higher FREQUENCIES of sound are heard better in one ear C. sound is heard LOUDER in one ear D. electrical impulses are amplified in one ear

C. sound is heard LOUDER in one ear

A consensual light reflex is present when which of the following occurs? A. the right pupil dilates when a light is shone on the left pupil B. the left pupil dilates immediately after the light is removed from the left pupil C. the right pupil constricts when a light is shone into the left pupil D. the left pupil constricts after the light is removed from the right pupil

C. the right pupil constricts when a light is shone into the left pupil

Which of the following is a true statement about the paranasal sinuses? A. they're fully developed in newborns and shrink in size during puberty relative to the growth of the skill of the bones B. they're totally absent in newborn and rapidly develop during puberty C. they're not all present in the newborn but grow and develop with the child, reaching full development after puberty.

C. they're not all present in the newborn but grow and develop with the child, reaching full development after puberty

Before inserting an otoscope in an adult client, which of the following maneuvers should the examiner perform A. irrigate the ear canal to make sure the tympanic membrane is visible B. have the client blow his or her nose to make sure pressure is equalized between the external ear and the middle ear. C. pull the pinna up and back to straighten the auditory canal D. tilt the client's head toward the side being examined

C. pull the pinna up and back to straighten the auditory canal

A post-stroke patient is going home on oral Coumadin (warfarin). During discharge teaching, which statement by the patient reflects an understanding of the effects of this medication? 1. "I will stop taking this medicine if I notice any bruising." 2. "I will not eat spinach while I'm taking this medicine." 3. "It will be OK for me to eat anything, as long as it is low fat." 4. "I'll check my blood pressure frequently while taking this medication."

Correct Answer: 2 Rationale: Warfarin is a vitamin K antagonist. Green, leafy vegetables contain vitamin K, and will therefore interfere with the therapeutic effects of the drug. Bruising is a common side effect, and the drug should not be stopped unless by prescriber order. Low-fat foods do not interfere with warfarin therapy, which is not prescribed to affect the blood pressure.

The healthcare provider orders 2.5 mg IV of morphine sulfate (Morphine) to be administered to a patient with a ruptured interverterbral disk. The nurse has a 1 milliliter (mL) syringe containing 10 mg of morphine sulfate. How many milliliters of morphine sulfate does the nurse need to withdraw from the syringe?

Correct Answer: 0.25

Which patient is at highest risk for a spinal cord injury? 1. 18-year-old male with a prior arrest for driving while intoxicated (DWI) 2. 20-year-old female with a history of substance abuse 3. 50-year-old female with osteoporosis 4. 35-year-old male who coaches a soccer team

Correct Answer: 1 Rationale: The three major risk factors for spinal cord injuries (SCI) are age (young adults), gender (higher incidence in males), and alcohol or drug abuse. Females tend to engage in less risk-taking behavior than young men.

A patient with a spinal cord injury is recovering from spinal shock. The nurse realizes that the patient should not develop a full bladder because what emergency condition can occur if it is not corrected quickly? 1. autonomic dysreflexia 2. autonomic crisis 3. autonomic shutdown 4. autonomic failure

Correct Answer: 1 Rationale: Be attuned to the prevention of a distended bladder when caring for spinal cord injury (SCI) patients in order to prevent this chain of events that lead to autonomic dysreflexia. Track urinary output carefully. Routine use of bladder scanning can help prevent the occurrence. Other causes of autonomic dysreflexia are impacted stool and skin pressure. Autonomic crisis, autonomic shutdown, and autonomic failure are not terms used to describe common complications of spinal injury associated with bladder distension.

The nurse must be alert to complications in the patient who has suffered a ruptured intracranial aneurysm. The nurse should assess the patient for signs of which of the following? Select all that apply. 1. headache 2. hydrocephalus 3. rebleeding 4. vasospasm 5. stiff neck

Correct Answer: 2,3,4 Rationale: Headache is a sign of a probable rebleed. Hydrocephalus, rebleeding, and vasospasm are the three major complications that a nurse must anticipate following a ruptured intracranial aneurysm. Stiff neck is a manifestation of intracranial aneurysm, not a complication.

The nurse recognizes that the most common type of brain attack (CVA) is related to which of the following? 1. ischemia 2. hemorrhage 3. headache 4. vomiting

Correct Answer: 1 Rationale: Eighty percent of all strokes are caused by ischemia. Hemorrhagic strokes are less common than ischemic strokes. Headache and vomiting may be symptoms associated with CVA, but not common causes.

A patient has the nursing diagnosis of Impaired Swallowing and complains of frequent heartburn. What is the most appropriate action by the nurse? 1. Assist the patient in maintaining a sitting position for 30 minutes after the meal. 2. Teach the patient the "chin tuck" technique when swallowing. 3. Check the patient's mouth for pocketing of food. 4. Assist the patient to a 90-degree sitting position, or as high as tolerated, during meals.

Correct Answer: 1 Rationale: Keeping the patient upright for a time after the meal will help prevent food from being regurgitated back into the esophagus. The position of the patient during the meals as well as teaching the "chin tuck" technique will assist with the swallowing mechanism, but will not help with regurgitation. Pocketing food does not cause regurgitation.

A patient whose status is post-stroke (CVA) has severe right-sided weakness. Physical therapy recommends a quad cane. Which of the following is proper use of the cane by the patient? 1. The patient holds the cane in the left hand. The patient moves the cane forward first, then the right leg, and then the left leg. 2. The cane is held in either hand and moved forward at the same time as the left leg. Then the patient drags the right leg forward. 3. The patient holds the cane in the right hand for support. The patient moves the cane forward first, then the left leg, and then the right leg. 4. The patient holds the cane in the left hand. The patient moves the left leg forward first, then moves the cane and the right leg forward together.

Correct Answer: 1 Rationale: Proper use of the cane is essential to fall prevention. The patient should hold the cane in the left hand. The patient should move the cane forward first, then the right leg, and then the left leg.

A patient is recovering following a carotid endarterectomy. The blood pressure has risen this morning to 168/60. The nurse should do which of the following? 1. Recheck the blood pressure and make sure the correct size cuff was used. Then compare the trend of blood pressure readings and call the physician now. 2. Recheck the blood pressure every hour and report this change to the physician when he or she makes rounds the next time. 3. Record the blood pressure and find out who took this reading. Have that staff member demonstrate his or her blood pressure procedure and offer tips to obtain more accurate readings. 4. Check the standing orders and see if there is a medication ordered p.r.n. for lowering blood pressure. If so, administer it and document the action.

Correct Answer: 1 Rationale: Take a blood pressure reading manually to check technique, compare the results to the last several blood pressures recorded, and call the physician to report this blood pressure. Physicians typically have a range for maintaining the blood pressure following carotid endarterectomy, with standing orders for higher or lower blood pressures. If the blood pressure becomes higher, it is a danger and should be reported to the physician and documented in the patient record along with orders received. Although the skill of the staff is important, it is a priority to notify the physician of the blood pressure reading so that treatment can begin. Antihypertensives may be ordered and administered p.r.n., but physician notification after verification of the reading is the priority, so that further evaluation can occur.

When caring for a patient admitted post-stroke (CVA) who has altered consciousness, the nurse should place the patient in which position? 1. side-lying 2. supine 3. prone 4. semi-Fowler's

Correct Answer: 1 Rationale: The side-lying position is the safest position to allow adequate drainage of fluids without aspiration.

Prodromal manifestations prior to an intracranial aneurysm rupture could be recognized by the nurse as which of the following? Select all that apply. 1. visual deficits 2. headache 3. mild nausea 4. dilated pupil 5. stiff neck

Correct Answer: 1,2,4 Rationale: Often intracranial aneurysms are asymptomatic until rupture but patients can complain of headache and eye pain, and have visual deficits and a dilated pupil. Nausea and vomiting and stiff neck are not usually associated with the prodromal manifestations of an intracranial aneurysm, but may occur with leaking or rupture.

An unconscious patient receiving emergency care following an automobile crash accident has a possible spinal cord injury. What guidelines for emergency care will be followed? Select all that apply. 1. Immobilize the neck using rolled towels or a cervical collar. 2. The patient will be placed in a supine position 3. The patient will be placed on a ventilator. 4. The head of the bed will be elevated. 5. The patient's head will be secured with a belt or tape secured to the stretcher.

Correct Answer: 1,2,5 Rationale: In the emergency setting, all patients who have sustained a trauma to the head or spine, or are unconscious should be treated as though they have a spinal cord injury. Immobilizing the neck, maintaining a supine position and securing the patient's head to prevent movement are all basic guidelines of emergency care. Placement on the ventilator and raising the head of the bed will be considered after admittance to the hospital.

A patient was diagnosed with a left cerebral hemorrhage. Which topics are most appropriate for the nurse to include in patient and family teaching? Select all that apply. 1. how to use a sign board 2. transfer techniques 3. information about impulse control 4. time adjustment to complete activities 5. safety precautions for transferring

Correct Answer: 1,2,5 Rationale: The left cerebral hemisphere is responsible for the language center, calculation skills, and thinking/reasoning abilities. Reading and speaking could be compromised if there is left-sided brain damage. The patient also might display overcautious behavior and might be slow to respond or complete activities. Transfer techniques would apply regardless of the side involved. Impulse control problems can arise with right-sided involvement.

A patient with a spinal cord injury was given IV Decadron (dexamethasone) after arriving in the emergency department. The patient also has a history of hypoglycemia. During the hospital stay, the nurse would expect to see which of the following? 1. increased episodes of hypoglycemia 2. possible episodes of hyperglycemia 3. no change in the patient's glycemic parameters 4. both hyper- and hypoglycemic episodes

Correct Answer: 2 Rationale: A common side effect of corticosteroids is hyperglycemia. Stress as well as the medication could cause this person to have periods of elevated blood sugars.

The nurse is educating a patient and the family about different types of stabilization devices. Which statement by the patient indicates that the patient understands the benefit of using a halo fixation device instead of Gardner-Wells tongs? 1. "I will have less pain if I use the halo device." 2. "The halo device will allow me to get out of bed." 3. "I am less likely to get an infection with the halo device." 4. "The halo device does not have to stay in place as long."

Correct Answer: 2 Rationale: A halo device will allow the patient to be mobile since it does not require weights like the Gardner-Wells tongs. The patient's pain level is not dependant on the type of stabilization device used. The patient does not have a great risk of infection with the Garnder-Wells tongs; both devices require pins to be inserted into the skull. The time required for stabilization is not dependant on the type of stabilization device used.

A patient with a spinal cord injury at the T1 level complains of a severe headache and an "anxious feeling." Which is the most appropriate initial reaction by the nurse? 1. Try to calm the patient and make the environment soothing. 2. Assess for a full bladder. 3. Notify the healthcare provider. 4. Prepare the patient for diagnostic radiography.

Correct Answer: 2 Rationale: Autonomic dysreflexia occurs in patients with injury at level T6 or higher, and is a life-threatening situation that will require immediate intervention or the patient will die. The most common cause is an overextended bladder or bowel. Symptoms include hypertension, headache, diaphoresis, bradycardia, visual changes, anxiety, and nausea. A calm, soothing environment is fine, though not what the patient needs in this case. The nurse should recognize this as an emergency and proceed accordingly. Once the assessment has been completed, the findings will need to be communicated to the healthcare provider.

A lumbar puncture (LP) is done on a patient to rule out a spinal cord tumor. The cerebrospinal fluid (CSF) is xanthochromic, has increased protein, no cells, and clots immediately. What syndrome do these findings describe? 1. Glasgow's syndrome 2. Froin's syndrome 3. cord tumor syndrome 4. reflex syndrome

Correct Answer: 2 Rationale: Froin's syndrome is seen with spinal cord tumors. A lumbar puncture, x-rays, CT scans, MRI, and myelogram are all common tests that are used to diagnose a spinal cord tumor. Glasgow's syndrome, cord tumor syndrome, and reflex syndrome are not terms associated with the symptoms of spinal cord tumor described.

The patient is admitted with injuries that were sustained in a fall. During the nurse's first assessment upon admission, the findings are: blood pressure 90/60 (as compared to 136/66 in the emergency department), flaccid paralysis on the right, absent bowel sounds, zero urine output, and palpation of a distended bladder. These signs are consistent with which of the following? 1. paralysis 2. spinal shock 3. high cervical injury 4. temporary hypovolemia

Correct Answer: 2 Rationale: Spinal shock is common in acute spinal cord injuries. In addition to the signs and symptoms mentioned, the additional sign of absence of the cremasteric reflex is associated with spinal shock. Lack of respiratory effort is generally associated with high cervical injury. The findings describe paralysis that would be associated with spinal shock in an spinal injured patient. The likely cause of these findings is not hypovolemia, but rather spinal shock.

Which of the following is the priority nursing diagnosis for the patient who has undergone surgery for a spinal fusion? 1. Acute Pain 2. Impaired Mobility 3. Risk for Infection 4. Risk for Injury

Correct Answer: 2 Rationale: The priority nursing diagnosis for a patient who has undergone a spinal fusion is Impaired Mobility, due to the assessment of the ABCs (airway, circulation, breathing). Impaired mobility can affect the patient's circulation, therefore affecting tissue perfusion and causing a risk for skin breakdown. Acute Pain is the next priority since it is an active diagnosis. Diagnoses with "risk for" do not take priority over active diagnoses.

The family of a patient who has had a brain attack (CVA) asks if the patient will ever talk again. The nurse should do which of the following? 1. Explain that the patient's speech will return to normal with time. 2. Explain that it is difficult to know how far the patient will progress. 3. Tell the family that nurses cannot discuss such issues. Tell them to ask the physician. 4. Tell the family what they see today is all they can expect.

Correct Answer: 2 Rationale: Therapeutic communication is needed. It is important to allow hope but be honest by not promising progress, since no one knows how much the patient will improve. Progress may depend on the extent and the areas affected. The nurse does not know that speech will return in time. It is not therapeutic to tell the family to ask the physician, and it does not display a professional, caring attitude.

The nurse understands that when the spinal cord is injured, ischemia results and edema occurs. How should the nurse explain to the patient the reason that the extent of injury cannot be determined for several days to a week? 1. "Tissue repair does not begin for 72 hours." 2. "The edema extends the level of injury for two cord segments above and below the affected level." 3. "Neurons need time to regenerate so stating the injury early is not predictive of how the patient progresses." 4. "Necrosis of gray and white matter does not occur until days after the injury."

Correct Answer: 2 Rationale: Within 24 hours necrosis of both gray and white matter begins if ischemia has been prolonged and the function of nerves passing through the injured area is lost. Because the edema extends above and below the area affected, the extent of injury cannot be determined until after the edema is controlled. Neurons do not regenerate, and the edema is the factor that limits the ability to predict extent of injury.

A patient has manifestations of autonomic dysreflexia. Which of these assessments would indicate a possible cause for this condition? Select all that apply. 1. hypertension 2. kinked catheter tubing 3. respiratory wheezes and stridor 4. diarrhea 5. fecal impaction

Correct Answer: 2,5 Rationale: Autonomic dysreflexia can be caused by kinked catheter tubing allowing the bladder to become full, triggering massive vasoconstriction below the injury site, producing the manifestations of this process. Acute symptoms of autonomic dysreflexia, including a sustained elevated blood pressure, may indicate fecal impaction. The other answers will not cause autonomic dysreflexia.

A patient hospitalized with a known AV malformation begins to complain of a headache and becomes disorientated. Which is the most appropriate action by the nurse? 1. Recommend to the family members that they start to look for a long-term care facility. 2. Prepare to give aspirin or a "clot buster." 3. Prepare the patient for surgery. 4. Document the changes and monitor closely.

Correct Answer: 3 Rationale: An AV malformation is a cluster of vessels, usually located in the midline cerebral artery, that, if ruptured, becomes a surgical emergency to cut the blood flow to the vessels or the patient will bleed out into the brain. Symptoms of rupture include headache,,change in level of consciousness,, nausea and vomiting, and neurological deficits symptoms that mimic any brain bleed. Giving medication to affect coagulation will only make the bleeding worse. Recommending long-term care and merely documenting the changes are not appropriate interventions for a medical emergency.

While caring for the patient with spinal cord injury (SCI), the nurse elevates the head of the bed, removes compression stockings, and continues to assess vital signs every two to three minutes while searching for the cause in order to prevent loss of consciousness or death. By practicing these interventions, the nurse is avoiding the most dangerous complication of autonomic dysreflexia, which is which of the following? 1. hypoxia 2. bradycardia 3. elevated blood pressure 4. tachycardia

Correct Answer: 3 Rationale: Autonomic dysreflexia is an emergency that requires immediate assessment and intervention to prevent complications of extremely high blood pressure. Additional nursing assistance will be needed and a colleague needs to reach the physician stat.

A patient with a spinal cord injury (SCI) has complete paralysis of the upper extremities and complete paralysis of the lower part of the body. The nurse should use which medical term to adequately describe this in documentation? 1. hemiplegia 2. paresthesia 3. paraplegia 4. quadriplegia

Correct Answer: 4 Rationale: Quadriplegia describes complete paralysis of the upper extremities and complete paralysis of the lower part of the body. Hemiplegia describes paralysis on one side of the body. Paresthesia does not indicate paralysis. Paraplegia is paralysis of the lower body.

A school nurse is called after a student falls down a flight of stairs. The student is breathing, but unconsciousness. After calling the ambulance, which is the most appropriate action by the nurse? 1. Protect the patient's neck and head from any movement. 2. Place the patient on his side to prevent aspiration. 3. Immobilize the neck,,securing the head. 4. Try to rouse the patient by gently shaking his shoulders.

Correct Answer: 3 Rationale: Guidelines for emergency care are avoiding flexing, extending, or rotating the neck; immobilizing the neck; securing the head; maintaining the patient in the supine position; and transferring from the stretcher with backboard in place to the hospital bed. This patient is unconscious, and the nurse must protect the neck from any (or any further) damage. If the patient vomits, the nurse should utilize the log-roll technique to turn the patient while keeping the head, neck, and spine in alignment. Rousing the patient by shaking could cause damage to the spinal cord.

A hospitalized patient has become unresponsive. The left side of the body is flaccid. The attending physician believes the patient may have had a stroke (CVA). What is the nurse's priority intervention? 1. Move the patient to the critical care unit. 2. Assess blood pressure. 3. Assess the airway and breathing. 4. Observe urinary output.

Correct Answer: 3 Rationale: In any unconscious patient, the airway must be protected. Assessment of the current airway and breathing status is of highest priority and will continue to be. Blood pressure and output monitoring as well as ensuring appropriate level of care are important interventions, but assessment of the patient's ability to maintain an airway is the most vital.

Which of the following is the priority nursing diagnosis for a patient diagnosed with a spinal cord injury? 1. Fluid Volume Deficit 2. Impaired Physical Mobility 3. Ineffective Airway Clearance 4. Altered Tissue Perfusion

Correct Answer: 3 Rationale: Ineffective Airway Clearance is the priority nursing diagnosis for this patient. The nurse utilizes the ABCs (airway, breathing, circulation) to determine priority. With Ineffective Airway Clearance, the patient is at risk for aspiration and therefore, impaired gas exchange. Fluid Volume Deficit is the nurse's next priority (circulation), and then Altered Tissue Perfusion. If the patient does not have enough volume to circulate, then tissue perfusion cannot be adequately addressed. The last priority for this patient is Impaired Physical Mobility.

The nurse is caring for a patient with increased intracranial pressure (IICP). The nurse realizes that some nursing actions are contraindicated with IICP. Which nursing action should be avoided? 1. Reposition the patient every two hours. 2. Position the patient with the head elevated 30 degrees. 3. Suction the airway every two hours per standing orders. 4. Provide continuous oxygen as ordered.

Correct Answer: 3 Rationale: Suctioning further increases intracranial pressure; therefore, suctioning should be done to maintain a patent airway but not as a matter of routine. Maintaining patient comfort by frequent repositioning as well as keeping the head elevated 30 degrees will help to prevent (or even reduce) IICP. Keeping the patient properly oxygenated may also help to control ICP.

The nurse realizes that the goal of surgery for a patient with a secondary metastatic spinal cord tumor is 1. complete removal of the tumor and affected spinal cord tissue. 2. eradication of the tumor with excision and drainage. 3. tumor excision to reduce cord compression. 4. exploration to visualize the tumor and obtain a biopsy.

Correct Answer: 3 Rationale: The tumor can exert pressure on the spinal cord, which interferes with function. In the case of secondary metastatic spinal tumor (which means a second site of cancer) and the metastasis (spread of cancer) the patient outcome may be limited to preventing compression on the spinal cord and not totally removing the cancerous lesion. Complete removal along with affected spinal tissue or eradication by excision and drainage would not be likely due to the secondary nature of the tumor and the resulting disability. Biopsy can be accomplished without direct visualization.

A nurse is teaching a wellness class and is covering the warning signs of stroke. A patient asks, "What is the most important thing for me to remember?" Which is an appropriate response by the nurse? 1. "Know your family history." 2. "Keep a list of your medications." 3. "Be alert for sudden weakness or numbness." 4. "Call 911 if you notice a gradual onset of paralysis or confusion."

Correct Answer: 3 Rationale: Warning signs of stroke include sudden weakness, paralysis, loss of speech, confusion, dizziness, unsteadiness, and loss of balance the key word is sudden. Family history and past medical history can be indicators for risk, but they are not warning signs of stroke. Gradual onset of symptoms is not indicative of a stroke.

Of the following, which groups are the most at risk for bacterial meningitis? Select all that apply. 1. older adults 2. pregnant women 3. military recruits 4. college students 5. low-income

Correct Answer: 3,4 Rationale: Military personnel living on a base and young adults living in close proximity (such as college students living in a dormitory) are at a greater risk of contracting bacterial meningitis. The other populations are at lower risk.

A patient is placed in ventilator support with the diagnosis of botulism and failure to thrive. Which nursing actions would be most appropriate for this patient? Select all that apply. 1. maintaining intravenous fluids at KVO (keep vein open) 2. assessing bowel sounds once a shift 3. referring the patient for a physical therapy consult 4. recording the patient's ongoing calorie count 5. assessing the patient's urinary output every hour

Correct Answer: 3,4,5 Rationale: Maintaining fluids at KVO is inappropriate since this patient will be placed on NPO (nothing by mouth) status while ventilated. It is important that the patient receive adequate fluids for hydration and nutrition since nothing will be consumed by mouth. The patient's bowel sounds need to be assessed more often than once a shift (every one to two hours while in the ICU) since the patient is at risk for a paralytic ileus. Physical therapy will be beneficial for maintaining ROM (range of motion) while the patient is immobile from ventilation and sedation. The nurse must closely monitor the patient's calorie intake to determine nutritional needs while NPO. Any time a patient is on maintenance intravenous fluids urinary output must be monitored closely. Additionally, this particular patient is at risk for urinary retention.

The nurse is teaching regarding risk factors for stroke (CVA). The greatest risk factor is which of the following? 1. diabetes 2. heart disease 3. renal insufficiency 4. hypertension

Correct Answer: 4 Rationale: Hypertension is the greatest risk factor for stroke, and should be controlled. Diabetes, heart disease, and renal insufficiency can all lead to stroke, however hypertension is the greatest risk.

A hospitalized patient with a C7 cord injury begins to yell "I can't feel my legs anymore." Which is the most appropriate action by the nurse? 1. Remind the patient of her injury and try to comfort her. 2. Call the healthcare provider and get an order for radiologic evaluation. 3. Prepare the patient for surgery, as her condition is worsening. 4. Explain to the patient that this could be a common, temporary problem.

Correct Answer: 4 Rationale: Spinal shock is a condition almost half the people with acute spinal injury experience. It is characterized by a temporary loss of reflex function below level of injury, and includes the following symptomatology: flaccid paralysis of skeletal muscles, loss of sensation below the injury, and possibly bowel and bladder dysfunction and loss of ability to perspire below the injury level. In this case, the nurse should explain to the patient what is happening.

When comparing air conduction vs. bone conduction, which is expected to occur? A. bone conduction is normally 2 times as long as air conduction B. bone conduction and air conduction are equal C. air conduction is normally 3 times as long as bone conduction D. air conduction is normally 2 times as long as bone conduction

D. air conduction is normally 2 times as long as bone conduction

Discuss the nursing implications for medications ordered for patients with a spinal cord injury.

Corticosteroids: reduces damage and improves functional recovery by protecting the neuromembrane from further destruction. Monitor for increased infection rate, hyperglycemia, GI bleeding. May also use osmotic diuretics, analgesics, antacids, anticoagulants, stool softeners, vasopressors. Histamine H2-receptor antagonists (ranitidine) are used to prevent stress-related gastric ulcers. Antispasmodics: baclofen, diazepam, dantrolene. they are used to control muscle spasm and pain associated with acute or chronic musculoskeletal conditions. they are not always effective in controlling spasticity resulting from cerebral or spinal cord conditions. assess the client's spasticity and involuntary movements. give with food to decrease GI symptoms. monitor for drowsiness and dizziness.

When examining the set of the ear, the top of the pinna should match an imaginary line drawn from the A. tip of the lateral point of the eyebrow to the occiput B. tip of the nose to the crown of the head. C. tip of the corner of the mouth to the crown of the head D. corner of the eye to the occiput

D. corner of the eye to the occiput

The nurse assesses a patient for signs of meningeal irritation and observes her for nuchal rigidity. Which of the following indicates the presence of this sign of meningeal irritation? A) Tonic spasms of the legs B) Curling in a fetal position C) Arching of the neck and back D) Resistance to flexion of the neck

D) Resistance of flexion of the neck Nuchal rigidity is a clinical manifestation of meningitis. During assessment, the patient will resist passive flexion of the neck by the health care provider.

Differentiate decorticate posturing; decerebrate posturing, and flaccid response.

Decorticate posture: an abnormal posturing in which a person is stiff with bent arms, clenched fists, and legs held out straight. The arms are bent in toward the body and the wrists and fingers are bent and held on the chest. This type of posturing is a sign of severe damage in the brain Decerebrate posture: an abnormal body posture that involves the arms and legs being held straight out, the toes being pointed downward, and the head and neck being arched backwards. The muscles are tightened and held rigidly. This type of posturing usually means there has been severe damage to the brain. Flaccid response: quality of lack of tone of muscular or vascular organ or tissue.

Somnolent

Extreme drowsiness but will respond to stimuli

How many cervical spinal nerves are there? What do they control?

Eat breakfast at 8 in the morning = 8 spinal nerves Controls: diaphragm C1-4, chest, wall muscles, arms and shoulders.

How many lumbar and sacral spinal nerves are there? What do they control?

Eat dinner at 5 in the afternoon = 5 lumbar spinal nerves and 5 sacral spinal nerves. Controls: lower body, bowel and bladder

How many thoracic spinal nerves are there? What do they control?

Eat lunch at noon = 12 thoracic spinal nerves Controls: upper body and gastrointestinal function

Describe the adaptive equipment available for patient care.

Eating Devices • Nonskid mats to stabilize plates • Plate guards to prevent food from being pushed off plate • Wide-grip utensils to accommodate a weak grasp Bathing and Grooming Devices • Long-handled bath sponge • Grab bars, nonskid mats, handheld shower heads • Electric razors with head at 90 degrees to handle • Shower and tub seats, stationary or on wheels Toileting Aids • Raised toilet seat • Grab bars next to toilet Dressing Aids • Velcro closures • Elastic shoelaces • Long-handled shoe horn Mobility Aids • Canes, walkers, wheelchairs • Transfer devices such as transfer boards and belts

What is characteristic of a Stage II pressure ulcer?

Epidermis is broken, lesion is superficial and there is partial thickness skin loss, blister

Discuss manifestations of autonomic dysreflexia and nursing care to prevent or relieve symptoms.

Exaggerated unopposed autonomic response to noxious stimuli for individuals with SCI at or above T6 (as low as T8). Nursing Interventions: bowel/skin care regimen, flushing catheter daily, monitor for distention, I&O, monitor VS for indicators of AD such as hypertension, pounding HA, bradycardia, blurred vision, nausea, nasal congestion, flushing and sweating above the level of injury. If AD is suspected, raise head 90 degrees to lower BP. Monitor BP q3-5min during hypertensive episode. Assess for the cause, implement measures for removing the noxious stimulus. Could be: Bladder distension, bowel constipation/impaction, skin problems (pressure, infection, injury, heat, pain, cold).

West syndrome

Generalized seizure disorder of infants -Recurrent spasms, EEG pattern of hypsarrhythmia, retardation -Associated with tuberous sclerosis -Tx: ACTH

Discuss components of a nutritional plan for a patient with Alzheimer's disease.

Ginko Biloba seems to improve memory. Antioxidants such as Vit-C, Vit-E, and coenzyme 10 may slow progression. Huperzine A, a traditional Chinese medicine, acts as an acetylcholinesterase inhibitor, encourage fluids and fiber.

What kind of diet should someone with a post CVA be on?

High protein and high Vitamin C, e.g. custard and strawberries, to assist with pressure ulcer prevention or healing.

What should a diet for someone with a pressure ulcer consist of?

Increased protein and vitamin C:strawberries, custard, orange juice and tomato juice

Lamotragine

Initially, led to big rash (starting pt on it too quickly) but if start pt on it slowly, less rash; careful of DDIs; Very effective

Describe seven signs and symptoms of increased intracranial pressure.

LOC: EARLY IICP: restleness, irritability, LATE IICP: coma, no response to stimuli Pupils: EARLY IICP: equal round and reactive to light. LATE IICP: sluggish response, progressing to fixed response, pupils may dilate only on one side. Vision: EARLY IICP: decreased visual acuity, blurred vision. LATE IICP: unable to assess Motor Function: EARLY IICP: weakness in on extremity or side. LATE IICP: decorticate or decebrate posturing. Speech: EARLY IICP: difficulty speaking. LATE IICP: cannot assess due to decrease in LOC. Blood Pressure: EARLY IICP: elevated blood pressure. LATE IICP: Cushing's Triad, increased systolic BP, wideining pulse pressure, bradycardia Pulse: EARLY IICP: slighty elevated. LATE IICP: widening pulse. Respiration: EARLY IICP: rate may increase. LATE IICP: decreased respiratory rate or cheyne-stokes breathing. Temperature: EARLY IICP: may be decreased or increased. LATE IICP: significantly elevated. Other sx: EARLY IICP: headaches worse on rising in the morning and with position changes, LATE IICP: Continual headache, projectile vomitiing. Loss of pupil, corneal, gag, and swallowing reflexes.

What are the goals of treatment for a subarachnoid hemorrhage?

Locate site of bleeding and repair the damage. Also manage complications (hydrocephalus, hyponatremia, seizures, and increased ICP)

Discuss nursing interventions for a patient having a seizure.

Loosening of clothing around the neck Turn client to side Suction at bedside O2 as ordered Record symptoms during seizure Pad side rails Bed in low position Fall pads on floor

Formula for MAP (mean arterial pressure)

MAP = [(2 x diastolic) + systolic] / 3

bromocriptine (Parlodel) & pramipezole (Mirapex)

Medication that activate release of dopamine. Work in conjunction with dopaminergics for better results. Monitor orthostatic hypertension, dyskinesia, and hallucinations.

COMT- entacapone (Comtan)

Medication that decreases breakdown of levadopa making more available to brain for dopamine. Monitor for dyskinesia, hyperkinesia, diarrhea. Dark urine is a normal finding.

Anticholenergics- benzotropine (cogentin) & artane

Medication that helps control tremors and rigidity. Monitor for signs of dry mouth, constipation, and urinary retention

Carbidopa

Medication that mimics dopamine action, increases efficacy of dopamine

Antivirals

Medications that stimulate release of dopamine and prevent re-uptake. Monitor for signs of swollen ankles and discoloration of the skin.

Compare and contrast four inflammatory neurological conditions.

Meningitis: inflammation of the meninges of the brain and spinal cord. Enchephalitis: an acute inflammation of the white and gray matter of the brain. Brain abscess: collection of purulent material within the brain.

Is periorbital edema present with glaucoma?

NO

Stage 3

Mild cognitive decline; problems w/memory or concentration may be measurable in clinical testing or during a detailed medical interview; mild cognitive deficit, including losing or misplacing important objects; decreased ability to plan; short-term memory loss noticeable to close relatives; decreased attention span; difficulty remembering words or names; difficulty in social or work situations

Stage 4

Moderate cognitive decline; mild or early-stage AD; medical interview will detect clear-cut decline; personality changes - appearing withdrawn or subdued, especially in social or mentally challenging situations; obvious memory loss; limited knowledge & memory of recent occasions, current events, or personal history

Stage 5

Moderately severe cognitive decline; moderate or mid-stage AD; increasing cognitive deficits emerge; inability to recall important details such as address, telephone number or schools attended, but memory of information about self & family remains intact; disorientation & confusion as to time & place

Discuss the nursing interventions are for the patient with Parkinson's disease.

Monitor neuro status for changes, monitor respiratory status for changes, encourage self-care, allow patient extra time, encourage exercise; assist with passive ROM if necessary, weigh patient; I&O; explain importance of following med schedule as well as effects of medication wearing off; reduce falls at home.

What are the risk factors for Guillain Barre Syndrome?

More common among the 20-50 age group, associated with swine flu immunizations, frequently preceded by mild respiratory or intestinal infection.

Mesial temporal sclerosis

Most common cause of intractable complex partial seizures in adults -Due to hippocampal neuron atrophy and gliosis -Tx: temporal lobectomy

Status epilepticus management

Most common causes: stroke, meningitis, encephalitis, hypoxic-ischemic encephalopathy 1. Ensure ABCs 2. Get labs and start normal saline 3. IV lorazepam, then phenytoin/fosphenytoin 4. If seizures persist -> additional IV fosphenytoin -> IV midazolam -> IV phenobarbital -> induce barbiturate coma

Ventral Spinal Nerves Function

Motor

Differentiate between MS, Myasthenia, and ALS.

Multiple Sclerosis (MS) starts in ages 20-50 usually, in females more than males. It is due to a demylization of the myelin sheaths of neuron cells in the CNS. Symptoms include extreme fatigue, dizziness, muscle twitching/spasms, numbness, tingling, loss of concentration, sensory and/or visual and/or speech impairment., depression. Myasthenia Gravis starts in ages 20-30 usually, and in females more than males. Autoantibodies from the thymus gland directed at acetylcholine receptor sites impair transmission of impulses across the myoneural junction. This reduces the number of receptor sites. The difference (from MS) is that M. Gravis does not affect the CNS, but instad the nerve-muscle communication point of the PNS. Symptoms include at first diplopia (double vision) and ptosis (dooping of eyelids), and often are accompanied by facial muscle weakness, speech and swallowing impairment, and generalized weakness of the muscles. It is purely a motor disorder and has no effect on sensation or coordination Amyotrophic Lateral Sclerosis (ALS) is a fatal disease of known cause. Death usually occurs as a result of infection, respiratory failure, or aspiration with an avg. time from onset of 3 years. There is a loss of motor neurons in the brain and spinal cord, which decreases function of all smooth and skeletal muscles. The muscles eventually atrophy. Symptoms depend on the location of the affected motor neurons, because spefic neurons activate specific muscle fibers Chief complaints are fatigue, progressive muscle weakness, craps, fasciculations (twitching), and incoordination.

List the common signs and symptoms of Parkinson's diseases.

Muscle rigidity Pin rolling Bradykinesia Stooped posture, and shuffling gait Difficulty swallowing

Discuss the community resources available to the patient with a neuromuscular disorder.

Muscular Dystrophy Association (MDA) Outpatient Therapy Support Groups

Juvenile myoclonic epilepsy

Myoclonic jerks of shoulders and arms that usually occur after awakening -Typical age of onset: 12-18 y.o. -Requires lifelong Tx w/AEDs (esp. levetiracetam)

What are the signs and symptoms of risk factors for retinal detachment?

Myopia, nearsightedness, seeing flashing spots, recent intraocular eye surgery, direct eye trauma, peripheral retinal holes, or history of retinal degeneration.

Who is retinal detachment more common in?

Myopic, nearsighted whites and men who suffered traumatic injuries.

Explain the etiology of each extra-pyramidal disorder.

Parkinson's Disease: chronic progressive degenerative neurologic disease that alters motor coordination. Myasthenia Gravis: chronic autoimmune disorder. MS: chronic degenerative disease that damages the myelin sheath aurrounding the axons of the CNS Huntington's disease: progressive neurologic disease. ALS: rapidly progressive, fatal neurologic disease.

Differentiate kinds of seizures by types and symptoms.

Partial seizures: simple partial seizures: uncontrolled jerking movements of a finger hand, foot, leg, or the face (jacksonian march). Complete partial seizures: repititive non-purposeful actions (lip-smacking) Generalized seizures: absence seizures: blank stare, blinking of the eyes, eyelid fluttering. Tonic-clonic seizures: sudden onset, most common seizure

Valproic acid

Partial, generalized seizure Broad spectrum AED that is also good for h/a but has s/e related to weight gain, tremors, alopecia

Discuss the precautions to teach when instructing a patient on anticoagulants.

Precautions: in pts with fever, heart failure, diarrhea, diabetes, malignancy, HTN, renal/hepatic disease, psychoses, depression, or spinal procedures. Interactions: aspirin, acetaminophen, NSAIDS, penicillin, aminoglycosides, tetracyclines, cephalosporins, beta blockers, loop diuretics, oral contraceptives, vitamin-K, barbiturates Contraindicated: hemorrhagic diseases, TB, leukemia, uncontrolled HTN, GI ulcers, recent surgery of eye or CNS, aneurysm. Use during pregnancy can cause fetal death. May be contraindicated with a hypersensitivity to pork products.

Discuss the nursing care for the patient with increased ICP.

Prevent IICP, and avoid the complications of IICP (ie; ineffective breathing patterns, cerebral edema, IICP, coma, brain herniation)

Discuss nursing care of patients with neurological infection or inflammatory disorder.

Preventing injury Monitor for decreased cerebral tissue perfusion. Preventing increased temperature Reducing headache Decreases enviornmental stimulation

Jacksonian march

Primarily motor seizure that begins in one focus but then spreads throughout the cortex -Often secondarily generalizes -> loss of consciousness and possible GTCS -Most often starts in the hand; face may be involved early

Describe the criteria for determining dosage of anticoagulant drugs. (PT, INR, APTT)

Prothrombin time (PT) and the international normalized ratio (INR) are used to monitor the pts response to warfarin therapy. The daily dose is based on these labs. Therapeutic range of the PT is 1.2 to 1.5 times the control value (11-13 seconds, think "pre teen"). INR should be maintained between 2 and 3. The The Activated Partial Thromboplastin Time (APTT) determines the overall capacity of the blood to clot for pts on heparin. 1.5-2.5 the control value (25-45 seconds, think "prime teaching time"). The APTT needs to be drawn q6hrs, heparin has a short half-life and so the amount can vary greatly within a short period of time. If the numbers are too low, they are at risk for clots. If too high, then they are at risk for bleeding. There is a narrow therapeutic range for anticoagulants.

What if you end up with a foreign body in your eye? And what if it were a chemical?

Put a dressing over both eyes and go to the ER. If chemical, then you should flush eyes first then head to the ER.

Who does ALL the care of patients with spinal cord injuries

RN or LPN

Discuss nursing care for the child with CP.

ROM exercises to prevent contractures, use special appliances to help the child perform ADLs, provide protective head gear and bed pads to prevent injury, provide a high-calorie diet because the child will have a high metabolism rate due to high motor function, explain the disorder and treatment to the family and that efforts should be made to ensure that the child reaches the optimal developmental level possible.

What are the people with myasthenia gravis always at risk for?

Respiratory Distress

What kind of death does a person with multiple sclerosis normally die from?

Respiratory. Watch for hypoxia, restlessness, and agitation.

What is the cause of subarachnoid hemorrhage?

Rupture of a blood vessel or an intracranial aneurysm, typically at circle of Willis or major cranial artery branches and commonly after head trauma

Recognize signs and symptoms of client's impaired cognition.

STAGE 1: 2-4 years after onset short term memeory loss; forgets locations and names of objects attempts to cover up memory loss has difficulty learning new information or making decisions decreased attention span can be angry or depressed antidementia meds are trying to prolong this stage. STAGE 2: 2-10 years after end of stage 1 unable to remember names of family members and gets lost in familiar locations easily agitated and irritable has difficulty using objects; reading, writing, and speaking cannot follow a conversation personal hygiene declines unable to make decisions (choose clothing) walks and unsteady gait, head down, shoulders bent, shuffles exhibits "sundowning" and wandering behavior STAGE 3: 1-3 years after stage 2 cannot recognize self or others inability to communicate has delusions and hallucinations bowel and bladder incontinence

Stage 6

Severe cognitive decline; moderately severe or mid-stage AD; memory difficulties continue to worsen loss of awareness of recent events & surroundings; may recall own name, but unable to recall personal history; significant personality changes are evident (delusions, hallucinations, & compulsive behaviors); wandering behavior; requires assistance w/usual daily activities such as dressing, toiling, & other grooming; normal sleep/wake cycle is disrupted; increased episodes of urinary & fecal incontinence

What kind of shoes should you recommend for a patient with Parkinson's Disease?

Shoes with 1 thread so they can slide through lie. No tennis shoes or leather bottom shoes. Slippers are good.

What is the short term treatment for diagnosis of myasthenia gravis? And for long term?

Short term is TENSILON. Long term treatment is PROSTIGMIN, an airway medication and MESTINON.

Discuss the common drugs, side effects, and precautions when administering drugs prescribed for extra-pyramidal disorders.

Sinemet: Dopaminergics - carbidoma-levodopa mixture. Levodopa is converted to doapaimine in the brain and carbidopa prevents levodopa from being destroyed. Comtan is used in adjunct to Sinemet sometimes. Enhances Tasmar: last resort Dry mouth/difficulty swallowing, anorexia, nausea, diskinesia vomiting, abdm pain and constipation, increased hand tremor, headache and dizziness. Caution should be used in combination with opioids, antacids, anticonvulsantsm and tricyclic antidepressants. Choreiform movements and dystonic movements are the most adverse reaction to levodopa.

a,b,c,d,e

Social effects of a chronic neurologic disease include (select all that apply) a. divorce b. job loss c. depression d. role changes. e. loss of self esteem

Discuss drugs used to treat seizures by: name, action, adverse reactions and special precautions.

Sodium Luminal (Phenobarbital) Diphenylhdantin (Dilantin) Mephenytoin (Mesantoin) Valproic Acid (Depakene) Carbamazine (Tegretol)

Identify common drugs in the treatment of TIA.

The most frequently used anti-platelet medication is aspirin. Aspirin is also the least expensive treatment with the fewest potential side effects. An alternative to aspirin is the anti-platelet drug clopidogrel (Plavix). Also maybe Aggrenox, a combination of low-dose aspirin and the anti-platelet drug dipyridamole, to reduce blood clotting. The way dipyridamole works is slightly different from aspirin. Ticlid is used when there is an aspirin allergy, are are used with aspirin in order to avoid clots from forming on coronary stents. Persantine is also an antiplatelet used. Anticoagulant drugs include heparin and warfarin (Coumadin). Heparin is used short term and warfarin over a longer term. These drugs require careful monitoring. If atrial fibrillation is present, may prescribe another type of anticoagulant, dabigatran (Pradaxa). Think +10 to differentiate PTT from PT. (C+O+U+M+A+D+I+N + 2. PT.) Vitamin K is the antidote for Coumadin, and Protamine is the antidote for heparin. Heparin should only be used parenterally. Lovenox is used instead of Heparin during pregnancy (does not pass the placenta) and is more long term than Heparin, although not as long-term as Coumadin unless in a LTC. It is a type of Heparin. All are used prophylactically for DVTs, PEs.

Explain the nursing care of the patient with a CVA.

The priority of care during the initial period is preserving functional brain cells and preventing acute complications. Once the client's condition is stable, problems of physical mobility, communication, sensory-perceptual deficits, bowel and urine eliminations, and swallowing present the major nursing challenges. Diags: Ineffective Tissue Perfusion: Cerebral, Risk for Ineffective Airway Clearance, Impaired Physical Mobility, Impaired Verbal Communication, Disturbed Sensory Perception, Impaired Urinary Elimination and Constipation, Impaired Swallowing, Self-Care Deficit

Describe common symptoms of a TIA.

The warning signs of a TIA are exactly the same as for a stroke. Sudden numbness or weakness of the face, arm or leg, especially on one side of the body Sudden confusion, trouble speaking or understanding Sudden trouble seeing in one or both eyes Sudden trouble walking, dizziness, loss of balance or coordination Sudden, severe headache with no known cause

What is negative pressure wound care?

The wound is filled with a sponge and a vacuum is applied via a closed system to remove stagnant fluid and decompress interstitial space.

What should you teach your patient for post op care after cataract surgery?

They should wear sunglasses when in the sun and avoid sneezing or vomiting. If they have pain in the eye, they should call the physician and they should wear 1 eye patch over the affected eye at night. One word = one patch.

Describe common diagnostic tests for the patient with neurological manifestations.

X-rays CT Scans MRI Cerebral angiography: contrast material is injected and an combined X-ray and fluroscopy is performed. Myelography: X-ray of spinal cord and canal after contrast media is injected. PET: radioactive agent is injected and CT measures metabolic activity of the brain. Ultrasound Carotid duplex study: sound waves identify blood flow velocity to determine the presence of occlusive vascular disease. EEG EMG: needles inserted in muscles to record electrical activity. Evoked potentials: electrodes are placed on scalp and skin to record the visual or auditory stimulus along sensory pathways

Do you want bleeding when a wound is being debrided?

YES, yes because it means that there is a good supply of blood and healing is taking place.

Epidural or subdural sensor

a fiber optic sensor put into epidural space via burr hole. Uses light sensors to measure ICP. Does not penetrate dura.

intraventricular catheter (ventriculostomy)

a fluid filled catheter inserted in to lateral vetricles via burr hole in skull. Cath attached to CSF drainage system & transducer monitor

Subarachnoid screw or bolt

a hollow bolt put into subarachnoid space via burr hole in front of skull and connected to fluid filled transducer. LEVEL transducer to approximate location of lateral ventricles.

CT Scan

a series of x-ray photographs taken from different angles and combined by computer into a composite representation of a slice through the body.

Doppler Ultrasound

a study that uses sound for detection of blood flow within the vessels; used to assess intermittent claudication, deep vein thrombosis, and other blood flow abnormalities

Tapering Medications

a. in some patients who have gone for a length of time with no seizures, the seizure focus resolves; no test can reliably determine if this has happened, so patients must recognize it is a gamble, but often one worth taking (with physician supervision)

S/E of all AEDs

a. sleepiness b. nausea c. dizziness d. diplopia e. ataxia f. headache g. rash (**) h. TERATOGENICITY! (esp valproic acid)

Discuss the common causes of a CVA.

atherosclerosis of large cerebral arteries (thrombotic) a-fib, CHF, endocarditis, rheumatic heart disease, mitral valve disease (embolic) HTN (hemorrhagic) risk factors: male, over 65 years of age, african american, hypertension, DM, obesity, a-fib, atherosclerosis, smoking, high cholesterol diet, excessive use of alcohol, cocaine/heroin, oral contraceptives. Another common cause of intracerebral hemorrhage in the elderly is cerebral amyloid angiopathy, which involves damage caused by the deposit of beta-amyloid protein in the small and medium-sized blood vessels of the brain

Discuss functional abilities related to area of spinal cord injury.

c1-c3 = no movement or sensation below the neck; ventilator-dependent c4 = movement and sensation of head and neck; some partial function of the diaphragm c5= controls head, neck, and shoulders; flexes elbow c6 = uses shoulder, extends wrist c7-c8 = extends elbow, flexes wrist, some use of fingers T1-T5 = has full hand and finger control, full use of thoracic muscles T6-T10 = controls abdominal muscles, has good balance T11-L5 = flexes and abducts the hips; flexes and extends the knee S1-S5 = full control of legs; progressive bowel, bladder, and sexual function

Signs and Symptoms of PD (13)

classic triad (tremors, rigidity, kinesia) bradykinesia akinisea dyskenesia stupor mask like face, poor blink reflex, wide open eye unsteady on feet shuffling gait cognitive impairment drooling loss of postural control slowed, monotonous speech dysphagia

Cerebrum; Cerebral Lobes control

complex problem solving, value judgements, language, emotions, visual image interpretation, touch, pressure, and temp

Dysphagia

condition in which swallowing is difficult or painful

Sensory deprivation

condition resulting from decreased sensory input or input that is monotonous, unpatterned, or meaningless.

Sensory overload

condition resulting from excessive sensory input to which the brain is unable to meaningfully respond

Expressive Aphasia

damage to Broca's area can cause this condition in which person cannot talk, though understand speech

Describe s&s of a spinal cord injury.

depends on the location of the injury and other unique factors. general complications include hypotonia, autonomic dysreflexia, spinal shock, orthostatic hypotension, bradycardia, DVT, pressure ulcers, pain, limited chest expansion, pneumonia, stress ulcers GI, urinary incontinence, neurogenic bladder, UTIs, impotence, decreased vaginal lubrication, join contractures, muscle spasms, muscle atrophy, pathologic fractures, hypercalcemia spinal shock: temporary loss of reflex activity below the level of spinal cord injury, this usually happens 30-60 min after a complete SCI. There is loss of motor function, sensation, spinal reflexes, and autonomic function. other manifestations include bradycardia, hypotension, loss of sweating and temp control, bowel/bladder dysfunction, flaccid paralysis, loss of ability to perspire. could last from days to months before reflex activity returns. within the first year of injury even, the patient is at risk for spinal shock whenever they are moved.

Recognize changes in neurological status.

glasgow's coma scale, A&O x3, widening pulse pressure, abnormal body posturing, cushing's triad, cranial nerve checks, confusion, hallucinations, out of control emotions diagnostic tests for head injuries blood glucose, ABGs, tox screen, creatinine, BUN, liver function tests, CBC + diff, CT, MRI, LP, cerebral angiography, xray of the brain will be able to determine where the injury is, how big it is with an LP, encourage fluid intake - CSF reproduces after 24h hours when an LP is done, there is a space in the spinal column. pat may complain of HA because the air from that space naturally goes upwards, in this case towards the head. to test to see if leakage is CSF, check for glucose - see halo on gauze.

Describe nursing measures in response to unexpected negative response (bleeding).

hold next dose immediately, call PHP if bleeding will not stop after 10min of pressure if external, be prepared to administer antidote, FFP, or other drugs as ordered by PHP.

Distinguish the characteristics of right and left hemiplegia.

left hemisphere lesion: right hemiplegia, right visual field deficits, aphasia both expressive and receptive, agrahia - difficulty writing, alexia - reading problems, aware of deficits, impaired intellectual ability, no memory deficits, no hearing deficits, deficits in the right visual field as reading, problems and inability to discriminate words and letters, behavior slow cautious and disorganized, anxious when attempting new task, depression, sense of guilt, quick anger and frustration, feeling of worthlessness, worries over the future right hemisphere lesion: left hemiplegia, left visual deficits, disoriented to time place and person, cannot recognize faces, spatial - perception deficits, neglect of left side, patient unaware of paralyzed side, loss of depth perception, impulsive - easily distracted, unaware of neurological deficits, confabulates, euphoric impaired sense of humor, constantly smiles, denies illness, poor judgement, overestimates ability, loss of ability to hear tonal variations

Describe a therapeutic environment for a patient with Alzheimer's disease.

make schedule of the client's daily activities, label drawers containing client's clothes and label rooms, use communication techniques to the client's level of ability, when the client is agitated re-direct attention, if pt wanders they need a MedicAlert, schedule rest periods or quite times throughout the day, set boundaries by placing red or yellow tape on the floor, assign the same caregivers as much as possible, music/art therapy, orient to person place and time if needed.

Parasympathetic

peace

Symptoms of ICP

severe headache, deteriorating LOC, restlessness, irritability, dilated or pinpoint pupils, slow rxn time, altered breathing pattern, deteriorating motor function, abnormal posturing (decorticate, decerebrate, flaccidity)

Describe four types of CP.

spastic (most common): the cortex is affected resulting in the child having a scissor-like gait where one foot crosses in front of the other foot. other s&s: underdeveloped limbs, increased deep tendon reflexes, contractures, involuntary muscle contraction and relaxation, flexion. athetoid: the basal ganglia are affected resulting in uncoordinated involuntary motion. other s&s: uncontrolled involuntary movements, drooling writhing, all extremities move with voluntary movement, difficulty swallowing, facial grimacing. ataxic: the cerebellum is affected resulting in poor balance and difficulty with muscle coordination. other s&s: wide-based gait, unsteadiness, clumsiness, poor balance, unnatural muscle coordination.

Sympathetic

stress

List common signs and symptoms of head injury.

the most common general symptoms: Impulsive behavior Loss of memory Impaired perception Personality changes Loss of taste and smell Diminished concentration Hearing and balance disorders Cognitive fatigue Concussion Coma Epilepsy open head injury: open wound on head, no nerves receptors so patient might not even realize the extent of injuries. Most open head injuries expose the brain to the outside environment, leaving victims extremely susceptible to infection (meningitis). closed head injury: Loss of consciousness Dilated pupils Respiratory issues Convulsions Headache Dizziness Nausea and vomiting Cerebrospinal fluid leaking from nose or ears Speech and language problems Vision issues scalp injury: concussion: immediate loss of consciousness for <5min. drowsiness, confusion, dizziness, HA, blurred or double vision. contusion: varies with the size and location of injury. initial loss of consciousness;if LOC remmains altered, client may become combative. During unconsciousness, lies motionless; has pale, clammy skin; faint pulse; hypotension; shallow resps; altered motor responses. epidural hematoma: brief loss of consciousness followed by a short period of alterntess. the client rapidly progresses into coma with decorticate or decerebate posturing, ipsilateral pupil dilation, and seizures. subdural hematoma: acute - rapid deterioration from drowsiness and confusion to coma, ipsilateral pupil dilation and contralateral hemiparesis subacute - appear 48 hours - 2 weeks later; alert period followed by slow progression to coma chronic - develops within weeks/months after initial injury. slowed thinking, confusion, drowsiness; may progress to pupil changes and motor deficits intracerebral hematoma: decreased LOC; pupil changes and motor deficits.

Sensory reception

the process of receiving data about the internal or external environment through the senses

Explain the use of thrombolytic drugs in the treatment of a CVA.

thrombolytic drugs dissolve blood clots that have already formed within the walls of a blood vessel. is prescribed after an ischemic stroke has occurred, within 3 hours of onset this therapy is given. 0.9mg/kg, 10% given IV bolus over one minute, the rest given over 60 minutes. criteria for receiving thrombolytic drugs: • Age 18 years or older • Clinical diagnosis of ischemic stroke • Time of onset of stroke known and is 3 hours or less • Systolic blood pressure <185 mm Hg; diastolic <110 mm Hg • Not a minor stroke or rapidly resolving stroke • No seizure at onset of stroke • Not taking warfarin (Coumadin) • Prothrombin time <15 seconds or INR <1.7 • Not receiving heparin during the past 48 hours with elevated partial thromboplastin time • Platelet count >100,000/mm3 • No prior intracranial hemorrhage, neoplasm, arteriovenous malformation, or aneurysm • No major surgical procedures within 14 days • No stroke, serious head injury, or intracranial surgery within 3 months • No gastrointestinal or urinary bleeding within 21 days assess q 15min for first hour, then every 15-30 minutes for the next 8 hours, then at least q4hrs. bleeding and IICP are side effects to monitor for. draw CBC before starting thrombolytics patients is critical and cared for in ICU for 48 hours used as soon as possible after formulation of clot

4

which is the best response by the nurse if the client fails to follow the information or teaching provided? 1. give up because the client doesnt want to change 2. develop a tough approach 3. guide the client to create a plan of action 4. Remind the client of previous successes.

Discuss alternative communication methods with an aphasic patient.

• Face the patient and establish eye contact. • Speak in a normal manner and tone. • Use short phrases, "yes" and "no" questions, and pause between phrases to allow the patient time to understand what is being said. • Limit conversation to practical and concrete matters. • Use gestures, pictures, objects, and writing. • As the patient uses and handles an object, say what the object is. It helps to match the words with the object or action. • Be consistent in using the same words and gestures each time you give instructions or ask a question. • Keep extraneous noises and sounds to a minimum. Too much background noise can distract the patient or make it difficult to sort out the message being spoken. • Ask them to nod the head or blink their eyes, provide pad and pencil, magic slate, flash cards, computerized talking board, and/or pictures boards to help with communication.

Adverse Effects

• behavioral problems • rare psychosis

Advantages and DIsadvantages of Topirimate

• broad spectrum • doesn't alter other AEDs • weight loss Disadv - slow to load; unresponsive

Advantages and Disadv of Levetiracitam

• broad spectrum • no drug-drug interactions • relatively rapid titration Disadv - expensive

Disadvantages

• hepatic enzyme (CP450) inhibitor • drug interactions

Adverse effects of Topirimate

• metabolic acidosis (carbonic anhydrase inhibitor) • kidney stones • cognitive slowing, word-finding difficulty • glaucoma (rare)

Advantages of Gabapentoin

• no drug interactions • non-hepatic metabolism • easy to load orally • very safe, well tolerated

Advantages of pregabalin

• no drug interactions • non-hepatic metabolism • easy to load orally • very safe, well tolerated; effective at lower doses than gabapentin • linear kinetics

Disadvantages of Gabapentoin

• saturable absorption • short half-life • may be less effective than other AEDs • expense

Disadvantages

• short half-life • may be less effective than other AEDs • expense

Disadvantages of Lamotragine

• slow to load (especially in patients already taking valproate) • expensive • not good drug if need to get pt on seizure med ASAP (but can introduce later) May induce bad rash

Advantages

• very broad spectrum (good if you don't know what spec type of seizure) • good for migraines

S/E

• weight gain • peripheral edema

Adverse Effects

• weight gain • tremor • alopecia • thrombocytopenia • pancreatitis • hyperammonemia


Conjuntos de estudio relacionados

CFP module 5 estate planning quizzes

View Set

B101 Previous Quizzes Study Guide

View Set

Quick Recall Arts & Humanities Questions

View Set

SOC 110 Ch. 6 Knowledge Development

View Set